Reeds 1 Mathematics

You might also like

Download as pdf or txt
Download as pdf or txt
You are on page 1of 249

CONTENTS

CHAPTER 1-ARITHMETIC
Powers and roots. Surds. Ratio and proportion.
Method of unity. Variation. Percentage.
Constituent parts. Averages. Logarithms;
common, to different bases, natural (Naperian). ... 1-2 1

CHAPTER 2-ALGEBRA
Addition and subtraction, collection of terms,
powers and roots, multiplication and division.
Removal of brackets. Factorisation. Lowest
Common Multiple. Fractions. Remainder and
Factor theorems. Binomial theorem. . . . . . . . . . 23-44
CHAPTER 3-SIMPLE EQUATIONS
Problems involving simple equations.
Transposition and evaluation of formulae.
Logarithmic equations. . . . . . . . . . . . . . . . . . . 45-58
CHAPTER &SIMULTANEOUS LINEAR EQUATIONS
Various methods of solving. Examples involving
two or three unknowns. ............... 59-67
CHAPTER 5-QUADRATIC AND CUBIC EQUATIONS
Solutions by factorisation, completing the square,
and formula. Equations reducible to quadratics.
Simultaneous quadratics. Problems involving
quadratic equations. Cubic equations. . . . . . . . . . 69-86

CHAPTER M R A P H S
Plotting. Determining equation to straight line
graph. Graphical solution of simultaneous linear
and quadratic equations. Determination of laws.
Curve sketching. . . . . . . . . . . . . . . . . . . . . . 87-1 07
CHAPTER 7-TRIGONOMETRY AND GEOMETRY
Measurement of angles. Trigonometric ratios.
Complementary and supplementary angles.
Theorem of Pythagoras. Identities. Reading
ratios. Angles greater than 90". Graphical
representation. Latitude and longitude. Compass
bearings. Geometrical construction of triangles.
Some important geometrical facts. Crossed
viii CONTENTS CONTENTS ix

chords. Cyclic triangles and quadrilaterals. SOLUTIONS TO TEST EXAMPLES


.................. 279-399
Circumscribed and inscribed circles. Similar
.........
triangles. Congruent triangles. . . . . . . . . . . . . 109-142 A SELECTION OF EXAMTNATION QUESTIONS 401417
SOLUTIONS TO EXAMMATION QUESTIONS ......... 419 4 91
CHAPTER 8-SOLUTION OF TRIANGLES
Ratios for right angled triangles. Triangles other INDEX
.................................... 493-495
than right angled. Sine rule and cosine rule.
Areas of triangles. Equilateral and isosceles
triangles. Circumscribed and inscribed circles.
Compound and double angles. . . . . . . . . . . . .

CHAPTER 9-MENSURATION OF AREAS


Parallelogram, Rhombus. Trapezium. Triangles.
Polygons. Circle, sector and segment. Ellipse.
Surface areas of cylinder, sphere, cone and
frustum. Theorem of Pappus. Similar figures.
Irregular figures, Simpson's and mid-ordinate
rules. . . . . . . . . . . . . . . . . . . . . . . . . . . .

CHAPTER ICkMENSURATION OF VOLUMES AND MASSES


Volume, mass, density and specific gravity.
Volumes of prisms, pyramids, frustums and the
sphere. Segment of a sphere. Theorem of Pappus
applied to volumes. Force, weight and centre of
gravity (mass). Applications. Similar solids.
Simpson's rule applied to volumes. Flow of liquid
through pipes and valves. First moments for
centre of gravity. . . . . . . . . . . . . . . . . . . . . .

CHAPTER 11-DIFFERENTIAL CALCULUS


Gradient, zero gradient, gradient of a curve.
Differential coefficient, second differential
coefficient. Distance, velocity and acceleration.
Maxima and minima. Differential coefficient of
sin x, cos x , In x, ex. Functional notation. . . . .

CHAPTER 12-INTEGRAL CALCULUS


Integration, constant of integration. Integration
of sin x , cos x, I / x , ex. Area by integration.
Definite integral integration as a summation.
Volume of solid of revolution. Distance and
velocity by integration. ...............
RULES AND FORMULAE
Selection of rules and formulae from the
Chapters. ........................
CHAPTER 1

ARITHMETIC
POWERS
An index is a short method of expressing a quantity multiplied by
itself a number of times, thus,
4 x 4 is written 4', this is the 'second power' of 4 commonly
called the 'square' of 4.
x x x x x is written x3, this is the 'third power' of x commonly
called the 'cube' of x.
10 x 10 x 10 x 10 x 10 x 10 is written lo6, and so on.

TO MULTIPLY POWERS OF THE SAME QUANTITY, add their indices, thus,

23 24
written right out = 2 x 2 x 2 x 2 x 2 x 2 x 2
= 27

The same result is obtained by adding the indices 3 and 4.

TO DMDE POWERS OF THE SAME QUANTITY, subtract their indices, thus,


35 t 32

The same result is obtained by subtracting the indices,


35 + 32
-
- 35-2

= 33
2 R E E D ' S MATHEMATICS FOR E N G I N E E R S

NEGATIVE INDICES
RAISE A POWERED QUANTITY TO A FURTHER POWER, multiply their
Consider the example 52 t 56 indices, thus,

written right out - 5x5 Q213


5~5X5X5X5X5 = 22 x 22 x 22
1 written right out
=2X2X2X2X2X2
= 26
or, subtracting the indices,
therefore
-
-2 2~3

= 26
Hence a quantity with a negative index, in this case 5-', is equal to the
reciprocal of that quantity with a positive index, that is,
ROOTS
A root is the opposite of a power and the root symbol is J.
2,/ represents the 'square root' of a quantity and means that it is
Similarly, x-2 1
=-
x2 required to find the number which, when squared, will be equal to that
quantity.
~ h u s2fi
, = 5 because 52 is 25.
1 J represents the 'cube root' of a quantity, this means that it is
Likewise, -
J = x3 required to find the number whose cube is equal to that quantity.
Thus, 3&7 = 3 because 33 = 27.
Similarly, 4 ~ = 2
5m=3
POWR OF U N I Any
~ quantity to a power of unity is the quantity itself,
The square root of a quantity is usually written without any figure
thus.
indicating the root, thus the square root of 64 is usually written
instead of 2&i. The value of any other root must of course be clearly
stated.
Another method of indicating a root of a quantity is by expressing it
as a power equal to the reciprocal of the root, for example,
the square root of 49 may be written,
ZERO POWER. Any quantity to a power of zero is equal to unity, thus,
or 49f, which is 7;
23 + 23 = 23-3 = 20
the cube root of 27 may be written,
20 = I
3J27or 274, which is 3;
similarly, 2 s 2 = P2 = x0
the fourth root of x may be written,
x0 = I
4Jior xi
4 R E E D ' S MATHEMATICS FOR E N G I N E E R S ARITHMETIC 5

The square of the cube root of 64 may be written, Memorise:


(3J64)2 or 64$,which is 16.
f i = 1.414
The root of a minus quantity is inadmissible.
.J"; = 1.732
POWERS AND ROOTS OF W L G A R FRACTIONS

The power or root of a vulgar fraction is equal to the power or root


of the numerator divided by the power or root of the denominator.
!

62 36
or -=-=9
22 4
Example:

2x45
or - - - -
,A - 2 - 842
2x
The advantage of applying the above rule will be seen in the -
- 1/2cancels top and bottom
following examples, &xJ5
2
I = - now multiply top and bottom by
I J5
- on the bottom line x =3
f i x &

--2 x JS
-
I~ 3m
- - - - s
3
- 2 x 1.732
3m-3-I;
3
1 3.464
I --
-
SURDS 3
= 1.155 (to nearest four figures) Ans.
1 A root which does not work out exactly is termed a surd, such as:
I
I f i = 1.41421. . . etc.
= 1.414 to nearest four figures. RATIO
similarly, &,6, &,are all surds. A ratio is a comparison of the magnitude of one quantity with another
It will be found very useful if, at least, the values of JZ and to quantity of the same kind; it expresses the relationship of one to the
the nearest four figures, were committed to memory. Many useful other and therefore may be stated in fractional form. Being a means
manipulations can be performed easily and quickly when solving or of comparison it is often convenient to express a ratio in of
I
simplifying expressions which include these quantities as factors, unity. The ratio sign is the colon:
6 R E E D ' S MATHEMATICS FOR E N G I N E E R S ARIT H M E T I C 7

Thus, if the lengths of two bars are 250 mm and 2 m respectively,the ~~t =time (in min) to deliver 6000 1,
ratio of one to the other may be expressed, Ratio of times taken :: Ratio of quantities delivered.
250 : 2000 (note both quantities must be expressed in the
or, 1:8 same units in this case both in mm)
x x 4400 = 55 x 6000

-
or, the former is of the length of the latter.
55 x 6000
x=
Example. The mass of a solid shaft 300 mm diameter is 1200 kg. 4400
Another shaft is hollow, 300 mm outside diameter and 150 bore x = 75 min Ans.
and of the same length as the solid shaft. If the mass of the hollow shaft
is 900 kg, express the ratio of the mass of the hollow shaft to that of
the solid shaft.
E ~ ~ The
~ ratio
] ofthe
~ . areas of two circles is equal to the ratio of
Ratio of masses. hollow : solid the of their diameters. The diameter of one circle is 20 mm and
its area is 3 14 m 2 , another circle is 30 mm diameter, find its area.
900 : 1200
Let x represent the area of the second circle,
Ratio of areas :: Ratio of diameters2

PROPORTlON
Proportion is an equation of ratios, that is, it expresses that the ratio
of one pair of quantities is equal to the ratio of another pair. The
proportion sign is the double colon: : but the equal sign may be used.
For example, the ratio of 5 to 10 is equal to the ratio of 20 to 40, and
this may be expressed,
5 : 10:: 2 0 : 40
or, 5 : 10 = 20: 40
5 20 INVERSE PROPORTION. The above examples are cases of direct
or, -- proportion because, in the pump question for instance, an increase in
10-40
time of running results in an increase in quantity of water delivered,
It can also be seen that the two inside terms multiplied together is equal
thus, the quantity of water delivered varies directly as the time. There are
to the two outside terms multiplied together, thus,
however, many cases where the increase in one quantity causes a
decrease in another, this is inverse proportion where one quantity is
stated to vary inversely as the other. Suppose one pump could empty a
this is sometimes expressed in the manner 'the product of the means
is equal to the product of the extremes', and is a usefbl method to
tank in 20 min then two similar pumps drawing from this tank could
find that missing term when one of the terms is unknown. empty it in half the time, i.e., 10 min, or three pumps would do the work
ne-third of the time. Here the greater the number of pumps, the less
e taken; the time varies inversely as the number of pumps. In
Example. A pump takes 55 min to deliver 4400 1 of water. Under
similar conditions, how long would it take to deliver 6000 l? g down such a problem by the proportion method, one of the pairs
f ratios must be reversed.
8 R E E D ' S M A T H E M A T I C S FOR E N G I N E E R S ARITHMETIC 9
METHOD OF UNITY VARIATION
It is often more convenient to apply the method of unity to proportion variation is a further step in ratio and proportion. As previously
problems, especially when dealing with compound proportion in which npl&d, when an increase of one quantity depeqds upon the increase
there are more than two pairs of quantities. The following examples of another, one is said to vary directly as the other; when an increase
demonstrate working by this method. in one quantity depends upon the decrease of another, one is said to
inversely as the other. The variation sign is a, thus, if the cost of
Example. If 8 men can erect 2 engines in 18 d, how long would it casting a propeller varies as its mass, this may be written:
take 12 men, working at the same rate, to erect 5 similar engines?
cost oc mass
Time for 8 men to erect 2 engines = 18 d
and, if the time to travel a certain distance varies inversely as the speed,
Time for 1 man to erect 2 engines = 18 x 8 d this may be written:
18 x 8 1
Time for 1 man to erect 1 engine = - time oc-
2 speed
18 x 8
Time for 12 men to erect 1 engine = - Taking a simple numerical example on the first case: The cost of a
2 x 12 propeller of 2 t mass is f 800, the cost of another propeller of 3 t mass is
Time for 12 men to erect 5 engines = 18 x 8 x 5 £1200,
2 x 12
1st propeller, - = -- f4OOjt
= 30 d Ans. mass 2
cost - 1200
2nd propeller, -- ---- = £400/t
Example. A general service pump can pump out a tank in 10 h mass 3
and the ballast pump can pump out the same tank in 5 h. If both pumps cost
are working together, how long will it take to empty the tank? hence, -= a constant amount
mass

Quantity pumped by G.S. pump in 10 h = whole tank cost, cost2


therefore, -
mass, mass2
Quantity pumped by G.S. pump in 1 h = & of tank
Quantity pumped by ballast pump in 5 h = whole tank For the second case take the example: A train travels a certain
Quantity pumped by ballast pump in 1 h = of tank : distance in 5 h when travelling at an average speed of 60 km/h; another
train takes 4 h over the same journey travelling at an average speed of
Quantity pumped in 1 h by both pumps working together 75 km/h.

1st train, speed x time = 5 x 60 = 300


- ,
- 3 of the tank
2nd train,speed x time = 4 x 75 = 300 km
.'. Time to empty = h hence, speed x time = a constant (distance)
= 3 f h Ans. therefore, speed, x time, = speed2 x time2
10 R E E D ' S MATHEMATICS FOR E N G I N E E R S

Summing up, (i) when one quantity varies directly as another, their
quotient is constant; (ii) when one quantity varies inversely as another
their product is constant.

Example. Within a certain range, the power of an engine varies


directly as the mass/h of fuel burned. An engine uses 180 kg/h of fuel
when the Power developed is 800 kW, what will be the fuel consumption
when the power is 900 kW? resistance cx length
1
resistance cx --
area
also,
fuel/h cx power
resistance x area = a constant
. fuel/h length
. . -- - a constant
power
. fuel, fuel,
. . -- --
power, power,
180 fuel,
--
800 - 900

.'. fuel, = 180 x 900 R,=lOQ Ans.


800
= 202.5 kg/h Ans.

Example. If a gas is maintained at the same temperature its pressure


varies inversely as its volume. 6 m3 of gas at atmospheric pressure is PERCENTAGE
compressed until its pressure is four times as much, the temperature
Percentage is another method of expressing a ratio in fractional form,
remaining unaltered, find the volume of the compressed gas.
using 100 as the denominator, and substituting the denominator of 100
with the percentage symbol %.
1
pressure cx --- Thus, the ratio of 4 to 25
volume
pressure x volume = a constant
can be written 4
25
in fractional form

16
which is equal to - on a denominator of 100
100
and written 16% in percentage form.

Since a fraction is part of unity, and a percentage is part of one


hundred, a fraction is converted into a percentage by multiplying the
12 R E E D ' S MATHEMATICS FOR E N G I N E E R S ARITHMETIC
13
fraction by 100. Conversely, a percentage is converted into a fraction by CONSTITUENT PARTS
dividing by 100. Thus:
me ~ p l mwhich constitute a whole, such as the quantities of metals
d e d together to form an alloy, may be expressed in the form of ratios,
i=33f% fractions or percentages, and it is often convenient to convert one form
;=37;?4 im another when solving problems. Consider a brass consisting of two
pmcopper and one part zinc, in every 3 g of brass there will be 2 g of
To express percentage increase or percentage decrease: copper and I g of zinc, therefore two-thirds (i.e. 6 q h ) of the total mass
ofthe b r ~ iss copper and the remaining one-thud (i.e 33@) is zinc.
% increase = total increase Hence to convert ratio of parts into fractional amounts, add the ratios
original amount x 100 to@er to produce a common denominator, and then each ratio forms
% decrease = total decrease x 100 its own respective numerator. The fractions are multiplied by 100 to
original amount convert them into percentages.

Example. A fusible metal is composed of 5 parts bismuth, 3 parts


Example. (i) A spring of original length 80 mm, is stretched to a
lead and 2 parts tin. Express this ratio as fractional quantities and
length of 100 mm, what is the percentage increase in length?
percentages, and find the mass of each element required to produce
(ii) Another spring of original length 100 rnm, is compressed to a 1.5 kg of fusible metal.
length of 80 rnrn,what is the percentage decrease in length?
Ratio, bismuth : lead : tin = 5 : 3 : 2
(i) actual increase = 20 mm
Surnof ratios = 5 + 3 + 2 = 10
20
O h increase = - x 100
80 .: mass of bismuth = & = 3 = 50% of total mass
= 2 5 Ans.(i) mass of lead = $ = 30% of total mass

(ii) actual decrease = 20 mm


6
mass of tin = = f = 20% of total mass
To produce 1.5 kg = 1500 g of metal:
20
Oh decrease = -x 100 mass of bismuth = x 1500 = 750 g
100
= 20 Ans. (ii) mass of lead = $ x 1500 = 450 g
mass of tin = x 1500 = 300 g
Example. The ratio of the mass of engine A to engine B is 5 to 3. mass of fbsible metal = sum = 1500 g.
Find (i) by what percentage is A heavier than B, (ii) by what percentage
B is lighter than A.
Example. In a certain two-cylinder engine, the power developed
Difference = 5 - 3 = 2 in No. 1 cylinder is 20% more than the power developed in No. 2
cylinder. What percentage of the total power of the engine is developed
(i) % increase of A over B = - 2 x 100 = 662 in each cylinder?
3 3
.'. A is 664 % heavier than B Ans. (i) Ratio of powers, No.1 : No. 2
(ii) % decrease of B over A =- 2 x 100 = 40 = 120 : 100
5 = 1.2: 1
.'. B is 40% lighter than A Ans. (ii)
Sum of ratios = 1.2 + 1 = 2.2
R E E D ' S MATHEMATICS FOR E N G I N E E R S

:. percentage of total power developed in No. 1 cylinder

--
-
1 a2 x 100 = 54.55 Ans.
2.2

.: percentage of total power developed in No. 2 cylinder = 11.2% Ans. (iii)

-- x 100 = 45.45
-
2.2
Ans. wd d e d away up the stack = 28 - 1 1.2 = 16.8%)
A 6heat-ba]ance' diagram can now be drawn to illustrate the
distribution of the heat in the fuel:

Example. Of the total heat in the fuel burned in the cylinders of a


TOTAL H E A T I N FUEL
diesel engine, 40% is converted into indicated power, 30% is carried S U P P L I ED 10 ENGINE
100%
away by the cooling water, 2% is lost by radiation, and the remainder
is carried away in the exhaust gases. (i) What percentage of the total
heat of the fuel does the heat in the exhaust gases represent? (ii) If 80%
of the indicated power is usefilly imparted to the crankshaft as brake
Power and the remainder lost in friction, what percentage of the total
heat of the he1 does the brake power represent? (iii) If 40% of the heat
in the exhaust gases is recovered in an exhaust gas boiler and the
remainder carried away up the stack, what percentage of the total heat in
I 1.2% 18-896
the he1 is recovered in the boiler?
The fill amount of heat in the fuel is represented by 100%. Fig. 1
Heat converted to ip = 40%

Heat carried away in cooling water = 30% AVERAGES


A he average (or mean) value of a group of collected quantities is
Radiation loss = 2%
obtained by dividing the total of the quantities by the number of
total = 72% quantities in the group.

Remainder = 100 - 72 = 28 Example. The distances covered by a ship on four successive


days were 320, 300, 310 and 330 naut. miles respectively. Find the
.'. Exhaust gases carry away 28% of the total heat Ans. (i) average day's run.
total distance
Average day's run =
bp = 80% of 40% total number of days
- 320 + 300 + 310 330 +
4
--
1260
4
Heat equivalent of bp = 32% Ans. (ii) = 3 15 naut. miles. Ans.
16 R E E D ' S MATHEMATICS FOR E N G I N E E R S ARITHMETIC 17

Example. A ship travels a distance of 15 naut. miles up-river LOGARITHMS


against the current, in 72 min, and then 15 naut. miles down-river with
the current, in 50 min. Calculators have replaced logarithms (and slide rules) for numerical
Find (i) the average speed up-river; calculations. However there are cases where logarithms still apply in
mathematics and in technological applications (see examples in later
(ii) the average speed down-river;
chapters).
(iii) the average speed over the double journey. The logarithm (log) of a number to a given base is the power to which
the base must be raised to be equal to that number.
distance 23 =8, in this case, 3 is the logarithm of 8 to the base of 2, and would
Speed = -
time be written log, 8 = 3. xn = N, this represents n as the logarithm of N to
Speed up-river = 15 naut. miles +-
IL
h the base of x, and would be written log, N = n.
60 The whole number part of the logarithm is called the characteristic
- 15 x 60
--
and can be positive or negative; the decimal part is called the mantissa
72 and is positive. Values can be read off a table of logarithms.
= 12 naut. miles/h Antilogarithm tables (if used) are the reverse to log tables and can be
= 12 $ knot Ans. (i) used to read off the number corresponding to the given logarithm. Log
and antilog values are directly obtainable by the use of a suitable
Speed down-river = 15 naut. miles t -50 h scientific calculator.
60
--
- 15 x 60 The rules for using logarithms are the same as those for other indices,
50 hence,
= 18 naut. miles/h (i) to multiply numbers, add their logarithms,
= 18knot Ans. (ii)
Average speed over the double journey log(x x y) = logx + logy
-
total distance
total time (ii) to divide numbers, subtract their logarithms,
122
= 30 naut. miles t - h
60 log(x s y) = log x - logy

(iii) to raise a number to a power, multiply the logarithm of the


= 14.75 knot Ans. (iii) number by the power,

The above example illustrates speed being obtained by dividing distance log(P) = (logx) x m
by time. Units of speed are usually expressed in km/h, or m/s.
Speeds of ships and aircraft are usually measured in knots (naut.
mileslh). One international nautical mile is equal to 1.852 krn. (iv) to extract a root of a number, divide the logarithm of the number
It is important to note how essential it it to divide one total by another by the root,
total to obtain an average. In the last example, total distance was divided
by total time to obtain the average speed over the double journey, it logmfi = (logx) + rn
would be wrong to take the average of the average speeds up and down
river. the resultant, in each case, is the logarithm of the answer.
18 R E E D ' S MATHEMATICS FOR E N G I N E E R S ARITHMETIC 19
-
COMMON LOGARITHMS (Ig)
(In)
NATURAL ~ A P E R I A N )LOGARITHMS
Are logs to base 10. Are logs to base 2.718 i.e. exponential, e or &, sometimes called
hyperbolic logs. From previous:
e.g. Ig of 5 to base 10 is 0.6990 because =5
lg of 400.0 = 1.6021

Ig of 0,006 = 5.7782 (bar three, 5, is - 3)


also Ig of 0.006 = -2.2218 (as a minus number)

Values (log and antilog) for numbers 1 to 10 if read directly from the
main table include the characteristic (only shown in the first column,
RELATION BETWEEN LOGARITHMS TO DIFFERENT BASES under the zero heading) with complementary tables (2) to allow greater
As previously explained, the logarithm (log) of a number to a given 0, less numbers to be evaluated (i.e. In 10"and In lo-"). The following
base is the power to which the base must be raised to be equal to that illustrative examples are solved using tables. They could also be
number. evaluated directly using a scientific calculator:
Then, if x is the log of N to the base a then, d =N (i) N = 7.754 In N = 2.0482
(ii) In N = 1.0794 N = 2.943
an4 if y is the log of N to the base b then, b" = N.
Hence, (iii) N = 4189 = 4.189 x lo3
In N = 1.4325 f6.9078
= 8.3403

(iv) N = 0.2418 = 2.418 x lo-'


X In N = 0.8829 + 3.6974
- = log, b
Y = 5.5803 (i.e. - 1.4197)
(v) In N = 8.4304
Look for next figure below, i.e. 6.9078 for lo3
8.4304 - 6.9078 = 1.5226 which reads 4.584

Example. Find the log of 8 to base 2 using common logs. (vi) In N = 4.7177 (i.e. - 3.2823)
Look for next figure below, i.e. 5.3948 for
lg 8 0.9030 4.7177 - 5.3948 = 1.3229 which reads 3.754
log, 8 =- = -- - 3 Ans.
lg 2 0.3010
N = 3.754 x lo-'
this is obviously correct because 23= 8. = 0.03754
20 R E E D ' S MATHEMATICS FOR E N G I N E E R S ARITHMETIC 21
Note. Use of a calculator simplifies the work in that the bar quantity is and broke when the gauge length was 62.5 mm and cross- sectional
not used because the minus number can be entered into directly.
area 48 rnm2. Find the percentage elongation and the percentage
reduction in cross-sectional area.

TEST EXAMPLES I 11. In a certain three cylinder engine, the power developed in No. 1
,ylinder is 15% more than in No. 3, and 5% less power is developed
104 x lo4 x 10; in No. 2 than in No. 3. What percentage of the total engine power is
1. Find the value of developed in each cylinder? Give the answers to the nearest one-
lo3 x lo$ x 102
tenth %.
42 X 4: x 4-2
2. Evaluate 12. A leaded yellow brass is composed of 71% copper, 1% tin, 3% lead
44 x 4i
and the remainder zinc. Find the mass required of each constituent,
8-' to make 500 kg of this alloy.
3. Find the value of -
8-5
13. The heights of an indicator diagram measured at regular intervals
(3;); x .J5 x 3 along its length are as follows: 27, 39, 47, 51, 48, 32, 20, 11, 8,
4. Evaluate 5 mm respectively. Find the mean height of the diagram in mm.
3-$ x 33

(2213 273
c
5. Find the value of (1 ;l2 + - +

m
,
14. 200 t of oil were bought at one port at £60/t and 600 t of oil at
another port at.£70/t. What was the average cost of oil/t?

15. Evaluate, In-


850
+ 840
-+ 0.6 In-
1160
6. Evaluate 492 850 850
4mx4t
7. The ratio of the volumes of two spheres is equal to the ratio of the 16. Find a multiplier to convert logs to base 10 into logs to base 2, and
cubes of their diameters; the volume of one sphere is 24.25 cm3, use this to find the log to base 2 of 1.5.
find the volume of another sphere whose diameter is twice as much.
17. Evaluate 0.06326-0.25
8. A pump can empty a tank in 12 h, another pump can empty the (a) using common logs,
same tank in 4 h, and another can empty this tank in 9 h. If all three (b) using natural (Naperian) logs.
pumps are set working together on this tank, how long would it
take to empty it? 18. Find the value of k in the following expression, using natural logs
only,
9. The strength of a beam varies directly as its breadth, directly as
the square of its depth, and inversely as its length. A beam is 5 m
long, 40 mm broad, and 100 mm deep; find the breadth of
another beam of similar material, 3 m long and 80 mrn deep, to
have equal strength.

10. A piece of mild steel 50 mm long between gauge points and


80 rnm2 cross-sectional area, was tested in a tensile testing machine
CHAPTER 2

ALGEBRA

Algebra is a convenient system of short-hand arithmetic using letters


of the alphabet to represent certain articles, quantities or hidden
numbers. For instance if, in a box containing 25 bolts, 34 nuts and 47
washers, add 15 bolts and 18 washers and take away 14 nuts, there
would then be in the box 40 bolts, 20 nuts and 65 washers. This could be
set down in the following manner:

25b + 3411+ 47w


add 15b- 14n+18w

result 40b + 20n + 65w


If the sign of the first term of an expression is positive it is usually
omitted as in the above example. Similarly for any single term, thus 5x
+
means 5x.
The number which multiplies a term is called a coefficient. In the
above example, the result line shows + 40 as the coefficient of b, 20+
as the coefficient of n, and + 65 as the coefficient of w.
An algebraic expression is a collection of terms. One term may be a
single quantity or a group of quantities multiplied (or divided) together,
terms are separated from each other by plus or minus signs. When a term
consists of quantities multiplied together, it is usual to omit the
multiplication signs, x x y x z is written xyz.
+ +
In the expression 2x2y 4xy 6xy2 there are three terms,

and 6 ~ ~ x 3

An algebraic expression which consists of two terms is called a


binomial expression, and one consisting of three terms is called a
trinomial expression.
Set the expressions with similar terms under each other, r -
the terms if necessary (they can be written in any order), e
written - 8~ - (-5x1
add the numbers of each column separately in the usual way. +
= - 8x 5x
= - 3x Ans. (iv)
Add together 2a + 76 + 3c and 6c + 50 - 2b.
2a + 7b + 3c
add 5a - 2b +6c
SuWact 6c - 3b + 4a - 7d from 56 + 3d + 2a - 9c
7a + 5b + 9c Ans. expnssions consisting of a number of terns, set down as shown
l&e terms under each other; to subtract, change the signs of
Example. Add together 3x - 5y + 6.2 and 4~+ 2y - gZ. Mmaobtractingexpression mentally, add and write down the results.
3x - 5y 62 + From 2 a + 5 b - 9c+ 3d
+
add 4x 2y - 82 Subtract 4 a - 3 b + 6c- 7d

7x - 3y - 22 Ans. - 2 a + 8 b - 15c+ 10d Ans.

SUBTRACTION
To subtract numbers or algebraic terms, change the sign of the term to COLLECTION OF TERMS
be subtracted and then add. 4
P
An expression may contain a number of similar terms, the first
Examples. ~ s in solving
s an algebraic problem is to simplify such an
(i) From 8x take away 5x expression by collecting like terms together and writing down the
6x~fessionin its simplest form.
written 8x - 5x
= 3x Ans. (i)

(ii) From 8x take away -5x


.k
5
+ +
Simplify 2x 5y 4x - 32 - 2y - z - x
Re-arranging, 2x + 4x - x + 5y - 2y - 3.2- z
written 8x - (-5x) +
Collecting like terms, 5x 3y - 4z Ans.
= 8x 5x +
= 13x Ans. (ii)

(iii) From - 8x take away 5x


1
POWERS AND ROOTS
written - 8x - 5x same rules apply with regard to powers and roots as for ordinary
therefore when observing the following examples, refer if
= - 13x Ans. (iii)
to the explanations given in Chapter 1.
26 R E E D ' S MATHEMATICS FOR E N G I N E E R S ALGEBRA 27

Examples. Examples.
multiplied by equals
multiplied by equals
multiplied by equals
multiplied by equals
divided by equals
divided by equals
divided by equals
(213 = ~ 2 =~x6 3 divided by equals

f i may be written xi
(3&)2 may be written xi Example.
x3 +.,I?1=x3-2 x1=x Multiply (2x + 3y) by (3x - 21,)
2+2=2-2=xo= 1
Procedure:
(i) set down in long multiplication style;
+
(ii) multiply 2x 3y by one term of the multiplier, say 3x;
+
(iii) Multiply 2x 3y by the other term, - 2y, putting like terms
MULTIPLICATION AND DIVISION under each other;
(iv) collect terms by addition.
When algebraic quantities are to be multiplied together, the
multiplication sign may be omitted, thus,

x x y may be writtenxy
2 x a x b may be written 2ab
(3x + +
y) x (x - 4y) may be written (3x y)(x - 4y)

When one quantity is to be divided by another, such as x divided 62 + 5xy - 6 9 Ans.


by y, it may be written in any of the following forms:
When the two expressions to be multiplied together each have only
two terms, the multiplication can quite easily be done mentally. In
the above example,

The signs are very important, the above are ordinary positive values
- A

but care must be taken when any of the terms are negative. The rule is: Note that the first term of the answer is the product of the first terms of
like signs multiplied or divided give a positive answer, unlike signs the two expressions, i.e., 2x x 3x = 6x2; the last term of the answer is
multiplied or divided give a negative answer. the product of the last terms, i.e., 3y x -2y= -6y2; the middle term
REED'S MATHEMATICS FOR E N G I N E E R S ALGEBRA 29

of the answer is the sum of the product of the 'means' and the product of (,i) repeat the operations as from (ii);
the 'extremes', i.e., 3y x 3x plus 2x x - 2y= 9xy - 4xy= + 5 ~ .
3 ~ - 2 ) 6 ~ ~2 - 1 4 ~ + 8 ( 2 $ + x - 4 Ans.
Now check the following:
6x3 - 4x2

+ +
(X 4)(x 5) = 2 9x 20 + + 3 2 - 14x
(4x+ 3)(2x - 1) = 8x2 + 2 x - 3
32- 2x
(h-5y)(x+3y)=Gfxy- 153
- 6y) = 152 - 38xy + 2 4 9
( 3 -~ 4y)(5~ -12~+8
(a + b)(a + b) = a 2
+2ab +b 2 - 1 h t 8

- 2ab + b2
2
(a - b)(a - b) = a
(a + b)(a - b) = a 2 - b2 +
Example. Divide 2x2 5x - 7 by x 4
(a 2
+ b )(a
2 2 2
- b ) =a 4
- b4
x + 4)22 + 5x - 7(2x - 3
2 2 + 8x

Example. Multiply 4x + 3y - 22 by x - 2y -3x- 7


- 3x- 12
4x + 3y - 22
x -2y + 5 remainder

42 + 3xy- 2xz (i) Proceed as in the previous example by setting down as long
division (see above).
-~ X Y -63 + 4yz (ii) The first term of the divisor, x, into the first term of the dividend,
2x2, goes 2x times, write this as the first term of the answer.
4 2 - 5 ~-
y 2xz - 6 3 + 4yz Ans. +
(iii) Multiply the divisor by 2x, which gives 2x2 8x and set this
down under the dividend.
(iv) Subtract to get -3x.
(v) Bring down the next term of the dividend, that is -7, to give
Example. Divide 6x3 - $ - 14x+ 8 by 3x - 2
-3x - 7.
Procedure: (vi) Repeat procedure as from (ii), i.e., x into -3x goes -3 times,
(i) set down in long division style; put - 3 into the answer. Multiply the divisor by -3 to get
(ii) see how many times the first term of the divisor goes into the -3x - 12 and set this down. Subtract, and this leaves 5. +
first term of the dividend and write this down as the first term of There is now nothing more to bring down from the dividend and
the answer: thus, 3x into 6x3 goes 2 2 times; x will not go into + 5, hence this is left over and written as a
(iii) multiply the divisor by 2x2and set down under the dividend like remainder:
terms under each other; . (2x2 + 5x - 7) + (x + 4) = (2x - 3) remainder + 5 Ans.
(iv) subtract, this gives 3x2;
(v) bring down the next term of the dividend (i.e., - 1 4 ~ ) ; or 2 2 + 5x - 7 = (x + 4)(2x - 3) + 5
ALGEBRA 31
EFFECT OF ZERO IN MULTIPLICATION AND DIVISION t
REMOVAL OF BItACKETS
When any quantity is multiplied by zero, the result is zero.
A immediately in front (or behind) a bracket indicates that
every term inside the bracket must be multiplied by that quantity on
Examples. 3xO=O removing the brackets. Care must be taken with regard to the signs, if
axbxcx0=0 the multiplier is positive, the signs of the terms in the brackets will
remain the same, if the multiplier is negative all the signs of the terms
(x+y) x o = o
will be changed. If the bracket is preceded by a sign only, e.g., + or - ,
it indicates that the multiplier is unity.
When zero is divided by any quantity, the result is zero.
Examples. 2 ( 2 +3x - 5 )
Examples. Ot4=0 = 2 3 ? + 6 x - 10 Ans.

When any quantity is divided by zero, the result is infinity.


Examples. 5tO=m

--+
2r 3y
- 00
0
When an expression works out to be zero divided by zero, the result is BRACKETS CONTAINED WITHIN BRACKETS
indefinite, that is, it can have any value and indicates that the problem
requires to be solved by a different method. The common sets of brackets are,

( ) small, used as inner brackets,


Examples. Solve a2 - b2 when a = 3 ,
- and b = 3 I ) intermediate brackets,
a- b [ ] square, used as outer brackets.
32 - 32
One method, ---
3- 3 When simplifying expressions composed of bracketed quantities within
other brackets, it is usual to remove the inner brackets first, then the
intermediate brackets, and finally the outer brackets.
- b2
a 2-
Other method,- -
a- b Example. y(1 + 3x) + [3x(x- y) - (-2(x - 1 ) + 3 2 ) - y ]
+
- (a b)(a - b)
(a - b) cancels
=y+3v+[32-3xy-{-%+2+32)-y]
a- b =y+3xy+[32-3xy+2x-2-32-y]

=3+3 =32-32+3xy-3xy+2x+y-y-2
= 6 Ans. = 2 x - 2 Ans.
32 R E E D ' S MATHEMATICS FOR E N G I N E E R S ALGEBRA 33
FACTORISATION hemo terms. Therefore any expession which consists of the difference
Factorising is the reverse of multiplying, that is, it is the process of between ~ W Osquares can be readily factorised.
finding the numbers or quantities which, when multiplied together, a 2 - b2 = (a + b)(a - b)
Factors of
will constitute the expression given to be factorised. Generally,
expressions to be factorised fall into one or more of four types: (i) Factors of # - 16=0,+4)O,-4)
those made up of terms with common multipliers, (ii) difference Factors of 1 6 d - 25d2 = (40 + 5d)(4D - 5d)
between two squares, (iii) perfect squares, and (iv) those whose factors
have to be found by trial and error. (iii) Factorising expressions which are perfect squares. Firstly, check
(i) Factorising expressions which contain common multipliers. If the following and note the form of the result in each case.
every term contains a common factor, divide throughout by this factor
and express the factor as a multiplier to the resultant terms enclosed in a
bracket, thus,

The above are simple cases where the coefficients of x2 is unity, such
,,pressions can be easily recognised as perfect squares because, in each
case, the third term is equal to the square of half the coefficient of x.
This is the reverse process to removal of brackets. Note that,

Examples. 3x+ 2xy - xz +9 is the square of half of + 6


$25 is the square of half of - 10
= x(3 + 2y - z) Ans. +169 is the square of half of - 8y

Examples. J@TiZT)=a+2 Ans.

Now check and examine the following slightly more difficult cases.
(ii) Factorising expressions which are the difference between two
squares. Examine first the following cases and note the similarity of the J(9x2 + 24x + 16) = 3x + 4 h s .
factors and the results of the multiplications: because, (3x + 4)(3x + 4) = 9 2 + 24x + 16
J(16x2-48~+36) = 4 x - 6 Ans.
because, (4x - 6)(4x - 6) = 1 6 2 - 48x + 36
J(25x2 + 70xy + 493) = 5x + 7y Ans.
because, (5x + 7y)(5x + 7y) = 2 5 3 + 70xy + 4 9 9

Expressions which are perfect squares can therefore be recognised at


once and their square root becomes obvious. Note particularly the effect
Hence if the sum of two terms is multiplied by the d z f e ~ n c ebetween of the minus sign.
those two terms, the result is the difference between the squares of (iv) Factorising by tial and error.
34 REED'S MATHEMATICS FOR ENGINEERS
ALGEBRA 35
Check the following and the form of the expressions: X
N~~ wmbining the pairs of factors of 6x 2 with the pairs of factors
of -20 until it is seen that the combination which will produce the
c o m t middle term is ( l r + 5) and (3x - 4 ) hence, the factors of
6 2 + 7x - 20 = (2x + 5)(3x - 4) Ans.

Note that the first term of the expression is the product of the first After a little practice, with an eye on the middle term, obvious misfits
terms of the factors, the last term of the expression is the product of can quickly be eliminated and the correct factors spotted
the last terms of the factors, the middle term of the expression is the
sum of the product of the mean terms and the product of fhe extremes. much trouble.

Example. Factorise x2+ 5x + 6

The factors of 2 are x and x


LOWEST COMMON MULTIPLE
The factors of + 6 can be + 6 and + 1
or - 6and - 1 As in arithmetic, the L.C.M. of a group of algebraic terms is the
lowest quantity into which each of the terms will divide.
+
or 2 and 3 + For example, the L.C.M. of 4x2y, 63y2, 2.xy.z and x2i? is 12x2@
or - 2 and - 3

Although any of these pairs multiplied together will give the third term,
+
i.e., 6 , only one of the pairs added together will give the coefficient of
+ + +
the middle term, i.e., 2 added to 3 is 5 , hence the factors of FRACTIONS
To add and subtract algebraic fractions, bring them all to a common
-?+5~+6=(~+2)(~+3) Ans. denominator by finding the L.C.M. of the given denominators of the
fractions, then add and subtract, like terms being combined together.
The procedure is demonstrated in the following example.
Example. Factorise 6x 2 + 7x - 20

The factors of 6 2 can be 6 x and x 3 x + l --


- x f 2 - h-
- 3
Simplify
2x 3x 6x
or lr and 3x
+
The factors of -20 can be 20 and - 1
The L.C.M. of the denominators is 6x.
or -20and + I
+
or 10 and - 2 First fraction: 6 x + 2x = 3 , thus the denominator of 2x is to be
multiplied by 3 to make it 6x, therefore the numerator must also be
or -10and + 2
multiplied by 3 .
or + 5and - 4
Second fraction: 6x + 3x = 2 , the denominator of 3x is to be
or - 5 and 4 + multiplied by 2, therefore the numerator must also be multiplied by 2 .
36 R E E D ' S M A T HE M A T I C S F O R E N G I N E E R S

Third fraction: the denominator is already 6x therefore the numerator


- whenx=2, and y = 3
ALGEBRA 37

also remains unchanged.

3(3x+ 1 ) - 2 ( x + 2 ) - ( 2 x - 3 )
usually written
6x = 10 Ans.

--5x f 2
- Ans. 2 2 2
6x Example. Evaluate 3a - 3b - 4c
Note that the given expression contained no brackets, the reason when a=4, b=- 2, and c = 3
being that division lines of fractions act as a double purpose, to indicate
that the numerator is divided by the denominator but also to indicate that 3a2 - 3b2 - 4c2
everything above (or below) the line is one complete term, thus
= 3 ~ 4 ~ - 3 ~ ( - 2 ) ~ - 4 x 3 ~
3x + 1 (3x f 1 )
could be written - =3x 16-3x4-4x9
2x 2x
However, when proceeding with the simplification of the expression, = 48 - 12 - 36
it is necessary to insert the brackets before multiplying out to ensure that =o Ans.
all the quantities constituting the term are so affected.

(5x + 2)
- +
5x 2
The final answer could be written or -
6x 6x Example. Evaluate (a + b)2 - (c + d) +x3 - y
when a = 3 , b=4, c = 5 , d=6, x = - 2 , ~ ~ = - 3

EVALUATION
Evaluation is the process of substituting the numerical values of the
(a + b)2 - (C+ d ) + x3 - Y
algebraic symbols and working out the value of the whole expression. - (3 + 4)2 - (5 + 6 ) + (-2)3 - (-3)
-
The substitution of the algebraic symbols by their numerical values
may be done in the original expression to be evaluated, or, the = (712 - ( 11) + (-8) - (-3)
expression may be simplified first and numerical values substituted later, = 4 9 - 11 - 8 + 3
whichever is the more convenient.
The usual rules of arithmetic apply, i.e., quantities enclosed in = 33 Ans.
brackets should be solved first, and multiplication and division must be
performed before addition and subtraction.
3x+2y x - z 2y+z
Example. Evaluate -+---
z Y X
38 R E E D ' S MATHEMATICS FOR E N G I N E E R S ALGEBRA
-
2
Example. Find the remainder when 2x3 - 3x - 3x + 5 is divided
byx-2.
3x+2y +---
x - z 2y+z
Z Y X Letx-a=x-2
then a = +2
f (x) = 2x3 - 3 2 - 3x +5
R =f (a) = 2(213 - 3(2)2 - 3(2) + 5
=16- 12- 6+5
= 3 Ans.

2
Example. Find the value of y in the expression x - 5x + y if
(X- 7) is a factor.
If (x - a) is a factor off (x), the remainder is zero, and f(a) = 0

Letx-a=x-7
REMAINDER AND FACTOR THEOREMS
then a = +7
The remainder theorem states that if a polynomial in x is divided x - 7 is a factor, :. f(+7) = 0
by x - a, the remainder is equal to the result obtained when a is
substituted for x in the polynomial. f(x) =,? - 5 x f y
A polynomial is an expression consisting of a number of terms, f (+7) = (7)2 - 5(7) +y
each term being a multiple of an integral power of a quantity such 0=49-35+y
as x. The degree of the polynomial signifies the highest power.
+
For example: 3x2 - 4x 2 is a polynomial of the second degree y=-14 A~s.
+ +
in x; 4y3 2y2 3y - 5 is a polynomial of the third degree in y, and
SO on.
A function of a quantity is an expression which depends upon that Example. Find the values of b and c if x3 + bx2+ cx - 6 is divisible
+
quantity. For example: 2x3 3x2 - 2x - 5 is a function of x and +
by (x l)(x - 2) with no remainder.
+
denoted by Ax); 2a3 3a2 - 2a - 5 is a function of a and denoted
+
by f(a); 2(213 3(212 - 2(2) - 5 is a function of 2 and denoted
Letx-a=x+l
by f (2).
Hence, using the above notations, and R for the remainder, the then a = -1
remainder theorem may be written: x + 1 isafactor, :. f ( - l ) = O
f (x) = x3+ b,? + cx - 6
R =f (a) when f (x) + (x - a) f (- 1) = (- 1)3+ b(- 1)2+ c(- I) - 6
0=-l+b-C-6
Note that iff (a) = 0, the remainder is zero, which means that x - a is
then a factor of f(x). This is the factor theorem. c=b-7 ... ... ... . . . (i)
40 41
R E E D ' S MATHEMATICS F O R E N G I N E E R S

Letx-a=x-2
-
using Remainder theorem:
ALGEBRA

then a = +2
x - 2 is a factor, :. f(+2) = 0 ~etx-a=~+2
f (x) = x3 + bx2 + cx - 6 then a = -2

f (+2) = (2)3 + b(2)2+ c(2) - 6 f(x> = x3 2 2+ + bx + c


0=8+4b+2~-6 R =f( = (-2)3 + 2(-2)2 b(-2) + +c
2~=6-8-4b 11 = - 8 + 8 - 2 b + c
c = - 1 - 2b ... ,.. . . , (ii) c=2b+11 ... ... .. (ii)

From (i) and (ii),


From (i) and (ii):

Substituting value of b into (i),

c=3~2+9=15
From (i):
Constants are 2 and 15 Ans.

Values of b and c are, 2 and -5 Ans. BINOMIAL THEOREM

Example. Find the constants b and c if, when x3 + + +


bx c is
Is used in approximation and error calculatjons. It can be shown, by
+ +
divided by (x 3) there is no remainder, and when divided by (x 2) the induction, that:
remainder is 11.
Using Factor theorem: (a+x)"=an+na
"-1
x
1
+---n(n2 x-11) a n - 2 ~ 2
n(n - l)(n - 2) C 3 x 3+ etc.
+
Letx-a=x+3 3 x 2 ~ 1
then a = -3
x + 3 is a factor, hencef (-3) = 0 which is more readily applied as:
+@ + b x + c
f(x) =x3
f (-3) = (-313 + 2(-3)* + b(-3) + c (1 +x)" = 1 + nx+ n(n-1)
2x1 *+ n(n-l)(n-2) x3
3 x 2 ~ 1
+ etc.
O=-27+18-3b+c
c=3b+9 ... ... ... and is true for all values of n provided -1 c x < 1 i.e., x is numerically
(1) less than one.
6. (i) Divide (8a2 - 8ab - 66') by (2a - 3b)
(5) hide (92 - 9 2 y - lo$ + 8 9 ) (3x - 4 ~ )

=1('(' - f x 0.002 - $ x 0.000004) (iii) Divide ( 2 - 9 ) by (x - y)


= 9.99333 ., Simpli~the following by removing brackets and collecting terms:
(i) (a + b) + (C- d ) - (a - b) - (c + d) + (a - b)
+ +
(ii) 2{a - 3(a 2) 4(2a - 1) + 5)
(iii) 2 - [2x - (2x - (2x - 2) - 2) - 21 - 2
TEST EXAMPLES 2
+
(') Add together 3x 4y - 5, and -h - 5y 4z +
8. Factorise the following expressions:
(") Add 2a 2b - + 3ab2 and .jab' - a2b
+
+

(jii) Sub'act h 5y - 3r from Sx - 4z 3y +


+
(ii) pv PVX
(iv) Subhct -4, - 56 - a hm 3a + 6c - 2b +
(iii) & - 2 b 2 ~ C X
2. Simplify the following expressions by collecting terms, (iv) 12a3b3c3- 8a2b2c2 + 4abc
(i) 5x - 32 - 4~ - 2y + 4y + 2 9. Factorise the following expressions:
+ + 2-Y
(ii) 2.5a c - 1.20 2.56 - 3c + b + 1.7a (i) d - d 2
+
(iii) b2 - 3ab2 2a2b - 4a2 - 2b2 + 5a2 - 2ab' (ii) 1 - a 2
(iii) 4 x V - 9 2
3. Simplify the following,
xxX3xx5 (iv) T, - T2
x2 9 10. Factorise the following expressions:
(ii) x5 x x - ~x x - ~
(iii) & xixi x xi
+ +
(i) a2 8a 16
+
(iv)
(v)
2m
(x2)3 x 3 aX x-5
(ii) d 2 - 10d 25
(iii) 9$ + + 12v 4
+
(iv) 4x2 - 12xy 9 3

11. Simplify the following:


x x 2 x
(i) -+---
2 4 3

5. Work out the following:


+
(i) (a b)2
(iii) -
4 18 -- 3
x+2+x2-2x-8 x- 4
R E E D ' S MATHEMATICS FOR E N G I N E E R S

Find the value of


2~~- x~~ + - 3y3
whenx=2, andy=-2 CHAPTER 3

Simplify the following and find the value if x = -2, and y = -3.
SIMPLE EQUATIONS
3[4x + 2{x - 2y - (3x + y)] - 3x1
Find the remainder when 2r3 - 4x2 + 5x - 6 is divided by (x - 3). An equation is an expression consisting of two 'sides', one side being
equal in value to the other. A simple equation contains one hidden
Find the value of b if, when 2 + 6x2 + bx + 9 is divided by (x + 5) number of the fist order (e.g., x, and not x2 or x3, etc.) which is usually
the remainder is 4. referred to as the unknown and to solve the equation means to find
h e value of the unknown.
Find the value of c if (x+ 4) is a factor of x3 + 1 1Ox + c.
- Since one side of the equation is equal to the other side, it follows
that if, in the process of solving, it is convenient to make some change to
Find the constants b and c in the expression 2 + 3x2 - bx - c if, one side, exactly the same change must be made to the other side to
when it is divided by (x + 1) there is no remainder, and when ensure that the equation is still true. Hence, equality will be preserved if:
divided by (x + 2) the remainder is 15. (i) the same quantity is added to both sides;
(ii) the same quantity is subtracted from both sides;
By using the binomial theorem, find the cube root of 1.012 correct (iii) every term on both sides is multiplied by the same quantity;
to 7 places of decimals. (iv) every term on both sides is divided by the same quantity;
(v) both sides are raised to the same power;
(vi) the same root is taken of both sides;
and any one or more of the above steps may be necessary in solving an
equation.
There can be any number of terms in an equation, some may be
fractions, others may be bracketed quantities: the unknown quantity may
be included in more than one term and perhaps on both sides of the
equation. In such cases it will be necessary to simplify the equation,
preferably one step at a time, following the usual rules of arithmetic and
algebra thus:
(i) eliminate fractions; this is done by multiplying every term on
both sides by the L.C.M. of all the denominators;
(ii) remove brackets; in doing so observe strictly the rules on
removal of brackets explained in Chapter on Algebra;
(iii) place all terms containing the unknown on one side, and pure
numbers on the other side; terms taken over from one side to
the other must have their signs changed;
(iv) collect and summarise terms on each side;
(v) find the value of the unknown; for the simplest of equations this
is done by dividing both sides by the coefficient of the unknown,
46 R E E D ' S MATHEMATICS FOR E N G I N E E R S S I M P L E EQUATIONS 47
other equations may require further or different treatment such cancel out denominators into the 6 on the top, and simplify each term,
as taking the same root of both sides or raising both sides to
the same power. 4~+9x=96-3x
The above is a general purpose guide to the usual steps in solving 4x+9x+3x=96
simple equations. However, other processes can be introduced when 16x = 96
most convenient, for instance, in some equations which include
x = 6 Ans.
algebraic fractions, it may be possible to cancel common factors in the
numerator and denominator of one or more of the fractions, if this is
done in the initial stages labour will be saved later.
Example.
The following examples will help to clarify the above steps.
Example. 4x+ 10 = 18
Take 10 to the other side and change its sign, this is the same as
subtracting 10 from each side, L.C.M. of denominators is 12, multiplying each term by 12 and
cancelling denominators in one step,

Divide both sides by 4 (the coefficient of x)


x=2 Ans.

Example. 7(2x - 4) = 2(x + 4) - 5(3 - x) x=4 Ans.


i
Remove brackets, multiplying the terms inside each bracket by the 1
(Note that -52 divided by -13 is +4).
multiplier in front. g
9x 6
1 4 ~ - 2 8 = 2 x + 8 - 15+5x Example. 8 - - - - = 2(4 - 3 ~ )
2 x
Place all terms containing x on one side and pure members on the L.C.M.of denominators is 2x, multiply every term by 2x and at the same
other, not forgetting to change signs when terms are taken over, time cancel out denominators,
1 4 ~ - 2 ~ - 5 ~ = 815+28
-
16x - 9 2 - 12 = 4x(4 - 3x)
Summarise each side,
16x - 9 2 - 12 = 16x - 1 p
7x = 21
Collect functions of x2 and x on one side, numbers on the other side,
Divide both sides by 7,
x=3 Ans. - 92 + 1 2 2 + 16x - 16x = 12
2x x
Simplify, note that + 16x and - 16x cancel out,
Example. -+1,5x=16--
3 2 3x2 = 12
L.C.M. of denominators is 6, multiply every term by 6,
Divide both sides by 3,

2 = 4
SIMPLE EQUATIONS 49
48 R E E D ' S MATHEMATICS FOR E N G I N E E R S -
Take square root of both sides, Give the unknown quantity a symbol, for example, 'let r = internal
in mm',or 'let m =mass in kg', and so on.
x=f2 Ans.
+
Note. The square root of 4 can be 2 or -2, because -2 Example. A port (left) tank and a starboard (right) tank,each of
+
multiplied by - 2 gives 4. Therefore when taking the square root of a 200 t capacity, are each half full of oil. Find what mass of oil must be
number in the solution of equations write f, indicating 'plus or minus'. pmped out of the port tank and into the starboard tank so that there will
If either + 2 or -2 be substituted for x in the original equation, it will be be four times the mass of oil in one tank as in the other.
seen that either value of x satisfies the equation.

Example. PORT STARE3CPD


3I 1 I I

1 ;f
CAPACITY O F
E A C H TANK
200 TONNE

On inspection of the numerators in this equation, it appears a


possibility of their respective denominators dividing into them, this is
always worth trying because it will greatly simplify the working. Instead 100 T O N N E
of actual division try factorising the numerators, one of the factors being
the same as the denominator. This example is such that it will factorise,
cancel, and form an easy solution as shown below. If however this was Fig. 2
not possible, the alternative procedure would be to multiply throughout
by the L.C.M. of the denominators as previously explained.

Initial mass of oil in each tank = 100 t


Let m =mass of oil pumped over, then,
final mass of oil in port tank = (100 - m) t
+
final mass of oil in starboard tank = (100 m) t
Mass in the starboard tank is to be four times the mass in the port tank,
therefore,

x = -2 Ans. Mass in starboard tank = 4 x mass in port tank


(100+m) = 4 x (100 - m)
100+m = 400 - 4m
PROBLEMS INVOLVING SIMPLE EQUATIONS 4m + m = 400 - 100
From the facts given in the problem and what is required to be found, 5m = 300
the first step is to make up a straightforward equation; this is the m = 60
sorting-out part of the solution. r
B .'. mass of oil to be transferred = 60 t Ans.
Make a sketch of the problem if it is at all possible and insert on this Ei
(Thn there will be 160 t of oil in the starboard tank and 40 t in the pon
sketch all the given data and indicate the unknown quantity. If a sketch
is not practicable it may help to write down the given quantities. 3 tank.)
is 15 knot, find the speed of the current.

can be found from the total time being equal to the time
up-river plus the time for it to return down-river,
ours can be expressed as distance (naut. miles) divided by
to suit h e quantities given in the problem.
~~t x = speed of current (hot),

speed of against went = (15 - x) h o t ,

Fig. 3 , speed of +
running with current = (1 5 x) knot.
+ Time to go down-river = Total time
distance distance down-river
Let x = distance from A to passing point c =3$ h
then (72 - x) = distance from B to passing point c
+up-fiver + speed down-river

As the trains leave at the same time and pass each other at the
instant, then the time taken from station to passing point is the same wq term by (15 - x)(15 +x), the L.C.M. of the
each train. This gives the equation,
Time for electric train Time for diesel train
to travel x km to travel (72 - x) km
375 + 25x + 375 - 25x = 3-5(15~
-2 )

750 = 787.5 - 3.52


distance travelled by electric train 3.52 = 787.5 - 750
speed of electric train 3.52 = 37.5
2 = 10.71
x = f m
L.C.M. ~ 3 2 0multiplying
, every term by 320,
x = 3~3.273
4x = 5(72 - x) .'. Speed of current = 3.273 knot Ans.
4~ = 360 - 5x
4~+5x=360
9x = 360
x = 40 a group of algebraic symbols which expresses a rule

.'. Trains pass each other at a point 40 km from A. Ans. e others. To transpose means to change the order of
52 R E E D ' S MATHEMATICS FOR E N G I N E E R S S I M P L E EQUATIONS 53
the symbols to produce a re-arranged equation giving the value of one of > Divide both sides by D,
the other symbols in terms of the remainder. Being an equation, the
procedure is the same as for equations in general. C x d3
-=S
D

I Example. Transpose the following formula to give p2 in terms of


the other quantities,
cd3
S =-
D
Ans.

Note the usual practice of omitting multiplication signs between the Example. Transpose the following expression to give the value
quantities to be multiplied, for the first few examples the multiplication of r in terms of E and n. Find the value of r when E = 60, n = 1.4.
signs will be put in while working through the various steps.
P l X V l -P 2 X V 2
TI T2
Note, I"-' = 1
Multiply both sides by the L.C.M. of the denominators which is T, x T2,
pl x Vl x T2=p2 x V2 x T I
Divide both sides by V2 x TI, so that p2 will be left on its own on one
side, Divide both sides by 100,

Take 1 to other side leaving unknown by itself,


It is usual to write expressions and formulae with the required unknown
on the left side of the equation and to omit multiplication signs, thus,

Multiply throughout by - 1 to make unknown positive,

Example. Express S in terms of the other quantities in the following


expression, Cross-multiply to bring unknown on top,

Cube both sides,


S x D Take n - 1 root of both sides,
d =-
C
Multiply both sides by C,
54 R E E D ' S MATHEMATICS FOR E N G I N E E R S

Insert numerical values,


14- 1

--1 ---1
- 2.5
= 1.453 Ans. (ii)
1.4 - 1 0.4
60
1 - -= 1 - 0.6 = 0.4
100
1 1 L
p-

60 - 0.4 = 2.5 i LOGARITHMIC EQUATIONS


'-loo
TO illustrate the basic principles, two examples are now solved using
Hence,
logarithms (to base 10). Simultaneous logarithmic equations are covered
r = 2.52 5 in Chapter 4.
= 9.88 Ans. (ii) x x
Example. Given ab = cd , find x when: a = 13.5, b = 1.8,
c = 68, d = 0.55.
Transpose the following expression to give V, in terns of
the other quantities,
p1 x v l n = p 2 X V2"
and find the value of VI when p , = 1.035, p, =4.14, V, = 0.5 and
n = 1.3.

PI X V l n= p 2 X V2" Bring all terms to the power of x to one side, and pure numbers to the
Divide both sides by p , other side,

P2 X
vln=% V2
PI
Take n'th root of both sides,
3.273' = 5.037
(Ig of 3.273) x x = lg of 5.037

or, vl = v2X k)' Ans. (1)


x=-
0.5149
x = 1.363 Ans.
56 R E E D ' S MATHEMATICS FOR E N G I N E E R S S I M P L E EQUATIONS 57

Example. Given T~ =
- ) 1
;!

find n when, 5. ~ j a~in the


d following:
T2 (1 - 2a 2 ) - -1+ -a= - -1 A 5
6a 4 3 6 2 a

2 3 - 4
-
1-3
+-
x+2
-
x2-X-6
find x.
n-1
Ig of 2.648 = (Ig of 32.76) x -
n
7. A ship travelling at 17.5 knot leaves one port bound for another
1
4 h after another ship whose speed is 16 knot leaves the same port
set on the same course. After how many hours and at what distance
from port will the fast ship overtake the slower one?
-.
n=- g. A rectangular plate is to be cut so that the length is four times the
~

1.0925 breadth and having an area of 1 m2. Find the length and breadth.
n = 1.387 Ans.

Note. Logarithms to any base can be used to evaluate these types 9. Transpose the following equation to give e2 in terms of the other
of quantities:
example involving logarithmic (and exponential) equations and
functions.

TEST EXAMPLES 3 and find the value of O2 when:

1. Find the value of x which satisfies the following equation:

8+5x-7=3x+9

2. Find the value of a in the equation:


10. The diameter in rnm of coupling bolts should not be less than that
2(a + 3 ) + 3(2a - 4 ) = 4(11 - 3a) given by the formula:

3. Solve this equation:

3[3 - {x + 2(1 - x)} - 4x1 = 2[x - 3(2 + x ) - 41 where D = diameter of shafting in rnm;
n = number of bolts per coupling;
4. Find x fiom the following equation: R = pitch circle radius in mm.
Express R in terms of the other quantities and find the pitch circle
radius when d = 82.5 rnrn, D = 381 mm, and n = 8 bolts per
coupling.
58 R E E D ' S MATHEMATICS FOR E N G I N E E R S

11. Given

CHAPTER 4
find the value of n when TI = 670, T2 = 324, p, = 2 1 andpi = 1-25. SIMULTANEOUS LINEAR
12. Find the values of x in each of the following equations: EQUATIONS
(a) 4°.s9x= 56.36
(b) 12.4~0.53 ~nthe preceding chapter, equations containing only one unknown
Sx
(c) t = 46.382.6 quantity were explained; now two (or more) unknown quantities are
=quired to be found.
(d) 3J;I= €2.5
To find the values of two unknowns, two equations are required, each
containing the same two unknown quantities, this pair of equations is
called 'simultaneous'.
various different methods may be used in the solution of these;
three methods commonly employed will be explained here in the order,
(i) by elimination, (ii) by substitution, (iii) by equating expressions of
like unknowns. There is another method and that is by drawing a
graph of each equation on a common base; the point of intersection
of the graphs produces the solution of the unknowns; this method is
included in Chapter 6 .
Method (i). This process is to couple the two equations together in
such a form that by either (a) adding them together, (b) subtracting one
from the other, ( c ) multiplying them, or (6)dividing one by the other,
one of the unknown quantities cancel out, leaving a single equation
containing only one unknown which is then simply solved. When one
of the quantities has thus been found its value is substituted into one
of the original equations which will then become another single equation
containing only the other one remaining unknown to be solved.
The following examples will serve to demonstrate some of the steps
normally involved in this method of solution.

Example. When twice one number is added to five times another


number, the result is 34, and when three times the latter number is
subtracted from four times the former, the result is 16. Find the numbers.

Let x = one number,


and y = other number,
From the first statement, 2x + 5y = 34 . . . . . .
... .. . (9
From the second statement, 4x - 3y = 16 . . . . . . . . . . . . (ii)
60 R E E D ' S MATHEMATICS F O R E N G I N E E R S
.
Multiply equation (i) by 2 SO that it will contain h e same multiple
S I ~ ~ ~

= 3.5 into equation (ii)


~ LINEAR
~ ~ E EQUATIONS
O U S
61

I of x as in equation (ii), and set the pair down again,

I
-1

> I
subhering (iv) fmm (iii), 4~ will cancel, leaving one equation
only Y as the unknown. Remember that when submcting a
I p = 4.5

1 1 minus quanfi% chmge its sign and add, thus subtracting -3y kom The values of and q are 4.5 and 3.5 respectively.
I
+ 10y gives +
13y !
4 ~ + 1 0 ~ = 6 8 . . . . . . . . . (iii) Example. Find the values of x and y in the simultaneous equations,
subtract 4x - 3y = 16 - +Y- = -7
X
. . . . . . . . . . . . (iv) 8 5 8
13y = 52
2x - Y- - 3
-
y=4 3 2-4
the value of^ found to be 4, into any one of he original simplify these equations by multiplying the first by the L.C.M. its
equations which contain x and y; this will produce mother equation denominators which is 40, and the second by its L.C.M. which is 12,
containing only x as the unknown.
Substituting Y = 4 into equation (i) 5x + 8y = 35 . . . . . . . . . . . . 6)
8x - 6y = 9 . . . . . . . . . . . . (ii)
2x+5x4=34
2x = 34 - 20 Multiply (i) by 3, and (ii) by 4,
2 x = 14 15x + 24y = 105 . . . . . . . . . . . . (iii)
x=7 32x - 24y = 36 . . . . . . . . . . . . (iv)
merefore the numbers are 7 and 4. Ans. Add (iii) and (iv) 47x = 141
x=3
Example. +
Given 2p 59 = 26.5
and 3p - 29 = 6.5 Substitute x = 3 into equation (i),
find the values of p and q.

2~+5q=26.5 . . . . . . . . . . . . (i)
3~-2q=6.5 ...... . . . . . . (ii)
Multiply (i) by 3, and multiply (ii) by 2,
The values of x and y are 3 and 2; respectively. Ans.
Example. Given the simultaneous equations,
6 p - 4 q = 13 . . . .
1.5" x 2' = 18
Subtract (iv) h r n (iii) 19q = 66.5
4x X 1.Y= 54
q = 3.5
find the values of x and y
62 R E E D ' S MATHEMATICS F O R E N G I N E E R S S I M U L T A N E O U S L I N E A R EQUATIONS 63
/

Writing the two simultaneous equations in log form (base 10); see Tonsgos~gthe hest equation to express x in terms of the other
logarithmic equations (Chapter 3). quantities.

(lg 1.5) x x+(lg 2) x y = lg 18


+
(lg 4) x x (lg 1.5) x y = lg 54

Inserting log values,

+
0 . 1 7 6 1 ~ 0 . 3 0 1 ~= 1.2553 . . . . . . . . . . . . 0) substituting this value of x into the second equation, simplifying and
+
0 . 6 0 2 1 ~ 0.1761y = 1.7324 . . . . . . . . . . . . (ii) solving.

Dividing (i) by 0.1761, and dividing (ii) by 0.6021,

x+1.709y=7.127 . . . . . . . . . . . . (iii)
x + 0.2925~= 2.877 . . . . . . . . . . . . (iv)

Subtracting (iv) from (iii),

Substituting y = 4 into the first equation and solving,

Substituting y = 3 into (iii),

The values of x and y are 2 and 3 respectively. Ans. The values of x and y are 7 and 4 respectively. Ans.
Method (ii) of solving simultaneous equations is by transposing one
of the equations to express one of the unknowns in terms of the other
quantities, then substituting this expression for that unknown into the Method (iii) is to transpose both equations to express the same
other equation, thus producing a single equation containing only the unknown in terms of the other quantities, the two expressions are then
other unknown which can then be solved. equated together, making one equation with one unknown.
Repeating the first example to be solved by this method, Taking the same example again,

...... (9
. . . . . . (ii)
S I ~ ~ ~ ~ ~ NLI N
E EO
A RUEQUATIONS
S 65

Transposing both equations, in each case expressing x in terms of the


other quantities, ~~d equations (i) and (iih
3x+2y-z=4
From (i) 2x = 34 - 5y 2x+y+z=7
x = 17 - 2 . 5 ~ . . . . . . (iii) 5x+3Y =11 . . . . . . . . . . . . (1~)
From (ii) 4 x = 16+3y
submet (iii) from (ii),
x =4 + 0.75~ . . . . . . (iv) 2x+y+z=7
Since x in one equation has the same value as in the other, then, x-y+z=2
(iii) = (iv) xf2y =5 . . . . . . . . . .

Multiply (v) by 5 and subwct (iv) from the result,


+
5x 1Oy = 25

Y Substituting y = 4 into equation (i)


I I y =2

Substitute y = 2 into (v),


x+2x2=5
x=5-4
I
1 x = 7 and y = 4 as before. x=l
I
I
I The method to employ in solving a given simultaneous equation will substitute x = 1 and y = 2 into (iii),
be that which is easiest to apply depending upon the form in which
I
1
I
I the equation is given.
1-2+2=2
I 2=2+2-1
1
1 z=3
I I THREE UNKNOWNS
I I Just as two unknowns can be found from two equations, three .'. x = 1, y = 2 and z=3. Ans.
l1 1
unknowns can be solved if three equations be given.

Example. To fkd the values of x, y and z which satisfj, the TEST EXAMPLES 4
equations,
i i
' I
1I
Ill
1 1
I
3x+2y-z=4 ............
(i)
1. When 2 times one number is added to 3 times another the result
1
is 19; and when 3 times the first number is subtracted from 2
times the second, the result is 3. Find the numbers.
i
1 1 1 ; I ~x+Y+z= 7 . . . . . . . . . . . . (11)
1 I 1 x-y+z=2 . . . . . . . . . . . . (iii) 2. Find the values of x and y in the simultaneous equations:
1 1 11 I Eliminate one unknown at a time, take z first as it appears the easiest
2- -3 ~ - 2 and
- - - -Y - - 13
to get rid of 3 5-4 2 4- 16
67
66 S I ~ ~ ~ ~ ~ I NNE A~R EQUATIONS
L O U S
R E E D ' S MATHEMATICS FOR E N G I N E E R S

3. Find the values of a and b which satisfy the equations: I I . Given me following relationship,
a(1 + 2b) = 3 and a(1 - 3b) = 0.5 d-2= - x2
=- 9
x2 3 D2
4. A man and his wife are 72 and 68 years old respectively, and have
one grandson and one grand-daughter. The man's age is equal to express x in terms of d and D, and find the values of x and y when
the sum of four times the grandson's age and three times the D=75 and d=25.
grand-daughter's. The woman's age is equal to the sum of three
times the grandson's age and four times the grand-daughter's. Find 12. Find me values of a, b and c in the simu~taneousequations,
the ages of the two grandchildren. 3a + 6b - 2c = 7.25
I 2a+3b+4c=26
5. The difference between two numbers is 2 and the difference 4a - 2b + c = 10.25
between their squares is 6. Find the numbers.

6. The linear law of a simple lifting machine is given by F=a bm+


where m =mass lifted, F= effort applied, a and b being constants.
In a certain lifting machine it was found that when m =30 kg,
F= 35 N, and when m = 70 kg, F= 55 N. Find the constants a and
b, express the law of this machine, and find the effort required
to lift a mass of 60 kg.

7. Two ships, A and B, leave one port bound for another on the same
course. B leaves 1 h later than A and overtakes in 8 h. If the
speeds of each ship had been 4 knot slower, B would have
overtaken A 2 h earlier. Find the original speeds of the ships.

8. Given the simultaneous equations,

find the values of


x 1
-, , -
x and y.
2- y x

9. Find the values of x and y in the simultaneous equations,


2" = 47 and 4"-' = 2Y+'

10. Find the values of x and y in the simultaneous equations,


1.259+' x 1 . 1 7 ~ - '= 2.323
3.162" x 1.77gy = 25.12
CHAPTER 5

QUADRATIC A N D CUBIC
EQUATIONS

A quadratic equation is one which contains the square of the


d o w n quantity, thus, x2= 36 is a quadratic equation in its very
elementary form and solved simply by taking the square root of both
sides, X = f 6.
The form usually associated with the title quadratic equation contains
the first power of the unknown as well as its square, the general form
being,
d+bx+c=0
where a (the coefficient of x2), b (which is the coefficient of x), and c
are given quantities, and x is the unknown. These may be solved by
various methods, those which will be explained here are (i) by
factorisation, (ii) by completing the square, (iii) by formula. Another
method is by graphical means and this is explained in Chapter 6.

Method (i) by factorisation. This method is applied only when


the expression can be readily factorised.

Example.
2-5x+6=O . . . . . . . . . . . . (1)
Factorise, as explained in Chapter 2,
(x - 2)(x - 3) = 0 . . . . . . . . . . . . (ii)
From previous, when any quantity is multiplied by zero, the result is
zero. It therefore follows that if the result of the product of two quantities
is zero, one of those quantities must be zero. In equation (ii) the
product of (x - 2) and (x - 3) is zero, hence either (x - 2) or (x - 3)
must be a zero quantity.
If x- 2=0
then x=2
or, if X-3=0
then x=3
70 REED'S MATHEMATICS FOR ENGINEERS QUADRATIC A N D CUBIC EQUATIONS 71

The value of x in the equation can therefore be 2 or 3. By trying each of half the coefficient of x. The coefficient of x here is 3, half
sf is E and this squared is ())2. Therefore add ())2 to both sides,
of these values in the original equation (i) it will be seen that either
will satisfy the equation.
.'. x = 2 or 3 Ans.

Example. 2$ - 5x - 12 = 0
Factorise: (2x + 3)(x - 4) = 0 NOW take the square root of both sides, and solve
either 2x + 3 = 0 or X-4=0
then 2x = -3 then x=4
x = -112

Therefore, x = 4 or -1 Ans.

Method (ii) by completing the square. Any quadratic expression,


whether it can be factorised or not, can be solved by this method.
It is based upon the process of changing the expression into one which
is a 'perfect square', that is, one whose square root can be readily
obtained, then, by taking the square root, the expression so found
contains only the first power of the unknown, thus producing a
simple equation which can be easily solved.
Note the f sign. As previously seen, the square root of a number can be
plus or minus, and both values must be taken into account. The root of a
Example. 4$+12x-7=0
minus quantity is inadrmssible.
Divide throughout by 4 to reduce coefficient of x2 to unity,
Example. 1 2 2 f x - ; = O

Dividing throughout by 12,


The square root of this expression cannot be taken therefore move -:
over to the other side,

and add such a quantity that will make the left hand side into a perfect S h i h g third term to other side and completing the square,
square, not forgetting that whatever is added to one side the same
quantity must also be added to the other side to preserve equality. In
Chapter 2, section on factorisation, paragraph (iii), it was pointed out
that an expression is a perfect square when the third term is equal to the
72 REED'S MATHEMATICS FOR ENGINEERS QUADRATIC AND CUBIC EQUATIONS 73

Taking square root of both sides and simplifying, ~ ~ ksquare


i ~ groot of both sides and simplifying,

x+~=*/q
2a
b2 - 4ac

b fJ-
'+G= 2a

x=--f
b 4-
2a 2a
x = f - - 5- 1 x= -bf JKzG
24 24 2a
4 6
x=- or - - This is a very useful formula which provides a straightforward solution
24 24 to any quadratic equation and should be committed to memory.
1
x=- Of - - Ans.
6 4 Example. 1.52 + 2x - 10 = o
Applying the formula,
2
Method (iii) by formula. Let 'a' be the coefficient of x , 'by the -bf Jb2-
coefficient of x, and 'c' the third term, gives the general form of a x=
quadratic equation: 2a
Substituting,
,&, +2 b~xh+ ~c =~ 0, - = n
a = 1.5,b=2,~=-10,
and solving for x by the method of completing the square, a ready-made
formula is produced for solving any quadratic equation. - 2 f , h 2 - 4 x 1.5 x (-10)
x=
2 x 1.5
&+bx+c=0
x=
-2 f Jm
Dividing throughout by a, 3
-2f J64
1 + - bx
+a - =aco x=
3
-2f 8
Talung the third term to the other side and adding to both sides the x=-
nf
nn,,are nf h a l f t h o ~ n e f f i ~ i e n
3
square of half the coefficient of x, to complete the square,
t v t n o n m n l o t o t h e cnllare

6 -10
x=- or -
3 3
x=2 or -34 Ans.
74 R E E D ' S MATHEMATICS FOR ENGINEERS QUADRATIC A N D C U B I C EQUATIONS 75
2 4
Great care must be taken with the signs. Note that the value of c represent x2, then y = x ,
in the above examples is - 10, and the term -4ac becomes
(- 4) x 1.5 x (- 10) which is 60.+ J-8iY+14&=O
Another point to note is when the value of b is a minus quantity, in
the next example the value of b is - 9, the term -b in the formula then Solving by formula,
becomes - (- 9)which is + 9. E
Example. G-9x-35=0
- b f JFZTc
x=
2a
where a = 2, b = - 9, c = -35

x=7 or -2i Ans.

EQUATIONS REDUCIBLE TO QUADRATICS


+ ' ~ =0
Example. 0 . 3 ~-~1 '. 5~ 6 ~ ~ 1.98
Equations of the form,
Note first that every term is divisible by 3, therefore take advantage
of this by dividing every term by 0.3 to produce an equation with a
simpler set of figures,
where n is a power of x, and 2n is twice that power, can be solved as
a normal quadratic by substituting another symbol of the first order v ~- 5. . 2~p 4 + 6.6 = 0
for x" such as y, consequently x2" would then be represented by y2, thus,
Let x = ~ 2= v ~ ' ~
v ' .then

y is solved by one of the usual methods of solving quadratic equations


then the value of x is obtained from the substitution y =xn.

Example. x4 - 84x2 + 14& = 0 By formula,


I
76 R E E D ' S MATHEMATICS FOR E N G I N E E R S 77
1 Q U A D R A T I C A N D C U B I C EQUATIONS
where a = 1, b = -5.2, c = 6.6 Solving this quadratic by formula,

where a=10, b=3, c=-27

= 2.192 or 1.756 Ans.

Substituting x = 1.5 into the second equation,


SIMULTANEOUS QUADRATIC EQUATIONS
In the previous chapter methods of solving simultaneous linear
equations were explained, but the examples were such that only simple
equations (linear) were involved. Further examples can now be dealt
with where quadratic equations are either given in the fist place or are
produced in the process of solving. Substituting x = - 1.8 into the second equation,

Example. Find the values of x and y which satisfir the equations, -1.8 + 5y = 4
5y = 5.8
y = 1.16
x = 1.5 and y = 0.5 Ans.
Multiplying the first equation by 5 and the second equation by 3, then or x = -1.8 and y = 1.16
adding the results together to eliminate y,
An alternative method to the above is to find the value of one
unknown in terms of the other from the simplest equation, and substitute
this into the other equation. Thus, in the previous example,

2$-3y=3 ... . . . . . . ... (1)


x+5y=4 ... . . . . . . . . . (ii)

From (ii) Y=$(4-x)


78 R E E D ' S MATHEMATICS FOR E N G I N E E R S Q U A D R A T I C A N D C U B I C EQUATIONS 79

Substitute this value of y into equation (i) and simplify, Solving by quadratic formula,
I 1

1 ~ : Multiply throughout by 5,

the same quadratic to be solved as before. From (iii)

Example. Find the values of x and y which satisfy the simultaneous


Ifx=3,
equations:

(i) If x = 2.714, y = 2.5 - 1.5 x 2.714


(ii) = -1.571

I Find the value of y in terms of x from (ii),


Hence, values of x and y are, respectively,
3 and -2
] hs.
or, 2.714 and - 1.571
3x + 2y = 5
y = 2.5 - 1 . 5 ~ . . . . . . (iii)
PROBLEMS INVOLVING QUADRATIC EQUATIONS
I
Substitute for y into equation (i) and simplify, As explained in Chapter 3 on the solution of problems, the first
I

1 I
procedure is to look for and compose an equation from the facts given in
the problem, letting some symbol such as x representing the required
unknown, the equation is then simplified and worked out. If there are
two unknowns express one in terms of the other at the first opportunity.
When simplifying equations which contain terms of the unknown
to other powers than unity, it is usual to arrange all terms on the left
hand side in order of descending powers of the unknown, leaving
zero on the other side. If this produces a quadratic equation it is
solved by the most convenient of the methods just explained. If a
more complicated equation is produced, other means can be employed
such as solving by a graph which will be explained later.
REED'S MATHEM ATICS FOR E N G I N E E R S Q UADRATIC A N D C U B I C EQUATIONS 81

Example. Two ships sail from one port to another, a distance of solving this quadratic by method of completing the square,
825 naut. miles, on the same course, the speed of one ship being
4 knot faster than the other. The fast ship leaves port 2 h after the slow $+4x=206+
ship and arrives at their destination 14 h sooner. Find the speeds of the
2 +4x+(212 = 206;+(2l2

x + 2 = f J G q Z
Let x knot = speed of slow ship
+
then (x 4) knot = speed of fast ship

x+2=f14;
Difference in times for the two ships to cover the journey,
x=f14;-2

=2+14=16h x = 12; or -16;

The minus quantity is inadmissable, the practical value of x is


An equation can be formed from the times taken on the journey:
Time taken by slow ship - Time taken by fast ship = 16 h
;
Speed of slow ship = 12 knot
Speed of fast ship = 12 1+ 4 = 16 ;knot
time = ----

distance travelled by slow ship distance by fast ship


speed of slow ship
CUBIC EQUATIONS
A cubic equation is one which contains the cube of the unknown,
thus, x3= 8 is a cubic equation of the most elementary form and solved
simply by taking the cube root of both sides, x = 3J8= 2.
Multiplying every term by x(x + 4) and simplifying, However, the form usually associated with the title cubic equation
contains either or both of the first and second powers of the unknown as
825(x + 4) - 825x = 16x(x + 4) +
well as its cube, such as x3 2X2 - x = 2.
One method of solution is to bring all terms of the equation to the
825x + 3300 825x = 162 + 64x
- left hand side and leaving zero on the other side, resolve the expression
3300 = 162 + 64x into its three factors, each of these factors are then, in turn, equated
to zero which produces a value of the unknown that will satisfy the
1 6 2 + 64x- 3300 = 0 given equation.

Dividing throughout by 16, Example. x3 + -x-2=0

The three factors of this expression are,


2 +4x-206; =0 (x- l)(x+ l)(x+2)
82 R E E D ' S MATHEMATICS FOR E N G I N E E R S

By equating each factor to zero, the three roots are obtained.


- I Q U A D R A T I C A N D C U B I C EQUATIONS

x=2 satisfies the equation, therefore this is a root, and x - 2 must


83

be a factor. Divide this factor into the cubic expression, this will
produce a quadratic whose roots can be found.

The roots of this equation are + 1, -1, and -2, and any one of these
values of x will satisfy the given equation.

If the equation is not a very easy one like the above, the factors will
not be readily seen. In such cases, attempt to get one root by trial, from
this get the first factor. Dividing the cubic equation by this factor will Equate the resulting quadratic to zero and, if the factors cannot be seen,
produce a quadratic equation which can be solved by factors or by solve by formula,
quadratic formula. The following examples show how this is done.

Example. Find the roots of the equation,

Re-arrange with all terms on the left hand side, in descending powers
of x,

I The roots of the given equation are,


Find the first root by trial, try x = 1,
I 2, 1.5, and -2.5 Ans.

The result could be checked by substituting for x, in the given cubic


equation, each of these roots.

Example. Solve the following equation,


3
2r2=17+-
X

23 - z2 - 5.75 2 +7.5
Multiply throughout by the least common denominator, which is x, to
liminate the fraction,
= 8 - 4 - 11.5+7.5
= 0 this is it. 2 ~ 3= 17x +3
R E E D ' S MATHEMATICS FOR E N G I N E E R S
- f - Q U A D R A T I C A N D C U B I C EQUATIONS 85

Arrange all terms on left hand side and equate to zero, The roots of the given cubic equation are,
2x3-17x-3=0 3, -0.177 and -2.823 Ans.

Find the first root by trial, try x = 1, Note the last two examples. In the former the last term of the cubic
*tion is 7.5 and easy factors of this number which come readily to
2x13-17x1-3 a d are 1, 7.5; 2, 3.75; 3, 2.5; 5, 1.5, all plus or minus, and the
=2- 17- 3 first root of the equation, by trial, was one of these. In the latter example,
= - 18 the last term of the cubic equation is 3 and its ready factors are 1 , 3;
2, 1.5, all plus or minus, and the first root of this equation, by trial,
was one of these. Trial figures for the first root should therefore be
Tryx=2, 2 ~ 2 ~ ~ 1 7 x 2 - 3 chosen with this in mind.
= 16- 34- 3 If the first root is not a number which can readily be found by trial
= - 21 as in the foregoing examples, the above process could be quite
laborious and the equation would probably be more easily solved by a
pphical solution.
Tryx=3, 2~3~-17x3-3
=54-51 - 3
=0 I TEST EXAMPLES 5

1. Find the value of x in each of the following equations,


Hence x = 3 is a root and x - 3 must be a factor. Divide the cubic +
(i) (2x 8)(3x - 5) = 0
equation by this factor to obtain a quadratic which can be solved, (ii) ( 0 . 5 - +
~ 10)(0.25x 5) = 0
x -
3
3)2x - 17x - 3(22 + 6x + 1 +
(iii) (5x 0.5)(4x + 0.8) = 0
3
2x - 6 2
2. Solve the following equations by the method of factorising,
6 2 - 17x- 3
(i) 3 2 + 2 x - 33 = 0
6 2 - 18x
(ii) 4 2 - 17x+4 = 0
x- 3 (iii) 1@ + lox - 12 = 0
x- 3
-
Equate the resulting quadratic to zero and solve by formula, . Solve the following equations by the method of completing the
square,
2~?+6x+1= O (i) 2 - x - 3 ? = 0

x=
-bf dFzz +
(ii) 3 2 2x - 1 = 0
2a +
(iii) 4 2 - 9x 2 = 0
-- 6 fJ 6 2 - 4 x 2 x 1
2x2 Solve the following equations by the quadratic formula,
-
-
-6 f J28 -
-
-6 f5.292 (i) 3 2 - 2x+ 0.25 = 0
4 4 +
(ii) 5 2 4x - 5.52 = 0
= -0.177 or -2.823 (iii) 102 - x - 0.2 = 0
R E E D ' S MATHEMATICS FOR E N G I N E E R S

5. Find the value of x when:


log 0 . 5 =
~ 2 x log(x - 6)
CHAPTER 6
6. Find the value of b in the equation,
6b4 - 2.46b2 + 0.24 = 0 i GRAPHS
I
7. Find the value of V in the equation,
E
v ~- .5.1~v ~+.5.6~= 0 I A graph is a diagram which shows the relation between two
Graphs are usually plotted on squared paper (metric).
8. Find the values of x and y in the following simultaneous equations,
Taking the related quantities to be x and y, Fig. 4 shows the
1 - x y + 2 $ = 16 elements of plotting graphs.
x+2y=8 Both quantities vary in value throughout the graph and the value
of one depends upon the value of the other. In practice there are often
9. In 5 h less time than it takes a certain ship to travel 330 naut. cases where the values of one of the quantities is entirely dependent
miles, another ship which is 3 $ knot faster can travel 4 naut. miles upon how the other is varied. Take for example the stretching of a
further. What are the speeds of the two ships? spring, if a load is hung on the spring hook a certain amount of stretch
2
takes place, if a heavier load is hung a greater stretch occurs, thus the
10. The area of a rectangle is 76 cm and the perimeter is 350 rnm. amount of stretch depends on the magnitude of the load. In this example
Find the length and breadth. the load would be referred to as the independent variable and the stretch
as the dependent variable. In the drawing of a graph of a function of x
11. Find the roots of the following cubic equation, +
such as ,? - 2x 5, let the value of this be denoted by y and write
2x3 +3,? - 17x = 30 +
=x2 - 2x 5, then for a series of chosen values of x the values of y
are calculated to obtain a number of plotting points to enable the graph
12. Find the values of x to satisfy the following equation, to be drawn. Choose the values of x and the values of y depend upon
these, hence x is the independent variable and y the dependent variable.
The larger the scale to which the graph is drawn the more accurate
the results obtained when reading points from the graph, therefore the
scale chosen should be as large as possible depending upon the highest
and lowest values to be represented.
The various values of x and y (or other similar pairs of variable
quantities) are plotted thus, referring to Fig. 4:
The point of intersection of the horizontal and vertical base lines,
marked 0, is the common zero point for both quantities and therefore
represents zero value for both x and y. Positive values of x are measured
horizontally to the right from 0 and negative values are measured
horizontally to the left. The horizontal base line is referred to as the
xx axis. Horizontal measurements are termed abscissae (singular:
abscissa). Positive values of y are measured vertically above 0 and
negative values are measured vertically downwards from 0. The
vertical base line is the yy axis. Vertical measurements are termed
-

89
88 R E E D ' S MATHEMATICS FOR E N G I N E E R S

f+@
5+5
0
s+4,
3
*3..
Y

-
3 2 .
YI

:+I
I
- 4 +> +3 *i +> *'
- +i
-5 3-2 -'I 0
.tNECATIV~VALUES OF X- ~ O S I T I V L L U E S O F X-
VA
I -1..
X
5-2
"4
. I

3-3-
3
w-
2 4.
+
d.

:-5*.
z
'-6-
Y
Fig. 4

PLOTTING OF STRAIGHT-LINE GRAPHS


+
Consider an equation of the form y = a bx where a and b are any
constant quantities and may be positive or negative and as an example
take such an equation where the value of the constant a is 2 and that of b
is 1.5, thus
y=2+1.5~
Working out the values of y for various values of x between the
extreme limits of, say, x = -2 and x = 4, +
When x=-2, y=2-3 = -1
When x=-1, y=2-1.5 =+0.5 Fig. 5
When x =0, y=2+0 = +2
When x= +l, y = 2 + 1.5 = +3.5 Note. (i) that the graph is a straight line, and with this knowledge the
When x=+2, y=2+3 = +5 graph of any equation of the form y = a + bx may be plotted
When x=+3, y=2+4.5 =+6.5 with two pairs of values only to give two points through
When x = +4, y=2+6 = +8 which to draw the straight line.
90 R E E D ' S MATHEMATICS FOR E N G I N E E R S
91

(ii) From left to right the graph slopes upwards and the value
of b determines the gradient of the slope. In this case, for
every unit increase of x the increase of y is 1.5; if b had
been greater than 1.5 the slope would have been greater
and vice- versa. The graph slopes upwards because b is
positive, if b had been negative the graph would have
sloped downwards.
(iii) The value of y when x is zero is 2, and this is the value
of a. It is the intercept on the y axis.
Now consider the equations,
y=4+x
y=3+0.5~
y = 2 - 0.25~
y = -0.5~
y = -2 - 0 . 5 ~
+
All of these are of the general form y = a bx. Graphs of these
equations are plotted in Fig. 6 between the limits x = 0 and x = 12.
Examine these graphs carefully.
I I Note. It is also useful to reduce the equation of a curve, by

'
1
I
substitution, to a linear graph to evaluate the constants, for example:
y=a+b2 let z = 2
y=a+bz i.e. linear
1 I
I 1 similarly y = a + b f i let z = f i

DETERMINING THE EQUATION TO A


STRAIGHT LINE GRAPH
Instead of drawing a graph to a given equation proceed to find the
equation to a given straight line graph.

Example. Find the equation to a straight line graph which passes


through the points (2, 4), (10, 7).
Plot the two points, x = 2, y = 4, and x = 10, y = 7, and draw the Fig. 6
straight line through them, extending the line to cut the axis oy as shown
in Fig. 7. The equation is of the form y = a + bx. The value of y when
x = 0 is the constant a, from the graph we read a = 3.25. The increase of
92 R E E D ' S M A T HEM A TI C S F O R E N G I N E E R S GRAPHS 93

1f the law of this machine expressing the relationship between effort


aPPlied (F) and load lifted (m) be expressed by the linear equation
F=a + bm, plot a graph representing the above experimental values on
a base of load and from it determine the linear law of this machine.
After plotting the experimental values a straight line is drawn as near
as possible through these points as shown in Fig. 8. Those points not
on this line are probably due to irregularities of the machine or
erron of observation during the experiment.
The value of a reads 3.
The value of b is the increase of effort per unit increase of load.
choosing two points on the line, P I and P2, Q to P2 measures 7.5, P I to
Q measures 100, therefore b = 7.5 + 100 = 0.075.
Hence the linear law of this machine is

Fig. 7 F = 3 f 0.075m Ans.

y per unit increase of x is the value b, to obtain an accurate value take


two points on the graph as far apart as convenient, say at the points
indicated by P I and P2. From Q to P2 is the increase of y when the
increase of x is PI to Q. Thus y increases by 3-75 when x is increased by
10, dividing 3.75 by 10 gives 0.375, hence y will increase by 0.375
when x is increased by 1 and therefore the value of b is 0-375.See Fig. 7.

~ I ' I 1 The equation to the graph is y = 3.25 + 0.375~. Ans.


There are many practical applications of determining the equation to
a given graph such as when performing a series of experiments on a
machine, an engine, or a piece of material under a strength test, to
ascertain how one quantity varies with another. For instance, various
loads can be hung from the lifting hook of a lifting machine and the
effort required to lift each load found experimentally and tabulated, a
graph is then drawn showing how the effort varies with the load, the
equation to the graph is determined and this is the law of that
particular machine.

Example. In an experiment on a certain lifting machine the


following data were observed,

I 1 1 11 Load 20 40 60 80 100 120


i
!
LO* D
Effort 4.5 6.1 7.8 8.9 10.4 12.1 t
I Fig. 8
f
94 REED'S MATHEMATICS FOR E N G I N E E R S GRAPHS 95
GRAPHICAL SOLUTION OF SIMULTANEOUS
LINEAR EQUATIONS
The method of solving simultaneous equations by graphical means
can best be demonstrated by an example:
To find the values of x and y which satisfy the equations,
k + 5 y =34
and 4 x - 3 y = 16
Find the value of y in the first equation,

Find the value of y in the second equation,


4 ~ - 3 y = 16
-3y= 16 - 4 x
3 y = 4 x - 16
......... (ii)
Finding two plotting points for each equation,

y = 6.8 - 0 . 4 ~ Fig. 9
when x = 0 , y = 6.8 - 0 = 6.8
when x = 10, y = 6.8 - 4 = 2.8
I
y = l f x -5f GRAPHICAL SOLUTION OF QUADRATIC EQUATIONS
when x = 0, y = O - 5 ' - - -5; Graphs of equations containing x to the first power are all straight
when x = 9, y=12-53=6+ lines. Graphs of equations containing x to other powers, such as 2
(quadratic equations) and x3 (cubic equations) are curves and obviously
The points x = 0 and x = 10 for the first equation, and x = 0 and x = 9 more than two plotting points are necessary as a guide to the drawing
for the second equation were chosen simply because they appeared to be of a curve; the more plotting points the more accurate can the curve
easy figures for substituting and to produce reasonably sized graphs. be drawn.
x= 0 is obviously the first choice for a plotting point of any graph, the As in previous cases the equation to be solved is first simplified, all
other point could be any value of x but it should be chosen with a view terms are brought to one side in order of descending powers of the
to produce simple figures. unknown, say x, leaving zero on the other side. Replace the nought by y,
The graphs are now plotted as shown in Fig. 9. At the point of that is, let y =the given expression and plot a graph for a series of
intersection of the graphs read the values x = 7 and y = 4 and these are values of x. Where this graph reads y = 0 gives the value of x which will
the only values which are true for both equations. satisfy the equation. The values of x when y = 0 are those points where
Hence, x = 7 and y = 4. Ans. the graph cuts the x axis, the values of y in proximity to these points
96 R E E D ' S MATHEMATICS F O R E N G I N E E R S GRAPHS 97
therefore change from positive to negative or from negative to positive,
the trial values of x for calculating the plotting points for the curve
should therefore be chosen with this in mind. The following worked
examples will clarify this explanation.

Example. Find, graphically, the values of x in the equation,

Simplify and bring all terms to one side,

1- 3 . 5 +
~ 1.96 = 0
Let y = 2 - 3 . 5 +
~ 1.96

Calculate values of y for selected values of x,

Fig. 10
when x = 1 y = 1 - 3.5 + 1.96 = -0.54
when x = 2 y=4-7+1.96 =-1.04
when x = 3 y=9-10-5+1.96=+0.46
when x = 4 y=16-14+1.96 =+3.96 Example. Solve, 2 - 0 . 8 -
~ 3.84 = 0
I.,ety=X2 - 0 . 8 ~ - 3.84
Note that the value of y changes sign between x = 0 and x = 1, and when x = 0, y = 0 - 0 - 3.84 = -3.84
again between x = 2 and x = 3, therefore the two values of x will be
when x = l , y=l-0.8-3.84=-3.64
obtained from the graph at these two intersections of the x axis and
hence there is no need to plot the graph beyond the limits of x = 0 and when x = 2, y = 4 - 1.6- 3.84 = -1.44
+
x = 3. The larger the scale of the graph the more accurate will be the when x = 3, y = 9 - 2.4 - 3.84 = +2.76
reading of the values of x therefore the graph should be plotted to the
largest scale possible on the paper. Note that the value of y changes sign from negative to positive
Results of greater accuracy could be obtained by calculating values between x = 2 and x = 3, therefore there is no need to proceed further in
+
of y for a few points between x = 0 and x = 1, and also between this direction. The other value where change of sign takes place, i.e.
+ +
x = 2 and x = 3 and drawing to a larger scale only those two h m positive to negative, must be when x is a negative quantity.
parts of the curve which cross the x axis. Proceeding then in this direction,
By drawing the graph as shown in Fig. 10, when y = 0, x = 0.7 and
+
2.8. The graph is concave upwards (i.e. x2). when x = -1, y = 1 +0.8 - 3.84 = -2.04
Therefore,x=O.7 or2.8. Ans. when x = -2, y = 4 + 1.6 - 3.84 = +1.76
98 R E E D ' S MATHEMATICS FOR E N G I N E E R S

plowg b e gaph as in Fig. 11 and reading the values of x when y = 0,


x = 2.4 or - 1.6 Ans.
maber metbod of solving quadratic equations, and one which can often
applied to solve cubic and more complicated equations is
demonsmted by the following example.

TO solve 2-4~+1$=0

This can be written


3 -(4x- I$) = O
2
~ ey,t represent x
md y2 represent (4x - 1 $)
then, 2-(4x-l$=O
is represented by, YI - Y Z = 0
Graph plotting points are found for each part:
y1 = 2 ............ (i)

when x=O, YI=O


when x = 1, YI = l
I when x = 2 , y1=4
Fig. 1 1 rI when x = 3 , y1=9
when x = 4 , y1=16
I
I y2 = 4 ~- 1 $ ......... (ii)
i
!
This is a straight line equation and two plotting points only required.
1I
when x = O , y2=0-l$ =-la
when n = 4, y2 = 16 - 1: = 14:
L
100 R E E D ' S MATHEMATICS FOR E N G I N E E R S
- GRAPHS 101

~ 1 - y ~check results by substituting them into the original equation


to see if they are Correct.

GRAPHICAL SOLUTION OF SIMULTANEOUS QUADRATICS


These are dealt with in a similar manner as previously explained
with regard to simultaneous linear equations. Two quadratic equations
could be involved, or one quadratic and one linear equation.

Example. Find the values of x and y which satisfy the equations,


y = 12 + 3x - 0 . 5 2
and y = 14- 1 . 2 5 ~
The extreme values of x to be taken for calculating the plotting points are
chosen by making mental estimates with a view to the two graphs
crossing.
Taking the linear equation fist, being a straight line graph only two
points are needed,
y = 14- 1 . 2 5 ~

when x = 0, y=14-0 = 14
when x = 10, y = 14 - 12.5 = 1.5

Taking the quadratic,


y = 12+3x-0.52

when x=O, y = 12+0-0 = 12


when x = 1, y = 12+3-0.5 = 14.5
when x=2, y = 12+6-2 = 16
when x = 3, y = 1 2 + 9 - 4.5 = 16.5
Fig. 12 when x=4, y = 12+12-8 = 16
2
The curve of y l =x and the straight line representing y, = 4x - 1 are when x=5, y=12+15-12.5 =14.5
now plotted as shown in Fig. 12. when x=6, y = 1 2 + 1 8 - 18 = 12
From the graph it is seen that the difference between the values of
y l and y2 is zero (i.e., y1 - y2 = 0) where the straight line intersects
when x = 7, y = 12 + 21 - 24.5 = 8.5
the curve. when x=8, y = 12+24-32 =4
y~-y~=0whichmeans~-4x+1~=0whenx=~andx=3$, when x=9, y=12+27-40.5 =- 1.5
therefore x = $ or 3;. Ans. This appears to be sufficient. Graph is convex upwards (i.e. -0.5x2).
REED'S MATHEMATICS FOR ENGINEERS GRAPHS 103
DETERMINATION OF LAWS
m e case has already been shown, that of determining the law
connecting effort and load in a lifting machine, this relation was
expressed by a straight line equation and the law found quite simply.
However, in engineering many cases arise where the vanation of one
quantitywith another are more complicated, most of them are
tepresented by curves, but some of these may be reduced to a straight
Me from which the law can be found.
One typical example is that of a mass of a gas being expanded or
in a cylinder, the law connecting the vanation of pressure
a the volume of the gas is increased or decreased by the movement

p Vn = a constant

Writing this equation in log form (see loganthmic equations, Chapter 3):
logp + n log V = log C
and transposing to express logp in terms of the other quantities:
logp = log C - n log V

It is now reduced to a straight-line equation similar to y = a + bx


the vanable logp taking the place of the vanable y
the vanable log V taking the place of the variable x
the constant log C taking the place of the constant a
X
the constant n taking the place of the constant b

A graph of logp on a base of log V is now plotted and the value of


n is determined in the same manner as the value of b would be found
in the equation y = a + bx.

Example. The following ordinates and abscissae were measured


Fig.13
from part of the expansion curve of an indicator diagram off an I.C.
engine, where p is the pressure and V the volume of the gases in the
Drawing the graphs as in Fig. 13 the points of intersection PI and PZ
cylinder. If the law of expansion can be expressed by pVn = C,
produce the values of x and y, thus,
estimate the value of n.

x = 0.5 and y = 13.375 16.6 13 9.7 6.8 4.7


or x = 8 and y = 4 V 0.6 0.7 0.9 1.1 1.4 1.8 2.4
104 REED'S MATHEMATICS FO R EN GIN E ER S GRAPHS 105
Tabulating the values of p and V with their respective logarithms:

When the values of Vare less than unity, their logarithms have
negative characteristics, to plot these they are expressed as all-negative
values. Base 10 logarithms have been used here.
The graph is now plotted as shown in Fig. 14. Note that the lowest
value of lg p is 0.6721, the graph can be drawn to a larger scale by
starting with a value of lg p just a little lower than this, say 0.6, instead
of commencing with zero origin.
Choosing two points on the graph,
decrease of lg p 0.65
n= - 1.3
increase of lg V - 0.5
Therefore, n = 1 -3 Ans.
Thus, lgp-lg C - 1.3 lg V
Written in nominal form this is,
p = c x v-I.3
or, as in the original setting,
pv"3 = C

CURVE SKETCHING
It is often informative to sketch the graph by inspection of the
algebraic equation. For example, from previous, general form
+
y = a bx: this is a straight line graph, +b slopes upwards, -b slopes
downwards, the greater the b value the steeper the slope (gradient), or+
-a is the intercept on they axis (when x = 0).
For the quadratic equation, general form y = cx2 + bx + a: this is a
parabolic graph, +c concave upwards, -c convex upwards, the
+
greater the c value the steeper the slope (gradient), or - a as above,
Fig. 14
+b gives vertical axis of symmetry to the left of the y axis and -b to
the right, roots evaluation gives points on the x axis where y = 0.
106 REED'S MATHEMATICS FOR ENGINEERS GRAPHS 107

TEST EXAMPLES 6 7. Find, graphically, the value of x in the equation,


1. On a common base, plot graphs representing the equations, 2-5x+5:=0
(i) y=2+x
(ii) y = 12 - 1 . 5 ~ 8. Draw the graph ofy = 0 . 5 2 - 2x - 6 between the values of x =- 4
(i ii) y = -1 - 0 . 5 ~
and x= +8.
From the graph read the values of x in the following equations:
(iv) +
y = -4 1 . 2 5 ~
0) 0.52-2x-6=0
all between the limits of x = 0 and x = 12.
(ii) 0.52 - 2x- 4 =0
(iii) 0.52-2x-1 = o
2. Draw a straight line through the two pairs of points ( - 2, 14.5),
(8, - 3) and from it derive the equation to the graph. (iv) 0 . 5 2 - 2x = 0
(v) 0.5X2-2+1 =O
3. Plot a graph using the following values and find the law of the
graph. 9. On a common base draw graphs of y, =x2, and y2 = 3.5 + 2.5x,
between the values of x = -2 and x = + 4; find the values of x in
the equation 2 - 2 . 5 -
~ 3.5 = 0.

10. Find, by graphical means, the values of x and y which satisfy the
4. The following data were taken during an experiment on a small simultaneous equations,
turbine where P represents the power developed and m the rate of y = 0.42 - 3x +2
consumption of steam. Assuming that the relationship between P y = 1.4~-2
and m can be represented by the straight line equation m = a bP,+
draw a straight line as near as possible through the plotted points of
11. Using graphical means only and taking values of x between 0 and
the experimental results, estimate the values of a and b and hence
the law connecting P and m.
+ 6, solve for x and y in the following pair of simultaneous
equations:
3 = 16x
y=5+8x-22

5. Find, graphically, the values of x and y which satisfy the


simultaneous equations,
I Note. Y2 = 16x can be expressed as y = f4&.
suggested scales: x axis 2 cm = 1
yaxis2cm=4

12. By drawing a graph of y = e" and = x 2 solve the equation:


2 e x =1
6. Find, graphically, the values o f p and q in the simultaneous
equations, !
I
Note: plot values between x = 0 and x = 1.
5p - 29 = 5.6
and 2p - 39 = -4.8
CHAPTER 7

TRIGONOMETRY AND GEOMETRY


angle is the corner of two joining lines and the magnitude of an
angle is measured in either degrees or radians.

MEASUREMENT OF ANGLES
A degree is one three-hundred-and-sixtieth part of a circle, hence
there are 360" in a circle.
Figure 15 shows a quarter of a circle which is 90" and is termed a
right-angle, an angle which is less than 90" (Fig. 16) is called an acute
angle, greater than 90" but less than 180" (Fig. 17) is an obtuse
angle, and greater than 180" (Fig. 18) is a rejex angle.

OBTUSE ANGLE
RIGHT ANGLE

One sixtieth part of a degree is termed one minute and the sixtieth
part of a minute is one second, thus,

60 s = 1 min
60 min = 1"
360" = 1 circle.

Symbols are used to represent degrees, minutes and seconds. An


angle of 35 degrees 23 minutes and 15 seconds is written 35'23'15".
In this work, accuracy to the nearest minute is all that is required.
A mdian is the angle subtended by a circular arc of length equal
to the radius.
In Fig. 19, the length of the arc A to B is equal to the radius OB
and OA, and the enclosed angle AOB is 1 rad.
110 R E E D ' S MATHEMATICS F O R E N G I N E E R S

The length of an arc to subtend 2 rad is 2 x radius, in symbols:

If 8 = number of radians
and r = radius
then, arc length = 8 x r
and o = rad/s
angular velocity = N rev/min
= 2nN rad/min

Example. Express in (a) rad, (6) deg, the angles subtended at the
centre of a circle of 50 mm radius, by arc lengths of 50, and 140 mm
respectively.
(i) Arc length of 50 mm:
Fig. 19 arc length - -
50 = l r a d . Ans. (a)(i)
6=
radius 50
(1 I / Similarly, if a wheel of 0.3 m radius turns through 4 rad in 1 s, then a = 57.3" Ans. (b)(i)
111,~ point on the rim moves a linear distance of 4 x 0.3 = 1.2 m/s. In
I I1! symbols: (ii) Arc length of 140 mm:
140
If o = angular velocity 8=---=2.8rad. Ans. (a)($
50
v = linear velocity, 2.8 x 57.3 = 160.44". Ans. (b)(ii)
r = radius
then, v = o r
Example. A flywheel of 1 m diameter is rotating at 120 rev/min.
Circumference of a circle = n x diameter (i) express this in rad/s.
= 2n x radius (ii) find the linear velocity, in m/s, of a point on the rim.

Therefore one circle contains 2n rad

2n rad = 360" = 12.568 rad/s Ans. (i)


360" V = or
1 rad =- = 12.568 x 0.5
2n
= 6.284 m/s Ans. (ii)
112 R E E D ' S MATHEMATICS F O R E N G I N E E R S

TRIGONOMETRIC RATIOS
A right angled triangle is one which contains an angle of 90°, the 1 AB
cosecant 0, abbreviated cosec 0 = =-
longest side (opposite the right angle) is termed the hypotenuse, the sin0 AC
other two sides are termed the opposite and adjacent depending upon 1 AB
which of the other two angles are under consideration. secant 0, abbreviated sec 0 = -- -
cos 0 - BC
1 BC
cotangent 0, abbreviated cot t3 = -=-
tan0 AC

Every angle has its own value of sine, cosine and tangent.
The sum of the angles in any triangle is 180°, therefore if one angle
ofa right angled triangle is 30" the other must be 60" and if one angle
is 450 the other must be 45".
Referring to Fig. 22,

100
sin30° = - 0.5
200
173.2
cos 30" = -= 0.866
200
100
tan 30" = -= 0.5773
Considering angle 0 (Fig. 20), side AC is opposite the angle and 173.2
therefore referred to as such, the other side BC is the adjacent.
Note that the sides are in the ratio, 1 : 2 : 2/5
If angle a is being considered (Fig. 2 I), BC is the opposite and AC is

The ratios of the lengths of the sides of a right angled triangle are
expressed by sine (abbreviated to sin), cosine (abbreviated cos) and
tangent (abbreviated tan), as follows:

173-2mm --+

Referring to Fig. 20,

173.2
sin 60" = -= 0.866

173.2
tan60" = -= 1.732
These ratios are the most often used.
R E E D ' S MATHEMATICS FOR E N G I N E E R S

~fthe sum of two angles is 180" they are said to be supplementary to


other. Thus 80" is the supplementary angle of 100°, 53" is the
supplementm angle of 127", and so on.

Fig. 23
THEOREM OF PYTHAGORAS
~n
a right angled triangle, the square of the hypotenuse is equal to
Referring to Fig. 23, tb sum of the squares of the other two sides.
This may be shown by a small square of side c contained in a
100
sin 45" = -= 0.7071
141.1
+
larger square of side (a b) arranged as in Fig. 25.
100
cos 45" = -= 0.707 1 base x perpendicular height
141.1 The area of a right angled triangle is 2
100
tan 45" =- = 1 .OOO
100
In the triangles of sides a, b and c,

area of one triangle = ab


COMPLEMENTARY ANGLES
area of four triangles = 4 x ab = 2ab
If the sum of any two angles is 90" they are said to be
complementary angles. 60" is the complementary angle to 30°, 50" is
area of large square = (a + b12 = a 2 + 2ab + b2
complementary to 40°, 70" is complementary to 20°, and so on. In area of small square = c2
all such cases the sine of an angle is equal to the cosine of its
complement. The notes refemng to Fig. 22 show that the sine of 30"
is the same as the cosine of 60°, and cos 30" is the same as sin 60".
In general, refemng to Fig. 24,
sin 8 = cos a = cos (90 - 8)
cos8 = sina = sin (90 - 8)
tan8=cota = cot(90- e)

Fig. 24 Fig. 25
R E E D ' S MATHEMATICS FOR E N G I N E E R S TRIGONOMETRY A N D GEOMETRY 117
Area of small square =area of large square - area of 4 triangles RELATIONSHIPS BETWEEN SINE, COSINE AND
TANGENT OF ANGLES
Referring to Fig.
AC BC
sine=- and cos6=-
AB AB

I
The simplest example is that of a right angle.',,
of 3 and base 4,
tf;-nnl-
&IA'&LL5,W v. LLhh
T1 G, ,l,,L&,,t

I and
fJL
is tan 6

sin 6
+." n -

+ (base)2 cos Cl \ I

(hypotenu~e)~
= (height)2
= (opposite)2+ (adjacent12
(hypoten~se)~
:. hypotenuse = J?2+42
-
= 425 =5
+
( A B ) ~= ( A C ) ~ ( B C ) ~

Dividing every term

This is illustrated in Fig. 26.

1 = sin2 6 + cos2 6
usually written, sin 6 + cos 6 = 1
2 2
. . . . . . . . . (ii)
The ratios of the reciprocals are also useful,

+
( A B ) ~= ( A C ) ~ ( B C ) ~

Dividing every term

Dividing every term

Fig. 26 sec2 6 = tan2 6 + 1 ...... (iv)


118 REED'S MATHEMATICS FOR ENGINEERS TRIGONOMETRY AND GEOMETRY 119
Example. Without using calculator or tables, find the cosine and
tangent of an angle whose sine is 0.8.
cosec 0 + cot 0 = 1 +cose
sin t3 + cos2 0 = 1
2 sin 0
simplifying left hand side of the equation:
1 case
cosec e + c o t e = -+-_
sin 0 sin 9
=JETa=fi 1 +cos0
-
= 0.6 Ans. (i) sin 0
sine 0.8 = right hand side of equation. Ans. (i)
tane=---
cos 8 - 0.6
= 1.333 Ans. (ii) (ii) Prove that
sec A - cos A
Take care, sin 8 x sin 0 is written sin2 t3 and not sin e2, = tanA
sin A
cos 0 x cos 8 is written cos2 0 and not cos e2,
and so on, simplifying left hand side of equation:
thus it is the sine or cosine which is squared, and not the angle. 1
c o s ~ cos A
--
set A - cos A -
-
sin A sin A

IDENTITIES - 1 - c0s2A
-
- ,,A -
An identity is an equation that is true for all values of the quantities sin A cos A. sin A
involved in the equation, thus: 2
sin A - sin A
-
- -
cos A. sin A cos A
= tanA
= right hand side of equation. Ans. (ii)
are examples of identities because they are true for any values of x and y.
A trigonometric identity is an equation that is true for any angle, thus (iii) Prove that
the four expressions derived in the previous paragraph: tan 0
sin t3 =
sin 8/ cos 6 = tan 8 4
-
2
sin 0 + cos20 = 1 Simplifying right hand side of equation:
cosec29 = 1 + cot 0
2
tan t3 tan 0
sec 2
e = 1 + tan2 e JiTG23- &a-sect3
sine
- -cose
cos 8
are all trigonometric identities because they are true for any value of x- 1 = sin 8
angle 8, and many more can be proved from the knowledge of these = left hand slde of equation. Ans. (iii)
fimdamental identities and the trigonometric ratios. A few simple
examples are shown below. Note. See also Compound and Double Angles in Chapter 8.
120 REED'S MATHEMATICS FOR ENGINEERS

READING TRIGONOMETRIC RATIOS


-w " w'
TRIGONOMETRY AND GEOMETRY

The values of the sine, cosine and tangent for angles 0" to 90" are
included in Tables (which often contain logarithms). They can also be
obtained directly from a suitable calculator.

SINE VALUES. Note that as the angle increases from 0" to 90°, the sine
increases from 0 to 1. All values of sines are therefore decimal fractions
except for 90" where the sine is of maximum value, i.e., unity.

COSINE VALUES. Note that as the angle increases, the value of the cosine Fig. 27a
gets smaller. The maximum value of the cosine is unity for an angle
of 0°, decreasing to zero for an angle of 90".

TANGENT VALUES. Note that the value of the tangent increases from
zero for 0" to infinity for 90°, below 45" the tangents are decimal
fractions, the tangent of 45" is unity.

ANGLES GREATER THAN 90"


Figures 27b, 27c and 27d show circles of 360" with a radius arm
swept around through given angles, in all cases the angle between the
arm and the horizontal diameter has the same numerical values .- - of
Fig. 27b
sine, cosine and tangent as the given angle. For angles between 0"
and 90" all the ratios have positive values; for angles between 90"
and 180" only the sine is positive, the cosine and tangent are negative
for
- angles
" between 270" and 360" onlv the cosine is ~ositive.the sine
and tangent are negative. As an aid to remember whether the ratios sin 124"23' = sin(180° - 124'23')
are positive or negative, divide a circle into its four quadrants and rrn*"'

memorise the positive values only, thus, All, Sin, Tan, Cos,
respectively, as illustrated in Fig. 27a, the remainder being negative
values.
I cos 124'23' = - c0s(l8O0 - 124'23')
= - cos 55O37'
ANGLES BETWEEN 90' AND 180". Subtract the angle from 180" and = -0.5648
read the values of the resultant angle, these are numerically the same tan 124'23' = - tan(180° - 124'23')
as for the given angle but, although the sine of the given angle is - to* <2"27'
positive, its cosine and tangent have negative values. See Figs. 27a
and 27b.
122 REED'S MATHEMATICS FOR ENGINEERS TRIGONOMETRY AND GEOMETRY 123

b e sine and tangent are negative. See Figs. 27a and 27d.

ample. sin 304'44' = - sin(360° - 304'44')


= -sin55"16'
= -0.8218
cos 304'44' = cos(360° - 304'44')
= cos 55"16'
Fig. 27c = 0.5697
tan 304'44' = - tan(360° - 304'44')
ANGLES BETWEEN 180" AND 270". Subtract 180" from the angle and = -tan55"16'
. .
read the values of the resultant angle. In this case only the tangent of
-:
~1.-
me glverl ar~glc:-
---I- ---:A:-.,. *
I
.
,
1 s puslrrv~,UIG
A-,. ,.-,.I,.--:-,.
alrlc: allu
,...,.
-A-n c,.
CIUJIIIG a 1 c I I G g a L l v c . DGG
= -1.4424
Figs. 27a and 27c.
m the foregoing there is more than one possible solutlon for a
Example. sin203O14' = - sin(203O14' - 180") he, cosine or tangent value.
= - sin23'14' xample (i), given that the sine of an angle is 0.3636, to find the
= -0.3944
cos 203"14' = - cos(203"14' - 180") I% &e
e sine being of positive value for all angles from O" to 180°, the
can lie in the first or second quadrant. Read 0.3636 as the sine of
= - cos 23"14' 1 + 21"g1, but it is also the sine of (180" - 21°19') which is 158"4lr
= -0.9189 is 21'19' or 158'41'. Ans. (i)
tan 203"14' = tan(203"14' - 180') ample (ii), given that the cosine of an angle is -0.3352, to
angle.
= tan23'14'
cosine is negative for all angles from 90" to 270" therefore the
= 0.4293 e can be in the second or third quadrants. Read 0.3352 as the
e of 70°25', therefore,
90°

-0.3352 is the cosine of (180" - 70'25')


= 109"35' Ans. (ii)
+
or 180" 70'25'
= 250°25' Ans. (ii)

xarnple (iii), given that the tangent of an angle is 2.4876, to find the

270° tangent is positive for angles between 0" and 90°, and also
Fig. 27d 180" and 270°, therefore the angle can be in the first or third
124 R E E D ' S M A T H E M A T IC S FOR E N G I N E E R S TRIGONOMETRY AND GEOMETRY
1

quadrants. Read 2.4876 as the tangent of 68"6', therefore,


2.4876 is the tangent of 68"6' Ans. (iii)

= 248'6' Ans. (iii)


From the above the values of the sine and cosine are between zero
and + 1, never numerically greater than unity. The tangent varies from
zero to f infinity. DECREES
It is sometimes useful to express some angles in radians instead of Fig. 29
degrees.
360" = 271 rad
180" = 71 rad height above the horizontal axis represents the value of sine 0 when 0 is
90" = 7112 rad etc. the angle between the arm and the horizontal. Since the cosine of an
mgb is equal to the sine of its complement, the vertical height above the
horizontal will represent the value of cosine 0 if 0 is taken as the
mgle between the arm and the vertical axis. Thus, although graphs
GRAPHICAL REPRESENTATION of sine and cosine can be plotted by taking values from the calculator
or tables, a quicker and more convenient method is to project their
A little more study of the previous circle diagrams, and also Fig. 28,
values from a circle diagram as shown in Fig. 29.
will show that if the radius arm of the circle is unity, then the vertical

Fig. 28 Fig. 30
REED'S MATHEMATICS FOR E N G I N E E R S
127
- TRIGONOMETRY A N D GEOMETRY
It will be seen that, with a proper understanding of the positive
and negative values, a half-circle only is required from which to Meridians of Longitude are circles around the earth passing through
project all points from 0" to 360". be ~0 poles and cutting the equator at right angles. The Greenwich
~ a i d i a nis the meridian of longitude which passes through Greenwich
Further, the tangent of an angle being opposite s adjacent, then
the vertical height (opposite) = adjacent x tan 8. Hence if the base ad this is taken as the standard from which other meridians of
line Ox is made constant at unity, the vertical height of the longitude are measured, East and West of it, from 0" to 180".
radius-arm projected as shown in Fig. 28 will represent tan 0. The ~ h u any
s position on the earth's surface can be defined by its latitude
graph of y = tan 8 is shown in Fig. 30. ad its longitude. Referring to Fig. 3 lb, let NGLS be the Greenwich
Meridian. Angle QOP is the degrees latitude of point P, North of the
cpmtor. Angle LOQ (which is the same as GMP) is the degrees
longitude of P, East of Greenwich.

I i/ LATITUDE AND LONGITUDE


The shape of the earth is approximately spherical, a straight line
I
joining the North and South Poles and passing through the centre of
the earth is called the Polar Axis. The earth rotates about its polar axis
in the direction from West to East, that is, anticlockwise looking down
on to the North Pole, and makes one complete revolution once in
(approximately) every 24 h.
Parallels of Latitude are circles around the earth, perpendicular to
the polar axis. The parallel of latitude midway between the two poles
is the Equator and this is the standard from which other parallels of
latitude are measured, North and South of it, from 0" to 90" either way.

Fig. 31b

Since the earth rotates in the direction West to East and the sun
being a fixed body, any point on the earth's surface East of another
will pass a direct line of the sun first. Hence at places East of
Greenwich the time is ahead of the time at Greenwich, and at places
West of Greenwich the time is behind.
Taking the earth to rotate once in 24 h:

360" = 24 h
15" = 1 h
1" =& h=4min
R E E D ' S MATHEMATICS FOR E N G I N E E R S

As an example, the longitude of Karachi is 67" East, therefore th,


difference in time between Greenwich and Karachi is 67 x 4 min =q h
28 min, and if the clocks were set by the sun, it would be 4.28 p.m.
in Karachi when it is 12 noon at Greenwich.
It is obvious that different parts of one country should not have
different times by the clock to other parts of the same country,
modifications are therefore made. The time by the sun at Greenwich
is referred to as Greenwich Mean Time (G.M.T.), all clocks in Britain
are set to G.M.T. in the winter, and put forward one hour in the Fig. 32b
summer.

THE NAUTICAL MILE. This is the length of an arc on the earth's surface
I
subtending an angle of 1 min (g)at the centre of the earth.
Figure 32b shows the same direction expressed as a three-figure
Taking the circumference of the earth as 40 000 km bearing, 097".
I
1 1 360" around surface = 40 000 km
40 000
I GEOMETRICAL CONSTRUCTION OF TRIANGLES
1 min around surface = Example. To construct a triangle of sides 64 and 48 mm and an
360 x 60
mde 0f42" opposite the 48 mrn side, and to measure the remaining
side and angles.
This is the intemational nautical mile which is the nautical unit of Referring to Fig. 33, draw the base Ox, measure 42" and draw Oy.
distance. Mark A at 64 mrn from 0. With centre A and compasses set to a radius
Aircraft and ship speeds are expressed in naut. miles/h, which are of 48 mm,describe an arc to cut Oy, here the arc will cut Oy at two
termed knots, one intemational knot being equal to 1.852 km/h. points B and C. Join AB and there is one triangle OAB which satisfies
the description. Join AC and now there is another triangle OAC

COMPASS BEARINGS
A direction expressed with regard to the points of the compass is
termed a bearing and it is usual to give a bearing in one of the two
following methods.
(i) Stating the angle in degrees less than 90" from either North or
South (whichever of the two is the nearer direction), the angle thus
lies within one of the four quadrants and is therefore called a
quadrant bearing.
(ii) Stating the angle in degrees measured clockwise from North and
expressing this in three figures, using noughts for hundred, tens or
units when there are none. This is called a three-figure bearing.
To illustrate these terms, Fig. 32 shows the direction of movement
in relation to the four cardinal points of the compass.
Figure 32a is the quadrant bearing and may be stated as 83" East
of South, or South 83" East which is abbreviated to S 83" E. Fig. 33
BE TRIGONOMETRY A N D GEOMETRY

which also satisfies the description. Hence there are two possible
angle formed by extending one side of a triangle
solutions to this example.
+he sum of the two opposite internal angles. Figure 36
Measuring the remaining side and angles:
I
For one triangle,
Remaining side, OB = 26 m m
Remaining angles, OAB = 21 " and OBA = 1 17"
I - -
For the other triangle,
Remaining side, OC = 69 mm
Remaining angles, OAC = 75" and OCA = 630
Fig. 36

SOME IMPORTANT GEOMETRICAL FACTS fan angle between two sides of a triangle is bisected and the
g line produced to meet the opposite side, the opposite side is
The opposite mgles of the intersection of two smight lines are equll. into two parts having lengths of the same ratio as the two sides
See Fig. 34 and note also that c~= 1800 - 0.
form the bisected angle. Thus, referring to Fig. 37, angle B is
and side b is divided into two parts x and y, and the ratio x toy is
the ratio a to c.

Fig. 37

CROSSED CHORDS
Referring to Fig. 38, the triangles CAB and CDB have the common
ase CB which is a chord of the circle, therefore the angles at the
es of these triangles where they touch the circumference of the
le are equal (see Fig. 42). Angle CAB =Angle CDB. Also, at the
rsection of the two straight lines AB and CD, angle COA =angle
OD (see Fig. 34). Now considering triangles COA and BOD, since
angles of each of these are equal, the remaining angle of each
Fig. 35 ust be equal, therefore angle ACO = angle DBO.
132 REED'S MATHEMATICS FOR ENGINEERS A N D GEOMETRY 133

Fig. 38 Fig. 39
In such similar triangles, the sides opposite the corresponding angles
1 Fig. 40 Fig. 41

of the case where the chords meet outside


must be in the same ratio, hence for two non-parallel chords of a bteresting
a given point
Ihe circle is to estimate the distance of observation
lbave the earth's surface to the horizon. Figure 41 shows a circle
A 0 DO wSenting the earth, AB is one chord which is the diameter, say
CO - BO 12 750 km, this is extended to the observation point 0, let this be 24 m
.'. A 0 x BO = CO x DO lbe the earth's surface. The other chord is imagined to be so small
where A and B are the points where the two ends of the chord AB touch it becomes tangential to the circle, i.e., the length of CD is nil
the circle, C and D the points where the two ends of another chord CD md p b c ~ and D are at one. The extension of this chord n~eetsthe
touch the circle, and 0 is the point of intersection of these two chords. at 0.co (or DO) is the distance from the point of ~b~ervation
This rule of crossed chords is also true when the intersection of the to the horizon.
chords occurs outside the circle as illustrated in Fig. 39.
A usefUl application of the case where the chords intersect inside the
circle is the estimation of the diameter of a circular vessel when a rule of 24 m = 0.024 km
sufficient length is not available. Thus, one chord is made the diameter AOxBO=COxDO
itself and this cuts another chord perpendicular to it as in Fig. 40. an (12 750.024) x (0.024) = CO x DO
example, let be the diameter of a vessel, a 1 m straight-edge is laid
horizontally across to form the chord CD, the vertical distance beheen
bottom of vessel and straight-edge is measured to be 100 -, this ~ d d 0.024
b ~ to 12 750 makes no difference in four-figure accuracy,
i.e., 12 750.024 can be taken as 12 750.
AOXBO=COXDO
A 0 x 100 = 500 x 500
A 0 = 2500 m co x DO = ( ~ 0 ) ~ b e c a u sCO
e is equal to DO
Diameter of vessel = AB = AO + BO :. 12750 x 0.024 = ( ~ 0 ) ~
= 2500 + 100 co = 4 1 2 750 x 0.024
= 2600 mm or 2.6 m = 17.49 km
TRIGONOMETRY AND GEOMETRY 135
B-
mtre of the circle and the apex of the other touching the circumference,
the angle at the centre is twice the angle at the circumference. Briefly:
The apex angles of all triangles at the circumference
. withir1 the same
. ~h~ angle at the centre of a circle is double the angle at the
segment of a circle and having the same chord as a base, are equal. for triangles in the same segment standing on the same
Stated briefly: Angles in the same segment of a circle are equal. See
I See Fig. 44.
Fig. 42.

Fig. 42
I Fig. 44

The sum of either pair of opposite angles in a quadrilateral (i.e., a


If the base of the triangle is the diameter of the circle, then the apex
angle at the circumference is 90". Stated briefly: The angle in a four-sided figure) inscribed in a circle is equal to 180". This is illustrated
c , = m iV-L~
.,VAL-
i r ~ l i'Y
pc .a.r i ~ h tanult. Set.
.I-D--. --- Fjp
--0.
43 in Fig. 45 with measured angles. Briefly: The opposite angles of a

I1
LV..,

If two triangles be drawn within the same segment of a circle and cyclic quadrilateral are supplementary. It can also be seen that any
---- chord
having nne
a&-. A-- - -- - - - - - their
as - -
common base. the a ~ e xof one being at the exterior angle produced by extending one side, is equal to the
opposite interior angle.

Fig. 43 I Fig. 45
136 R E E D ' S MATHEMATICS FOR E N G I N E E R S TRIGONOMETRY A N D GEOMETRY 137

As an exercise on the last few cases, measure the angles from Fig. 46.

ADB = AEB, ACB = 2 x ADB, AFB = AGB.

There are a number of cyclic quadrilaterals here, ADBF, ADBG,


AEBF and AEBG.
Measure any remaining angles in these quadnlaterals and check
that all opposite angles are supplementary.

Fig. 47

INSCRIBED CIRCLE
If a triangle is drawn around a circle such that the three sides of the
triangle just touch the circumference of the circle, the bisectors of the
three angles meet at the centre of the circle. See Fig. 48.

SIMILAR TRIANGLES
Triangles are similar when they are equiangular (i.e., contain the
same angles). It follows that their corresponding sides are in the same
proportion and the shape of the triangles are the same.

Fig. 46

CIRCUMSCRIBED CIRCLE
If a triangle is drawn inside a circle with its three corners touching the
I
circumference, the perpendicular bisectors of the three sides meet
at the centre of the circle. See Fig. 47. Fig. 48
REED'S MATHEMATICS FOR ENGINEERS

To ascertain whether two triangles are similar, apply any of the


following tests to see if
(i) they are equiangular, parallel to one side, they divide the other two sides into
(ii) their corresponding sides are in proportion,
(iii) one angle in each triangle are equal and the sides containing ting parallel lines by arrows is used.
these angles are in proportion. , XYZ and PQR are similar triangles and their sides are in the
Figure 49 shows some pairs of similar triangles, in each pair triangle
ABC is similar to triangle XYZ.

Congruent triangles have the same shape and size, and thus contain
the same area. Figure 51 illustrates some congruent triangles.

Fig. 5 1
140 R E E D ' S MATHEMATICS FOR E N G I N E E R S TRIGONOMETRY A N D GEOMETRY 141

TEST EXAMPLES 7 8. Find the values of 8 between 0' and 360" which satisfy the
1. (i) Express the following angles in rad,
2
cos8-sin 8=O
114'36', 286'30'.

(ii) Express in (a) rad, (b) deg, the angles subtended by arc lengths
of 10, and 30.4 m respectively, on a circle of 10 m diameter. I 9. ~fa = 2 sin 8, and b = 5 cos 8, find the values of 8 for angles
between 0' and 180" when:

2. A disc flywheel is rotating at a speed of 10.52 rad/s. Calculate the


linear velocity, in m/s, of points on the wheel at radii of 100 and
500 mm respectively. Find the linear velocity in m/s of the rim at +
10. Prove that c o s e c 2 ~ sec2 A = c o s e c 2 sec2
~ A
2 m diameter when rotating at 125 rev/min.
1 1. Prove that
3. In a right angled triangle ABC, length AC is 36 mm,length BC is 1 - sinA = (sec A - tan A ) ~
27 rnm, and the angle at C is 90'. Calculate the length of the +
1 sinA
hypotenuse AB and the sine, cosine and tangent of the angle at B.
12. Prove that:
4. Find, without the use of calculator or tables, the sine and tangent 1
of an angle whose cosine is 0.4924. (sec A - cos A)(cosecA - sin A) =
tanA + cotA

5. Write down the sine, cosine and tangent of the following angles,
10"33' 46"55' 150'47' 201'21' 287'14'
I 13. The velocity of the piston in a reciprocating engine is given by:

Sketch sine and cosine curves for angles between 0" and 360".
and the acceleration is given by:
6. The following refer to angles between 0" and 360°,
(a) find the angles whose sines are:
where w = angular velocity of crank in rad/s,
(i
r = throw of crank stroke) in m,
(b) find the angles whose cosines are: 8 = angle of crank past top centre, in deg,
0.9687, -0.8769, n =ratio of connecting rod length to crank length.
Calculate (i) the velocity, (ii) the acceleration, of the piston of an
(c) find the angles whose tangents are: engine of 1 m stroke, connecting rod length 2 m, at the instant the
crank is 80" past top centre and running at 150 rev/min.
0.2010, -3.2006,

7. If 8 = 80°, find the values of I 14. Construct a triangle, ABC, such that AB =48 rnrn, BC = 39 m m
and the anale at A = 50°, measure the remaining side and angles. A
142 REED'S MATHEMATICS FOR ENGINEERS

15. A trammel gauge supplied by the makers of an engine as a


measurement of the diameter of a cylinder when new, was exactly
750 mm.The cylinder is now gauged and, with one end of the CHAPTER 8
trammel held against the cylinder wall, there is 78 mm chord travel
at the other end. Find the wear (increase in diameter). SOLUTION OF TRIANGLES
16. Find the distance from an observation point 30 m above sea level, to TO solve a triangle means to find the other sides and angles of the
the horizon, assuming the earth is 12 750 km diameter. triangle remaining from the data given. Of the three sides and three
angles of any triangle, at least one side must be given with two other
17. Construct a triangle of sides 56, 48 and 40 mm long respectively, quatltities; the two other quantities may be the other two sides, or one
bisect each side to find the centre of a circle which will pass other side and one angle, or two of the angles.
through the three points of the triangle, draw the circle and A sketch of the triangle is advisable to understand the problem clearly
state its radius. and a scale drawing, especially in the early stages of learning, gives a
check on calculations.
18. Construct a triangle of sides 68, 58 and 42 mm long respectively,
bisect the angles to find the centre of a circle which will touch the RATIOS FOR RIGHT ANGLED TRIANGLES
three sides of the triangle, draw the circle and state its radius. Figure 52 is a right angled triangle because one of its angles is 90"
and one of the angles (other than the right angle) is given, therefore the
side opposite this angle is referred to as the opposite and the other side is
the adjacent.
The ratio of the sides are (see Chapter 7):
I
I
opposite
I
Sine = hypotenuse

adjacent
Cosine =
hypotenuse
opposite
Tangent =---
adjacent
144 R E E D ' S MA TH EM A T IC S F O R E N G IN EE RS SOLUTION OF TRIANGLES 145

Example. One of the angles in a right angled triangle is 59'42' Since the three angles of any triangle add up to 1 80°, the third angle
and the hypotenuse is 55 mrn. Find the remaining angle and sides. is found by subtracting the sum of the other two from 180".
Third angle = 180" - (90" + 59'42')
= 90" - 59"42'
=30"18' Ans.

Example. A light on a cliff is viewed from a position at sea, the


angle of elevation being 32"15'. At a point 100 m fixther away from
the cliff and in direct line with the first observation point, the light is
viewed again and the angle of elevation is now 20°44'. Find (a) the
height of the light above sea level, and (b) the horizontal distance
h m the second observation point to the light.

ADJACENT

Fig. 53

Referring to Fig. 53

opposite
= sin 59'42'
hypotenuse Fig. 54
opposite = hypotenuse x sin 59'42' Refening to Fig. 54,
= 55 x 0.8634
= 47.49 mm Ans.

The remaining side can now be found by the Theorem of Pythagoras:


--
CD
- tan 20°44'
(hypotenuse)' = (opposite)' + (adjacent)' 100 + x

:. adjacent = 2/55' - 47.49'


+ x 0.3786
.'. CD = (100 X)

CD = 37.86 + 0.3786~ (ii)


From (i) and (ii),
or by using the cosine ratio: 0.6309~= 37.86 + 0.3786~
adjacent 0.2523~= 37.86
= cos 59"42'
hypotenuse x = 150.1 m
adjacent = 55 x cos 59"42' AC = 100 150.1 +
= 27.75 mm Ans. Horizontal distance = 250.1 m Ans. (b)
REED'S MATHEMATICS FOR ENGINEERS S O L U T I O N O F TRIANGLES 147

Refefing
to Fig. 55, and considering the right angled hangular
CD = x x 0.6309
= 150.1 x 0.6309
Height = 94.69 m Ans. (a)
a
Considering the right angled triangular Part ABD7

TRIANGLES OTHER THAN RIGHT ANGLED


If the triangle does not contain one angle of 90°, the sine, cosine C

and tangent ratios of the sides cannot be applied as in the above right
angled triangles. Other rules are employed which will now be
asinC = csin A
The commonest method of notation of triangles is to let capital leaeIs
represent the angles and their corresponding small letters represent
the sides opposite to these triangles. Thus in Fig. 55 the three comers
a
-_- - C
... . . .
are lettered A , B and C,this means that the three angles at these (i)
comers are represented by these letters. The side opposite angle A is sin A sin C
represented by its lower case a, the side opposite angle B is
F i w e 56 represents the same triangle as Fig. 55, but with a
represented by b, and the side opposite C is represented by c. A
perpendicular erected from the base of the triangle to its apex is perpendicular h2 from side c to angle C.
usually denoted by h, this perpendicular divides the triangle into two
right angled triangles which can often provide a solution by the
knowledge of right angled triangles only.

Fig. 56

Considering the right angled triangular Part BCE,

k?=SinB ... h , = a s i n B
a
148 R E E D ' S M A T H E M A T I C S FOR E N G I N E E R S

considering the right angled triangular part ACE,

sin A - sin C
sf '&fir c sin A
a=-
hence, sin C
asinB = bsin A

B = 180" - (40" + 45") = 95"


a b b c
... ... . . . (ii)
sin A - sin B sin B - sin C

a b c
From (i) and (ii): -= -- -
sin A sin B - sin C
This is the SINE RULE which is very important and most usehl in
trigonometrical calculations. Note that the length of any side is
proportional to the sine of the angle opposite that side, the longest side is
opposite the largest angle, and the shortest side is opposite the
Length of tie = 9.088 m
Length of jib = 14.09 m
1 Ans.

smallest angle.
Example. Two ships leave the same port at the same time on
Example. The vertical post of a jib crane is 10 m long. The angle courses which diverge at 29". When one ship had travelled 40 naut.
between jib and post is 40°, and between jib and tie the angle is 45". miles the two ships were then 21 naut. miles apart, find how far the other
Find the length of the tie and the length of the jib. ship had travelled from port.
150 R E E D ' S MATHEMATICS FOR E N G I N E E R S

Refemng to Fig. 58, ~ ~ f ~ to


m Fig.
n g55 and considering the right angled triangular part
a c ABD, let AD be represented by x, then by the Theorem of Pythagoras,
--
sin A -
c sin A h2=c2-2 . . . . (i)
sin C = -
a
N~~ considering the right angled triangular part BCD,
- 40 x 0.4848 = 0.9234
21 DC=b-x
0.9234 is the sine of 67'26' but is also the sine of
180" - 67'26' = 112O34'.
I h2 = a2 - (b - X)2
= a2 - (b2 - 2bx + 1 )
Therefore there are two possible answers to this problem, as
illustrated in Fig. 58. = a2 - b 2
+ 2bx - 2 . . . . . . (ii)
If C = 67"26':
+
B = 180" - (29" 67'26') = 83"34' From (i) and (ii)
a b
sin A - sin B
a sin B
b=-
sin A
but X. = c x cos A, therefore,

a 2 = b2 + c2 - 2bccos A
This is the COSINE RULE and equally important as the sine rule
for solving triangles. It will be obvious that this can be written in
a sin B either of the three forms:
b=-
sin A
- 21 x 0.6216 = 26.93 (ii)
0.4848
For the two ships to be 21 naut. miles apart and one having travelled
40 naut. miles, the other ship has travelled:
either 43.04 or 26.93 naut. miles. Ans.
I Note that the last term is a negative value, if the angle involved is
between 90" and 180°, its cosine is negative, and -2bc multiplied by a
minus quantity will produce a positive term.
COSINE RULE Transposing the cosine rule to find one angle given the three sides:

Inspection of the Sine Rule will show that it would not solve a
triangle when two sides and the included angle between these sides
were the only given quantities, nor if the given quantities were the
three sides without any angles. Such cases can be solved by the
Cosine Rule which will now be explained.
ATHEMATICS FOR ENGINEERS S O L U T I O N OF T R I A N G L E S 153

a 2 + c 2 - b2
cos B =
2ac

or, cos C =
a2 + b2 - c2
2ab

Example. A ship going due North at 16 knot runs into a 5 knot


current moving North-East. Find the resultant speed and direction of the
ship.

Reference Fig. 59,

Angle A = 90" + 45" = 135"


Fig. 59
a 2 = b2 + c 2 - 2bccosA
+
= 1 6 ~ 52 - 2 x 16 x 5 x cos 135"

Example. The three sides of a triangle measure 8, 6 and 4 cm


respectively. Find the angles.

= 19.85 knot

--
a -- C
sin A sin C
c x sin A
sin C =
a

Resultant speed = 19.85 knot


Resultant direction = 10"15' East of North
] Ans.
Fig. 60
154 R E E D ' S MATHEMATICS FOR E N G I N E E R S

Angle A = 46'34'
I Refemng to Fig. 62,
SOLUTION OF TRIANGLES 155

a b
sin A - sin B
b sin A I Perpendicular height = a x sin C
sinB = -
a I Area = $ (b x a sin C)
ab sin C
.'. Area=- 2

Angle B = 28'57'
Note this formula, in words it is 'half the product of two sides
Angle C = 180' - (46"34' + 28'57') and the sine of the angle between these sides'. An easy formula to
apply and a very useful one.
Another important formula for finding the area of a triangle is one
which calculates the area directly from the three sides of the triangle:
The three angles are:
46"34', 28'57' and 104'29' Ans. Area = Js(s - a)(s - b)(s - c)
Summarising the formulae given so far for solving triangles, the sine, where a , b and c = the three lengths of the sides respectively,
cosine and tangent ratios of sides are used for right angled triangles, and
either the sine rule or cosine rule for other triangles. The sine rule is 1 and s = semi-sum of sides,
easier to calculate than the cosine rule and is therefore used wherever
possible, the two cases where the sine rule cannot be applied and the
cosine rule becomes necessary are when the only data available is:
(i) two sides and the angle between these sides,
(ii) the three sides. Example. The three sides of a triangle measure 8, 10 and 14 cm
respectively, find the area enclosed.
AREAS OF TRIANGLES
It can be seen from any of the previous illustrations of triangles,
or from Figs. 61 and 62 that the area of a triangle is half of the area of
its circumscribing rectangle, therefore,
Area of triangle = $(base x perpendicular height)

r ----

- BASE -+
Area = Js(s - a)(s - b)(s - c)

= 39.2 cm2
I Fig. 61 Fig. 62 Ans.
'I
156 R E E D ' S MATHEMATICS FOR E N G I N E E R S SOLUTION OF TRIANGLES 157

EQUILATERAL TRIANGLE B
An equilateral triangle is one which has three equal sides and three
equal angles. Each angle is therefore 60" (Fig. 63).

Perpendicular height = side x sin 60"


= side x 0.866
Area = (base x perp. ht.)
= ;(side x side x 0.866)
= 0.433 side2
A C
Fig. 64

slot
Fig. 63

ISOSCELES TRIANGLE
An isosceles triangle is one which has two equal sides and two
equal angles.
h Fig. 64 angles A and C are equal, therefore the lengths of
the sides a and c are equal.
Fig. 65
A= C = i ( 1 8 0 - ~ )
It is thus an easy matter to calculate the perpendicular height or other Since the angle at the centre is twice the angle at the apex, that is,
data required to find the area by any of the general formulae. angle COB = 2A, then angle COD =A.

CIRCUMSCRIBED CIRCLE a
- = sin A
To find the radius of a circle whose circumference will pass through 2R
the three angular points of a triangle. This construction was shown in a
.'. 2R =-
Chapter 7 (Fig. 47). Let the given triangle be ABC as shown in Fig. 65. sin A
I
1 <Q RFFn7< M A T H F M A T I C S FOR ENGINEERS cnr I ~ T T A N Tn c r n r A xrnr rn . -,.
tl I Therefore, by sine rule, I Finding the area of each triangle by the rule,
a b c
. . . ... i
area = (base x perp. height) and then adding together,

-
2R=-=--- (i)
sin A sin B - sin C
~reaofBOC=ixax~
Also, area of triangle = jab sin C
P
Area of COA = x b x R i
Substituting the value of sin C =
2R heaofAOB=;xcx~
I
I c Area of whole triangle ABC = ; ~ ( +
a b + c)
1 heaoftriang1e=ixaxbx-
2R 2 x area nf trinntrlp
abc
:. R = 4 x area of triangle ... (ii)
or, -
K =
2 x area of trianele
"
perimeter of triangle
INSCRIBED CIRCLE
To find the radius of a circle inscribed within a triangle, this COMPOUND ANGLES
construction was shown in Chapter 7 (Fig. 48). In some problems it is often convenient to express the relationship
Let R = radius of the circle between the trigonometrical ratios of a compound angle and those of the
Let a, b and c = lengths of the sides of the triangle. two single component angles.
Referring to Fig. 67,

Fig. 67

Area of CED = area of CFD area of CFE +


+
3 [ed sin (A B)] = $ (ge sin A) + $ (gd sin B)
1

Fig. 66
i
ividing throughout by ed,

Dividing the triangle ABC into three small triangles BOC, COA and
AOB as shown in Fig. 66, the radius of the circle R is the perpendicular
height in each of these.
R E E D ' S MATHEMATICS FOR E N G I N E E R S S O L U T I O N OF T R I A N G L E S 161
-
Usually written, Dividing all terms, top and bottom, by cos A cos B,
sin A cos B cos A sin B
sin(A+B)=sinAcosB+cos~sin~ ... ... 6) -cosAcosB
-
+
cosAcosB
By a similar process it can be shown that, cos A cos B sin A sin B
cos A cos B - cos A cos B
sin(A-B)=sinAcosB-cosAsinB ... . (ii)
- tanA+tanB
- ... ... ...
Again referring to Fig. 67 and applying the cosine rule, 1-tanAtanB (v)
and by a similar process,
cos C =
2 + d 2 - c2 tanA-tanB
2ed tan(A- B) = l+tanAtanB (vi)
+
cos (A B) =
+
2 d2 - (a + b)2
collecting the above formulae:
2ed
-
-
e2 + d 2 - a 2 - 2ab - b2 sin(Af B) = sinAcosBf cosAsinB
2ed cos(Af B) = c o s A c o s B ~ s i n A s i n B
but, by Pythagoras, e2 - a2 = 2 t a n A f tanB
tan(Af B) =
and d 2 - b2 = g2 1 tanAtanB

.'. cos(A + B) = 2 2 - 2ab


2ed DOUBLE ANGLES
2
- - - -ab
g
ed ed Formulae expressing the relationship between the ratios of angles
and double the angles are useful and can be obtained from those
- - x - - g- x - a b
- g
above by letting A = B, thus,
e d e d
= cos A x cos B - sin A x sinB sin (A + B) = sin A cos B + cos A sin B
Usually written, B =A, therefore,
sin 2A = sin A cos A + cos A sin A
cos(A+B)=cosAcosB-sinAsinB ... . (iii)
sin2A = 2sin Acos A .. . . . . .. . (9
By a similar kind of process it can also be shown that, cos(A+ B) = cosAcosB- sin AsinB
cos(A-B)=cosAcosB+sinAsinB . . . . . . (iv) B =A, therefore,

The tangent of an angle being the sine divided by its cosine, the cos2A = cos AcosA - sinAsinA
relationship for the tangent of a compound angle can be obtained by cos 2A = cos2 A - sin2 A ... . . . ... (iia)
+
dividing sin (A B) by cos (A B),+ Also, since I
sin (A + B) sin A + cos2A = 1 (see Chapter 7)
tan(A + B) =
+
cos (A B)
2

then, cos2A = 1 - sin2 A


-
sinAcosB+cosAsinB
cos AcosB- sin AsinB and, sin2 A = 1 - cos2 A
SOLUTION OF TRIANGLES 163

cos 2A = cos2 A - sin2 A so the height of the flagstaff.


1
2
= (1 - sin A) - sin2 A The length of the sides of a cube is 60 mm. Find the length of
cos2A = 1 - 2 s i n 2 ~ . . . . . . . . . (iib) the diagonal across the face and the length of the cross diagonal I

from one comer to the opposite comer passing through the centre

cos 2A = cos2 A - sin2 A


6. The length of the vertical post of a jib crane is 15 m. The angle
= cos2A - (1 - cos2A)
between jib and post is 35"301and between tie and post the angle is
= cos2A - 1 + cos2A 105030'. Calculate the lengths of the jib and tie.
c o s 2 ~ = 2 c o sA~- 1 . . . . . . . . . (ii,)
tanA+tanB
+
tan(A B) =
1 - tanAtanB
the top of its stroke when the crank is 35' past its top dead centre.
B =A, therefore,

tan2A=
tanA tanA +
1 -tanAtanA
2tanA
tan2A = . . . . . . . . . (iii) position of the crank from top dead centre when the valve closes.
1 - tan2 A
Note. Also see Identities in Chapter 7.

TEST EXAMPLES 8
1. The top of a vertical mast is viewed from a position at 15 m from its
. Rvo ships approach a port, their courses converging at an angle
base on a level ground and the angle of elevation measured to be of 23". At a certain time one ship is twice as far from port as the
45'34'. Find the height of the mast.
other and their distance apart is 32 naut. miles; find how far each
2. A boat is sighted from a point on a cliff 95 m above the sea, the
angle of depression of the line of view being 14'25'. Find (i) the
horizontal distance from cliff to boat, and (ii) the distance from
observation point to boat.
73'39'. If one ship will arrive in port half-an-hour before the
other, find their distances from port.
3. A right angled triangular plate has one angle of 28'37' and the
length of the hypotenuse is 120 mm. Find the lengths of the other
two sides and the area of the triangle.

4. The top of a flagstaff is viewed from a point on a level ground at


some distance from the foot of the staff, and the angle of elevation is . Two sides of a triangle measure 6.5 and 7.5 m respectively and
48'30'. At another point 10 m hrther away from the foot, the staff the angle included between these two sides is 46'51'. Find the
top is viewed again and this time the angle of elevation is 3T38'. area of the triangle.
164 R E E D ' S MATHEMATICS FOR E N G I N E E R S

14. Find the area in cm2 of a triangular plate of sides 71,42 and 53 mm
long respectively.
CHAPTER 9
15. The area of an equilateral triangle is 57.27 cm2. Find the length of
its sides. MENSURATION OF AREAS
16. From,
cm2 is a very practical size, between m2 and mm2, and is often
sin 8 2
+ cos2 8 = 1 used. Note the scale of areas:
and, cos 28 = cos2 8 - sin2 8 1 c m = 10mm .: 1 cm 2 = 102mm2
2
I Prove that (i) cos 28 = 1 - 2 sin 0 1 m = 1 02 cm :. 1 m 2 = 1 0 4 c m 2
2
(ii) cos 28 = 2 cos 8 - 1 1 m = lo3 mrn ... 1 m 2 = 106mm2
and, +
(iii) find the value of (1 col? 8)(1 - cos20) Note also
I
17. Given that,
sin 28 = 2 sin 8 cos 8
and, cos 28 = cos2 8 - sin2 8 A PARALLELOGRAM is a four-sided figure whose opposite sides
are parallel and equal in length to each other. It therefore follows that
2tane
prove: (i) sin 28 = opposite angles are equal, one pair of opposite angles being obtuse,
1 +tan2e and the other pair acute and supplementary to the obtuse angles. It may
1 - tan 2 8 be considered as a rectangular framework leaning over to one side as
(ii) cos 28 =
1+tan28 in Fig. 68 wherein it can be seen that the outer triangular area (shown
dotted) at one end is equal to the inner triangular area at the other
and, (iii) find the value of 0 between 0" and 90" end. Hence the area of the parallelogram is equal to that of a rectangle
which satisfies the equation: of the same base and same perpendicular height. Also, if a diagonal is
drawn from one comer to the opposite comer, it will bisect the
sin28+2cos28 = 1
parallelogram ixto two equal triangles, the area of each being half that of
the parallelogram.
18. Prove,
Area of Parallelogram = base x perp. height
sin38 = 3sin8-4sin3 8

7
- y , BASE ,*
PERPENDICULAR
MLICUT

?l1ifll
ill I 1 1 I Fig. 68
R E E D ' S MATHEMATICS FOR E N G I N E E R S M E N S U R A T I O N OF A R E A S 167

A RHOMBUS is a special kind of parallelogram. It is a diamond-shaped dividing the figure into two triangles by a diagonal, calculating the
four-sided figure with all sides of equal length and opposite sides Ivea of each triangle and adding them together.
parallel to each other. See Fig. 69. The diagonals of a rhombus are ,
perpendicular to each other and their intersection form right angles;
the diagonals bisect each other and each bisects its corner angles.
The area of a rhombus is half of the product of its diagonals.

A diagonal drawn joining two opposite comers of a parallelogram


(Fig. 72) divides the figure into two equal triangles, therefore the area of
a bangle is half that of a parallelogram of the same base and
Fig. 69 perpendl~~lar height. This and other rules for the areas of triangles were
given in the previous chapter, thus,

A TRAPEZIUM is a four-sided figure, two sides only of which are 4


Area of triangle = (base x perp. height)
parallel. Referring to Fig. 70, if a and b are the respective lengths = $ (ab sin C )
of the two parallel sides and h is the perpendicular height, then,
= Js(s - a)(s - b)(s - c)
Area of Trapezium = average length x perp. height
where a, b and c are the lengths of the three sides and s is their semi-
=i(a+b) x h

The equivalent rectangle of the trapezium is shown in dotted lines.


The area of any four-sided figure, such as shown in Fig. 71, wherein
all sides are different length and all angles different, can be found by

PERPENDICULAR
HEIGHT
- - BASE -
Fig. 72

H II b

Fig. 70
- POLYGONS

A Polygon is a figure bounded by more than four straight sides. A


regular polygon has all its sides equal in length and all its angles equal,
REED'S MATHEMATICS FOR ENGINEERS MENSURATION OF A R E A S 169

any regular polygon is therefore made up of as many equal triangles


as there are sides and the method of finding the area of a polygon is
to first find the area of one triangle and then multiply by the number
of triangles which constitute the polygon.
Special names are given to some regular polygons as follows,

A Five-sided figure is called a Pentagon


A Six-sided figure is called a
A Seven-sided figure is called a Heptagon
An Eight-sided figure is called an Octagon
A Nine-sided figure is called a Nonagon Fig. 74
A Ten-sided figure is called a Decagon
~tis, however, also convenient to express the area of the more common
polygons in terms of the length of the 'sides' or 'flats'.
As there are six sides to the hexagon, it is composed of six triangles,
the apex of each triangle is 360" t 6 = 60".
The base angles are each 1(180° - 60") = 60".
Therefore the triangles must be equilateral (see Fig. 74).

The area of an equilateral triangle = 0.433 side2 (previously shown)


:. Area of Hexagon = 6 x 0.433 side2
= 2.598 side2

The OCTAGON being composed of eight isosceles triangles, the apex


angle of each is 360" t 8 = 45" (see Fig. 75).
For any polygon, if n =number of sides, then, 1
The base angles are each (180" - 45") = 67 4"
number of triangles = n

Apex angle of each triangle = a = -


i
Perp. height of triangle = side x tan 67 i0
4
Area of triangle = (base x perp. height)
If r = radius of the circumscribing circle, then r is also the slant height of = $ x b a s e x i x sidex tan67i0
each triangle. Note that all the triangles are isosceles, see Figs. 73 to 75. = i x s i d e x i x side x t a n 6 7 r
Area of each triangle = (ab sin C) = 0.60355 side2
Area of Octagon = 8 x 0.60355 side2
= 4.8284 side2
R E E D ' S MATHEMATICS FOR E N G I N E E R S

umber of small triangles, like a polygon of an infinite number

r = height of each triangle


r x base
Area of each triangle = -
2
Area of circle = sum of areas of all triangles

r
:. Area of circle = - x sum of bases
2
but the sum of the bases constitute the whole circumference= 2nr
THE CIRCLE r
.'. Area of circle = - x 2nr = n?
The Circumference is the outer rim of the circle, an Arc is part of 2
the Circumference, other common terms are illustrated in Fig. 76. d
Also, since r = -
2
n 2
Circumference = n x diameter = nd Area of circle = - d = 0.7854d2
4
= 2n x radius = 2nr
where n = 3.142 to nearest four figures.
ANNULUS OR CIRCULAR RING

Area of Annulus = area of outer circle - area of inner circle

= n~~ - n? (See Fig. 77)


2 2)
= n ( ~-

n
or -(d- d 2 )
4
~ ~d 2 )
or 0 . 7 8 5 4 ( -

Note that the factors of R~ - ? are (R + r)(R- r )


+
and that the factors of d - d 2 are (D d)(D- d )

Use of the factors make calculations of annular areas much quicker.


R E E D ' S MATHEMATICS FOR E N G I N E E R S

Example. Find the effective under-face area of a reciprocating ~ & ~ rif,the angle 8 at the centre be measured in radians,
pump piston 41.5 mm diameter if the piston rod diameter is 8.5 -.
area of circle (= 271 rad) = n?
2
Area of Annulus = 0.7854(d - d )
area of sector of 1 rad = - 1.22
711.2 = -
= 0.7854(D + d)(D - d) 271
+
= 0.7854(41-5 8-5)(41.5- 8.5) 81.2
area of sector of 6 rad = -
2
= 0.7854 x 50 x 33
2
= 1296 mrn Ans. SEGMENTOF A CIRCLE

Refemng to Fig. 79,


SECTOR OF A CIRCLE Area of Segment = area of sector - area of triangle
A Sector of a circle is shown in Fig. 78. As in the case of the whole 61.2
area of sector (shown above) = -
circle we can consider the sector as being made up of a number of 2
small triangles: where 6 is the centre angle in rad.
ab sin C
Area of triangle = -in this case, a = r,
2
b =r, and C is the centre angle 6 in deg.
alternatively, if 8 is the angle at the centre, in degrees, 1.2 sin 8
.'. Area of triangle = -
2
area of circle of 360" = n? Area of segment = area of sector - area of triangle
81.2 1.2 sin 8
area of sector of l o = - -
2 2
v2
= - [6- sin81
2
MENSURATION OF AREAS 175
174 R E E D ' S MATHEMATICS FOR E N G I N E E R S
ELLIPSE
Note carefully that in this apparently simple formula, the first 6' in
the brackets is the angle expressed in radians, whereas the second
8 is in degrees.

Fig. 80

If D = major axis, and d = minor axis of an ellipse (as in Fig. 80),


Fig. 79
Circumference = 71 x average diameter (approx.)

Example. Calculate the area of a segment which subtends an angle


of 150" at the centre of a circle 150 mm diameter.
n
Area = - x D x d
4
150 150 x 271
8 - or
57.3 360 SURFACE OF CYLINDER
= 2.618 rad
Imagine the shell of a cylinder being un-rolled as in Fig. 81 it can be
seen that,
sin 1500 = sin (180" - 150") = sin 30" = 0.5 Curved surface area of a cylinder = nd x h

r=
Area of Segment = - [6' - sin 6'1
2

= 5957 mrn2 Ans. Fig. 81


176 REED'S MATHEMATICS FOR ENGINEERS MENSURATION O F AREAS 177

SURFACE OF SPHERE of gravity (mass), of many sections. With regard to areas the theorem is
stated thus:
The curved surface area of a sphere is equal to the curved surface area
of its circumscribing cylinder, that is, a cylinder of equal diameter and If a line, lying wholly on one side of a fixed axis, be rotated about that
height. axis in its own plane, it will sweep out a surface area equal to the length
The curved surface area of a segment of the sphere or any such of the line multiplied by the distance its centre of gravity moves.
sliced portion, is equal to the curved surface area of the Consider a 'line' such as a straight piece of wire of length I (Fig. 83),
corresponding slice off the circumscribing cylinder. positioned at r from an axis parallel to the wire, if the wire is moved
In all cases, referring to Fig. 82, around through a complete circle about the axis o o as centre, a
surface area like a thin cylinder will be swept out. The centre of
Curved Surface Area = nd x h gravity of the piece of wire is at its mid-length, the distance moved
by the centre of gravity in one revolution is the circumference = 2nr,
For the whole sphere, h = d ,
therefore:
Curved Surface Area of Sphere = nd2
Area swept out = length of line x distance c.g. moves
= 1 X 2nr
= 2nrl

SURFACE OF CONE

Now let the line be inclined with one end touching the fixed axis, as
in Fig. 84. If this line is rotated through one complete revolution the
Fig. 82 area swept out is that of the curved surface area of a cone. The centre
of gravity of the line is at its mid-length, let this be x from o o.

Area swept out = length of line x distance c.g. moves


THEOREM OF PAPPUS OR GULDINUS
=lx2m
This theorem is one of the most useful to employ in finding areas
and volumes of objects of circular shapes, or if the area or volume is
known it can be used to determine the position of the centroid, or centre

Fig. 83 Fig. 84
178 R E E D ' S MATHEMATICS FOR E N G I N E E R S
M E N S U R A T I O N OF A R E A S 179

If the radius of the base of the swept out cone be represented by r, A Frustum of a cone is the part left after a portion of the top of the
it can be seen that x is half of r, therefore: cone has been sliced off. Referring to Fig. 86, the line to be rotated is
r equivalent to the slant height of the frustum, = I. If the radii of the
Curved surface area of cone = I x 2n x - frustum at bottom and top be represented by ,R and r respectively, these
2
are the distances of the bottom and top ends of the line, respectively,
from the axis. The centre of gravity of the line is at its mid-length, the
position of this point from the axis is the mean of R and r which is
I being the slant height of the cone.
$ (R + r), hence,
Area swept out = length of line x distance c.g. moves
Curved surface area of frustum = 1 x 271 x (R + r)4
= nl(R r) +

SURFACE OF A CIRCULAR RING OF CIRCULAR SECTION

Now consider a line as a piece of wire bent around into a circle of


Fig. 85 radius r, the centre of this circle being at R from the axis (Fig. 87), when
this circle is swept through a complete revolution about the axis o o, the
As an alternative to the above, referring to Fig. 85, imagine a paper area swept out is the curved surface area of a ring. A practical example
cone being unrolled, the curved surface area develops into a flat sector of this is the surface area of a ring i.e. that area exposed to the air.
of a circle. The slant height of the cone, I, forms the radius of this
sector, and the circumference of the base of the cone (which is 2nr)
forms the arc of the sector.
The area of a sector of a circle is r/2 x length of arc, therefore if this
is now expressed in terms of the dimensions of the corresponding cone:
Curved surface area of cone = area of sector
1
= - x 2x1. 0
2
= nrl (as before). Fig. 87

Area swept out = length of line x distance c.g. moves


SURFACE OF FRUSTUM OF CONE
Curved surface area of ring = 2nr x 2nR
0
=4n2~r

SURFACE OF A CIRCULAR RING OF ELLIPTICAL SECTION

If the line is an ellipse of major axis D and minor axis d, the centre
being at R from the axis, the area swept out in one revolution is the
curved surface area similar to that of a circular lifebuoy of elliptical
Fig. 86 section (see Fig. 88).
IOU ., , -.... - 3 FOR E N G I N E E R S MENSURATION OF AREAS 181
0 Area swept out = length of line x distance c.g. moves
I

I+- Hence the centre of gravity of this piece of wire lies on a line parallel
to the diameter, at d/n fiom the diameter.
Fig. 88

SIMILAR FIGURES
Areas of similar figures vary as the square of their corresponding
Area swept out = length of line x distance c.g. moves linear dimensions.
(.ld)
Curved surface of lifebuoy = n - .2nR
Similar figures mean that they are of the same shape and proportions,
although their sizes are different.

From the foregoing examples the importance of this Theorem will be


appreciated. Not only can curved surface areas be easily calculated, but
if the area is known in the first place, the centre of gravity of the line
could be determined, this is demonstrated in the example:
To find the centre of gravity of a piece of wire bent into the shape of a
semi-circle of diameter d.
If this semi-circular piece of wire is swept through one revolution
about a fixed axis on its own diameter (Fig. 89) the area swept out will
be the surface area of a sphere, this has already been shown to be nd 2. Fig. 90
Let the centre of gravity be at x fiom the axis, that is, the position of a
knife-edge for the wire to rest upon and be balanced. Consider two circles, one of diameter d, the other twice as big in
diameter = 2d, as in Fig. 90.
Area of small circle = 0.7854d2
Area of large circle = 0.7854 x (2d12
Therefore the ratio of their areas is,
d 2 : (242
= d 2 : 22 X d 2
= 1 :22
Thus, if the diameter of the larger circle is twice the diameter of the
smaller circle, all linear dimensions (i.e., radius, circumference)
will be twice as much, but the area is 2' which is four times the area
Fig. 89 of the small circle.
I
182 R E E D ' S MATHEMATICS FOR E N G I N E E R S M E N S U R A T I O N OF A R E A S 183

If the diameter of one circle is three times that of another, the radius 'To the sum of the first and last ordinates, add four times the even
and circumference of the larger is three times the radius and ordinates and twice the odd ordinates, multiply this sum by one-third
circumference of the smaller, but the area is 32= nine times as much. the common interval and the result is the area of the figure.'
An odd number of ordinates, equally spaced, must be used for this
rule. Step by step, the procedure is as follows, referring to Fig. 92,
1. Divide this given figure into an even number of equally spaced
parts, this gives an odd number of ordinates.
2. Measure the ordinates and the common distance between them.
3. Add together: the first ordinate, the last ordinate, four times the
even ordinates and twice the odd ordinates.
Fig. 91 4. Multiply the above sum by one-third of the common distance
between the ordinates.
Figure 91 shows two similar triangles, the ratio of the base
dimensions is 30 : 45 which is 1 : 1.5, hence the base of the larger
i
triangle is 1 times greater than the base of the smaller.
i
Therefore all linear dimensions of the larger triangle are 1 times
greater than the corresponding linear dimension of the smaller. The ratio
of their areas is proportional to the square of their corresponding
dimensions thus:
Ratio of dimensions = 1 : 1.5
:. Ratio of areas = 1' : 1.5'
= 1 : 2.25

Hence the area of the larger triangle is 2: times the area of the Fig. 92
smaller.
Example. A flat plate is shaped as shown in Fig. 92, the dimensions
Example. A hexagonal plate is cut out of a sheet. Due to an error being in mm, find its area (cm2) by Simpson's rule.
in marking off, the sides were all made 10% longer than intended, Working in cm and setting out in tabulated form:
find the percentage of error in area.
Ratio correct dimensions to wrong = 100 : 110 Simpson's
= 1 : 1.1 Ordinates Multipliers Products
0 1 0
' '
Ratio correct area to wrong = 1 : 1.1 3.54 4 14.16
= 1 : 1.21 6.32 2 12.64
= 100 : 121 8.34 4 33.36
9.6 2 19.20
.'. Area is 21 % too large. Ans.
10.2 4 40.80
IRREGULAR FIGURES 9.96 2 19.92
8.68 4 34.72
SIMPSON'S FIRST RULE. This is a method of finding the area of an 5.8 1 5.80
irregular figure. Briefly it is stated: Sum = 180.60
184 REED'S MATHEMATICS FOR E N G I N E E R S M E N S U R A T I O N OF A R E A S 185

Common interval = length t number of spaces Example. The ordinates measured athwartships across a ship at
= 320 + 8 the load water line are: 0.2,9, 15.5,20, 21.5,20.5, 18.5, 12.5 and 1.3 m
respectively, and the length is 180 m. Find the water plane area.

180-6 x 4
Area = Simpson's
3
Ordinates Multipliers Products
=240.8 cm2 Ans.
0.2 1 0.2
9 4 36
From the explanation of Simpson's Rule, the multipliers of the
15.5 2 31
ordinates are:
20 4 80
For 3 ordinates 1,4, 1 21.5 43
20.5 82
ForSordinates 1,4,2,4,1 18.5 37
For7ordinates 1,4,2,4,2,4,1 12.5 50
For 9 ordinates 1 , 4 , 2 , 4 , 2 , 4 , 2 , 4 1, and so on 1.3 - 1.3
Sum = 360.5
This rule is often expressed in formula fashion thus:

Number of ordinates = 9
. Number of spaces = 8
h being the common interval, Common interval = length t no. of spaces
a, b, c, etc., being the ordinates.
The mean (average) height of an irregular figure can be obtained by = 22.5 m
dividing the area by its length, or can be found direct by Simpson's rule 360.5 x 22.5
by dividing the sum of the products of the ordinates and their mutipliers, Water plane area =
3
by the total of multipliers. = 2704 m2 Ans.
In the foregoing example the two methods of obtaining the mean
height would be: For shapes such as the water plane area of a ship which are
(i) symmetrical about the longitudinal centre-line, measurements from the
centre-line to the hull may be taken and referred to as 'half-ordinates'.
Area = 240.8 cm2
These half-ordinates are put through Simpson's rule, the half-area
Length = 32 cm calculated, then multiplied by two to obtain the fill area.
Mean height = 240.8 t 32 = 7.525 cm
(ii)
MID-ORDINATE RULE
Sum of products = 180.6
+
Sum of multipliers = 1 + 4 + 2 + 4 2 + 4 + 2 + 4 + 1 Another method of finding the area or the mean height of an irregular
figure is by the mid-ordinate rule.
= 24
The method is to divide the figure into any number of equally spaced
Mean height = 180.6 + 24 = 7.525 cm parts, erecting lines midway between these ordinates, these being the
186 R E E D ' S M A THEM A T I CS F O R E N G I N E E R S M E N S U R A T I O N OF A R E A S 187

3. Instead of marking the ten spaces and then the middle of these
spaces to indicate the mid-ordinates, it is quicker to slide the
rule until it registers 5 mm at the first perpendicular and 105 mm
at the other, then mark off every 10 mm point, this gives the
position of the mid-ordinates direct.
4. Erect perpendicular lines through these marks across the diagram,
these are the mid-ordinates.
5. Measure the mid-ordinates, in this case they are 13.5, 9, 7, 6,
4.5, 3.5, 2.5, 2, 1 and 0.5 mm. Add these measurements together,
this gives 49.5 mm. Divide by the number of mid-ordinates,
in this case 10, to obtain the mean height of the diagram, thus,
49.5 t 10 = 4.95 mm.
To obtain the mean effective pressure in the engine cylinder from
which this diagram was taken, the mean height is multiplied by the
pressure scale of the spring used in the indicator. In this diagram,
the spring stiffness was such that one mm of height represents a
pressure of 180 kN/m2. Hence, the mean effective pressure is
4-95 x 180 = 891 kN/m2.

Fig. 93 TEST EXAMPLES 9


1. In a parallelogram ABCD, the opposite parallel sides AD and BC
mid-ordinates; the mid-ordinates are measured, added together and the are each 100 mm long, the other sides are each 60 mm long, and
sum is divided by the number of mid-ordinates to get their average the diagonal AC is 140 mm. Calculate the angles, the short
height. diagonal, the perpendicular height, and the area.
If the area is required, this can be obtained by multiplying the mean
height by the length. 2. The sides of a rhombus are each 32 mm long and the length of the
The simplest method of dividing the diagram into ten parts and long diagonal is 48 mrn. Calculate the angles, the length of the
employing the mid-ordinate rule to determine the mean height is short diagonal, and the area.
explained in the following example.
Referring to Fig. 93 which is a copy of an indicator diagram from 3. In a trapezium ABCD, the two parallel sides are AB and CD, and
the cylinder of an internal combustion engine: their lengths are 100 and 60 mm respectively. Side BC is
perpendicular to the parallel sides and its length is 50 mm.
1. Erect a line at each of the two extreme ends of the diagram,
perpendicular to the atmospheric line (that is the straight (i) Find the area of the trapezium (cm2).
horizontal base line marked AL). (ii) Find the position of a dividing line EF parallel to AB to divide
2. Place a rule so that it measures 100 mm between these the trapezium into two equal half areas.
perpendiculars, if the length of the diagram was exactly 100 mm,
the rule would lie parallel with the atmospheric line, but almost 4. The lengths of the sides of a four-sided figure ABCD, are, in m,
all diagrams are less than this therefore the rule must be inclined AB = 1, BC = 2, CD = 1.5, DA= 3.5, and the angle BCD is
until it registers 100 mm between the lines. 117" 17'. Find the area of the figure.
188 R E E D ' S MATHEMATICS FOR E N G I N E E R S M E N S U R A T I O N OF A R E A S 189
5. The length of the sides of a regular hexagonal plate is 80 mm. 15. An equilateral triangular plate has sides 125 mm long, and
The plate is cut parallel to one of its sides and this reduces the another similarly shaped plate has sides 175 mm long. By what
area by lo%, calculate the thickness of the piece cut off. percentage is the larger plate greater in area than the smaller plate?

6. An octagonal plate, the sides of which are each 30 mm long, 16. Regularly spaced semi-ordinates measured transversely across a
has a circular hole 50 mm diameter cut out of it. Find the net ship at the load-water-line are as follows: 0.1, 3, 5.85, 7.2, 8.1, 8.4,
area of the plate in mm2. 8.4, 8.25, 8.1, 7.5, 6.3, 3.75 and 0.5 m respectively, and the
length is 150 m. Find the area of the water-plane by Simpson's
7. Find the length of the sides and the area of the largest equilateral rule.
triangular plate that can be cut out of a circular plate 120 mm
diameter. 17. (a) Plot the two curves y = x 2 + 3 x + 6 and y=2x2 - x + 1 on
common axes between the limits x = 0 and x = 4.
8. The outer and inner diameters of the collar of a single-collar thrust (b) By Simpson's rule find the area enclosed between the two
shaft are 755 and 415 mm respectively, and the effective area of curves.
contact with the thrust pads is 0.7 of the face of the collar. Calculate
(i) the effective area of contact, in m', and (ii) the total force 18. An internal combustion engine indicator diagram is divided into ten
on the collar, in kN, when the thrust pressure is 2000 kFJ/m2. mid-ordinates and their measurements are: 26, 15, 9.5, 8, 7, 5.5,
4.5, 4, 3 and 0 mm respectively. Find (i) the mean height in mm by
9. Find the area, in cm2, of the smaller segment of a circle of 200 mm the mid-ordinate rule, (ii) the mean effective pressure if 1 mm
diameter if the length of the chord is 180 mm. represents a pressure of 160 k ~ / m ~ .

10. Find the diameter of a solid hemisphere whose total surface area
(including the flat circular base) is 58.9 cm2.

11. The ball of a Brine11 hardness testing machine is 10 mm diameter.


Calculate the depth and curved surface area of an indentation
in a material under test when the surface diameter of the
indentation is 5 mm.

12. It is required to make a hollow cone out of thin flat sheet steel, the
base diameter of the cone to be 150 mm and the perpendicular
height 125 mm. Find the dimensions of the sector to be cut out of
the sheet to make this cone and sketch the pattern.

13. A lampshade has the form of a frustum of a cone, the diameters at


the base and top are 320 and 180 mm respectively and the
perpendicular height is 170 mm. Calculate the curved surface area.

14. A circular ring made of round bar is 640 mm outside diameter and
440 mm inside diameter. Calculate the surface area to be painted.
CHAPTER 10

MENSURATION OF VOLUMES
AND MASSES

VOLUME (V) is the result of the product of three dimensions


measured in similar units. Units m3, mm3 and, often, cm3.

MASS (m) is the quantity of matter possessed by a body and is


proportional to the volume and the density of the body. It is a constant
quantity, that is, the mass can only be changed by adding more matter
or taking matter away. Units g, kg, Mg. The latter is 1 tonne (t).
DENSITY (p) is a measure of the mass per unit volume. The unit of
density is kg/m3. Other units are, g/cm3 for solid materials, g/ml for
liquids, g/l for gases. In some cases t/m3 and kg11 may be used.
m = Vp
The density of pure water may be taken as 1000 kg/m3 which is
equal to 1 t/m3, 1 kg/l, and 1 g/ml.
The total mass of a body is therefore the product of the volume
and the density. Units must be consistent throughout, such as,
mass [kg] = volume [m3] x density [kg/m3]

mass [g] = volume [cm3] x density [g/cm3]


The cm, between mm (small) and m (large), is often used.
RELATIVE DENSITY Or SPECIFIC GRAVITY of a substance is the ratio
of the mass of a volume of the substance to the mass of an equal volume
of pure water. In other words, it is the ratio of the density of the
substance to the density of pure water.

PRISMS
A regular prism is a bar of regular cross-section, some examples are
given in Fig. 94.
192 R E E D ' S MATHEMATICS FOR E N G I N E E R S M E N S U R A T I O N OF V O L U M E S A N D M A S S E S 193

opposite comer. Find (i) the volume of the bar, (ii) the mass in kg
if the density of brass is 8.4 g/cm3.
Side of each equilateral triangle = 45 mm (See Fig. 95)
Perpendicular height = 45 x sin 60"
4
Area of each equilateral triangle = (base x perp. ht.)
4
= x 45 x 45 x sin60°
= 0.433 x 452
Area of hexagon = 6 x 0.433 x 4s2
Volume of prism = area x length
= 6 x 0.433 x 452 x 250
= 1.315 x lo6 mm3
or 1315 cm3 Ans. ... (i)
Fig. 94 Mass (m) = Volume (V) x Density ( p )
= 1315 x 8.4 g
In all these cases, Mass = 1315 x 8.4 x lop3 kg
= 11.05 kg Ans. ... . . . (ii)
Volume = area of cross-section x length
PYRAMIDS
Hence, to find the volume of a prism, calculate the area of the end and
multiply this by length (or height) of the prism. A pyramid is a body standing on a triangular, square or polygonal
base, its sides tapering to a point at the apex, some examples are
Example. A brass bar 250 mm long has a constant hexagonal cross- illustrated in Fig. 96. The cone may be considered as a pyramid with a
section measuring 90 rnrn across the face from one corner to the circular base.

Fig. 95 Fig. 96
194 R E E D ' S MATHEMATICS FOR E N G I N E E R S MENSURATION OF VOLUMES A N D MASSES 195
The volume of a pyramid is one-third of the volume of its
circumscribing prism. Thus, the volume of a cone is one-third of the
volume of a solid cylinder of the same section as the base of the cone

I";
and having the same height; the volume of a square pyramid is
one-third the volume of a bar of square section equal to the base of
the pyramid and of the same height (or length). In all cases: , ------
Volume of pyramid = f (area of base x perpendicular height)

Fig. 98
OBLIQUE PRISMS AND PYRAMIDS
If the prism or pyramid be imagined as being made up of a number of By similar triangles, from Fig. 99:
discs or laminations and pushed over to one side, it can be seen by
reference to Fig. 97 that the same rule for fmding the volumes of regular

,!/p~"j;~
prisms or regular pyramids can be applied provided the perpendicular
height is used.

h=23m
- - ~ = 2 23 + 4 = 6 $m
Volume of whole pyramid = f x 52 x 6 $ m3
Volume of top cut off = $ x 22 x 2 m3
PERPENDICULAR
.'. Volume of frustum = f (52 x 63 - 22 x 2;)
= 52 m3 Ans.

Fig. 97

FRUSTUMS
A h s t u m of a pyramid or cone is the bottom piece left, after a
portion has been sliced off the top (Fig. 98).
The volume can be found by subtracting the volume of the sliced-off
top part from the volume of the complete pyramid.

Example. A frustum of a square pyramid has a height of 4 m and


the lengths of the sides of the square base and top are 5 and 2 m
respectively. Find the volume of the frustum. Fig. 99
196 R E E D ' S MATHEMATICS FOR E N G I N E E R S MENSURATION OF VOLUMES A N D MASSES 197
71
SPHERE Volume of hollow sphere = - (D3 - d 3 )
6
The sphere may be considered as being made up of a great number of
very small pyramids whose bases lie on the surface of the sphere and
71
= - [(d
6
+ 213 - d3]
their apexes all meeting at the centre of the sphere.
Volume of each pyramid
= f (area of base x perpendicular height)

Volume of sphere 17c


Volume of material from cone = - x - x 42 x 12 = 71 x 16 cm3
= sum of volume of all pyramids 3 4
= f x perpendicular height x sum of areas of bases
= f x radius of sphere x curved surface area of sphere

3d 2 + 6d - 44 = 0
Solving this quadratic:

Volume of hollow sphere


d = 2.958 cm or 29.58 mm
D = 4.958 cm or 49.58 mm
} Ans.

= vol. of outer sphere - vol. of inner spherical space

SPHERICAL SEGMENT
A segment of a sphere is shown by ABCDA in Fig. 100. Let its radius
DC = r, thickness BD = h, and diameter of sphere = d. The volume of

Example. A solid lead cone, 40 mm diameter at the base and


120 mm perpendicular height is to be melted down and cast into a
hollow sphere of 10 mm uniform thickness. Find the inside and outside
diameters of the sphere.

Working in cm:

Let d = inside diameter,


then (d + 2) cm = outside diameter. Fig. 100
198 R E E D ' S MATHEMATICS FOR E N G I N E E R S MENSURATION OF VOLUMES A N D MASSES 199

the spherical segment can be obtained by subtracting the volume of the Substituting 3 in terms of d and h, by crossed chords,
cone OADCO from the volume of the spherical sector OABCO.
3 = h(d - h)
Volume of the sector of sphere. Consider this as being made up of a n
great number of very small pyramids whose bases lie on the spherical Volume of cone = - h(d - h)(d - 2h)
6
surface of the sector and their apexes all meeting at the centre of the
sphere.
Volume of segment = Vol. of sector - Vol. of cone
n n
Volume of pyramid = $area of base x perp. height = - d 2 h - - h(d 2 - 3dh + 2h 2 )
6 6
n
Volume of sector = $ x perp. height x sum of areas of bases. +
= - h(d 2 - d 2 3dh - 2h 2 )
6
n
= - h 2 (3d - 2h)
The sum of the areas of the bases is the curved surface area of the 6
slice of the sphere and this has been shown (Chapter 9 ) to be equal to the
curved surface area of the corresponding slice of the circumscribing Example. A segment 2 cm thick is cut off a sphere 10 cm diameter.
cylinder, which is ndh. Find the volumes of the segment cut off, the remaining segment, and
the whole sphere.
Volume of sector = $ x i d x ndh n
Volume of segment cut off = - h 2 (3d - 2h)
n 6
= -d2h
6
I
n
= - x 4 x 26 = 54.46 cm3 Ans. (i)
It will be seen that the volume of the spherical sector and the volume 6
of the whole sphere are in the ratio of their spherical surface areas, thus, Thickness of remaining segment = 10 - 2 = 8 cm
n
Volume = - h 2 (3d - 2h)
6
Volume of sector - Surface area of sector
-
Volume of sphere Surface area of sphere
n
n ndh = - x 64 x 14 = 469.2 cm3 Ans. (ii)
Volume of sector = - d x -2
6
6 nd Volume of sphere = sum of vols. of the two segments
n
=-d2h as above = 54.46 + 469.2
6
= 523.66 cm3 Ans. (iii)
Volume of cone = ;area of base x perp. height
As a check:

d = - x 103
' 1 3
Volume of sphere = -
6 6
= 523.66 cm3 as above
200 R E E D ' S MATHEMATICS FOR E N G I N E E R S M E N S U R A T I O N OF V O L U M E S A N D M A S S E S 20 1

Example. A sphere is sliced into three pieces by two parallel cuts. THEOREM OF PAPPUS OR GULDINUS APPLIED
The top segment is 8 cm thick and 24 cm diameter at its base, the TO VOLUMES
bottom segment is 5 cm thick. Calculate the volume of the zone of If an area, situated wholly on one side of a fixed axis, be rotated in its
sphere between the segments. own plane about this axis, it will sweep out a volume equal to the
product of the area and the distance its centroid moves.
In the majority of cases areas are being swept around one complete
revolution and the resultant volumes are referred to as 'solids of
revolution'.
For example, consider a flat circular disc of radius r, its centre being
at R from the axis o o as in Fig. 102, if this area is swept around the axis
through one complete revolution, it will sweep out a solid ring of
circular section, the mean radius of the ring being R, and the radius of
the cross-section of the material being r.

Fig. 101

Let d = diameter of sphere, by crossed chords (Fig. 101):

Fig. 102
71
Volume of sphere = - d 3 A practical example of this is the volume of a ring, thus:
6
Volume swept out = area x distance its centroid moves
2
71 Volume of ring = n? x 271R
Volume of spherical segment = -
6 h (3d - 2h)
71
=2n2~?
6 x g2(3 x 26 - 2 x 8 )
Volume of top segment = -
Now consider an elliptical area of major diameter D,minor diameter
71
= - x 64 x 62 = 2078 cm3 d, swept around through one revolution, R being the radius from the
6 axis o o to the centre of the ellipse (Fig. 103).
71
Volume of bottom segment = -
6 x 5'(3 x 26 - 2 x 5 )

Volume of zone = volume of sphere - top and bottom


segments
= 9204 - 2078 - 890
3
= 6236 cm Ans. Fig. 103
202 R E E D ' S MATHEMATICS FOR E N G I N E E R S MENSURATION OF VOLUMES AND MASSES 203
2
The volume swept out is the shape of a circular lifebuoy of elliptical 1 m/s , then the force (N) to give m kg of mass an acceleration of
cross-section. 9.81 m/s2 is m x 9.81. Hence, at the earth's surface, the gravitational
force on a mass of m kg is mg newtons, therefore:
Volume swept out = area x distance its centroid moves
71
Volume of lifebuoy = - Dd x 271R Weight [N] = mass [kg] x g [m/s2]
4
W=mg
712
= -DdR
2 The CENTRE OF GRAVITY of a body is that point through which the
It is best to work from first principles making use of this theorem whole weight of the mass can be considered as acting (Centre of Mass).
rather than applying formulae. For instance, imagine a body to be compressed in volume into one tiny
particle without losing mass, the position of this small heavy particle
Example. The inside diameter of a solid circular cork lifebuoy is would be at the centre of gravity of the body to have the same effect. If
500 rnrn and the section is elliptical 160 mm major diameter by 120 mm the body was suspended from this point, or supported on it, it would
minor diameter. Find (i) its volume in m3, (ii) its mass if the density of balance perfectly without tilting.
cork is 240 kg/m3. When dealing with an area instead of a solid, an area theoretically
has no mass, therefore it is not strictly correct to use the term centre of
Referring to Fig. 103, working in m: gravity, in such cases the term centroid can be used.

PARALLELOGRAM

A parallelogram would balance if laid on a knife edge along either


Volume swept out = area x distance its centroid moves one of its two diagonals, therefore its centre of gravity (for a plate)
= 0.7854 x D x d x 2zR or centroid (for a plane area) is at the intersection of its diagonals as
= 0.7854 x 0.16 x 0.12 x 271 x 0.33 shown in Fig. 104.
=0.03127m3 Ans. (i)
Mass (m) = volume (V) x density ( p )
= 0.03127 x 240
= 7.504 kg Ans. (ii)

FORCE, WEIGHT AND CENTRE OF GRAVITY


Fig. 104
FORCEis that which produces or tends to produce motion in a body.
The unit of force is the newton (N) and may be defined as the force
required to give unit acceleration (a gain of velocity of 1 m/s every
TRAPEZIUM
second the force is applied) to unit mass (one kg).
The WEIGHT p)of a body is the gravitational force on the mass of The centroid of a trapezium is at the intersection of EF and HG as
that body, that is, the force of attraction exerted on the body by the earth. shown in Fig. 105, found graphically as follows:
If a body is allowed to fall freely, it will fall with an acceleration of
Join EF (the mid points)
9.81 m/s2, this is termed gravitational acceleration and is represented
by g. Since 1 N of force will give 1 kg of mass an acceleration of Produce AD to G, length DG being equal to BC
M E N S U R A T I O N OF V O L U M E S A N D M A S S E S 205
204 R E E D ' S MATHEMATICS FOR E N G I N E E R S

Fig. 105

Fig. 107
Produce BC to H, length BH being equal to AD
Join HG
SEMI-CIRCULAR AREA
The intersection of EF and HG is the centroid.
The centroid of a semi-circular area is at 0.424r from its diameter,
as shown in Fig. 108. This can easily be shown by the Theorem of
Pappus:
TRIANGLE If the semi-circular area is swept around through one
The centroid of a triangle lies on a line joining a comer of the triangle revolution about an axis on its own diameter, the volume swept out
with the mid-point of its opposite side, at a point at one-third of the will be that of a sphere. Refemng to Fig. 108, let the centroid be at x
height from that side. See Fig. 106. from the diameter.

Fig. 108

Volume swept out = area x distance its centroid moves


Fig. 106

PYRAMID

The centre of gravity of a pyramid or cone is at one-quarter of the


height above the base. See Fig. 107.
206 R E E D ' S MATHEMATICS FOR E N G I N E E R S MENSURATION OF VOLUMES A N D MASSES 207

HEMISPHERE volume of a cone. If the radius of the base of the cone is r, the centroid
of the triangle is at 3 r from o o.
The centre of gravity of a hemisphere is at three-eighths of the radius
Volume swept out = area of triangle x distance its centroid moves
above the diameter. See Fig. 109.
5
Volume of cone = (base x perp. ht.) x (2n x f r)

4
= n?h (as shown previously)

- d -----+
Fig. 109
VOLUME OF FRUSTUM OF A CONE

To derive a formula for the volume of a frustum of a cone, let


r = radius at top, R =radius at bottom, and h =perpendicular height.
Consider a trapezium made up of a rectangle and a triangle as shown
in Fig. 11 1. If this area is rotated one revolution about the axis o o, a
frustum of a cone will be swept out.

VOLUME OF CONE

Now that the position of the centroid of a triangle is known, it can


be shown that the volume of a cone can be determined by the
Theorem of Pappus.
If a triangular area is swept about an axis o o which coincides with
its side as shown in Fig. 110, it will, in one revolution, sweep out the

0
Fig. 111
Area of rectangle = rh
Centroid from o o = $ r
Area of triangle = $(base x perp. height)
=$(R -r)h
+
Centroid from o o = r ;(R - r)
=r+iR-fr
=$R+;r
Volume swept out by rectangle = rh x 271 x r
Fig. 110
208 R E E D ' S MATHEMATICS FOR E N G I N E E R S
MENSURATION OF VOLUMES A N D MASSES 209

4
Volume swept out by triangle = (R - r)h x 2n(: R + i r) Thus, the ratio of their volumes is:
+
= nh(R - r)($R i r )
+ $Rr - $?)
= nh($R2 ... (ii)
Total volume = nr2h+ nh(i R2 + i Rr - $12)
=nh(3 +fR2 +iRr-i?)
= nh(:R2 +$Rr +f?) Hence, the diameter of the larger sphere being twice the diameter
of the smaller, all linear dimensions such as the radius and
- l3 n h ( + ~ +6 )
~ Rr circumference are twice as much, all areas such as the sectional area
and curved surface area are 22= 4 times as much (as explained in last
or, since R = D, and r = i d , chapter), the volume is Z3 = 8 times the volume of the smaller, and
Volume = nh(D2 + Dd + d2) consequently, the mass of the larger is 8 times the mass of the smaller
if they are made of the same kind of material.

SIMILAR SOLIDS
Volumes of similar objects vary as the cube of their corresponding
linear dimensions.
Similar objects mean that they are of the same shape and proportions,
but of different size.
Consider two solid spheres, one of diameter d, the other having a
diameter of 2d, as in Fig. 112,
n
Volume of small sphere = - d3
6
n
Volume of large sphere = - x (2d)3 Fig. 113
6

Figure 113 shows two similar cones, they are similar because they
have the same proportions, for instance, in each case the perpendicular
height is 1.25 times the diameter of the base. The dimensions of the
larger cone is 1.5 times the corresponding dimensions of the smaller,
thus:

Ratio of corresponding dimensions = 1 : 1.5


Ratio of corresponding areas = l 2 : 1.52
= 1 : 2.25
Ratio of corresponding volumes = l 3 : 1 . 5 ~
= 1 : 3.375
Fig. 112
210 REED'S MATHEMATICS FOR ENGINEERS MENSURATION OF VOLUMES A N D MASSES 211
Hence, all linear dimensions (such as diameter of base, circumference Example. A barrel is 960 mm long; the diameter at each end is
of base, perpendicular height, slant height) of the larger cone are 1.5 600 mm, at quarter and three-quarter lengths the diameter is 720 mm,
times those of the smaller cone; all areas (such as area of base, and at mid-length the diameter is 800 mm. Find the capacity of the
curved surface area, sectional area) of the larger cone are 1.52= 2.25 barrel in m3 and also in 1.
times the corresponding areas of the smaller; and the volume of the Working in m (see Fig. 114):
larger cone is 1 . 5=
~ 3.375 times the volume of the smaller.
Sectional Areas Multipliers Products
SIMPSON'S RULE APPLIED TO VOLUMES 0.7854 x 0 . 6 ~ 1 0.7854 x 0.36
The procedure of finding the volume of an irregular object by 0.7854 x 0 . 7 2 ~ 4 0.7854 x 2.0736
Simpson's rule is the same as for the area of an irregular figure, it is 0.7854 x 0 . 8 ~ 2 0.7854 x 1.28
merely a matter of substituting cross-sectional areas for ordinates. Thus: 0.7854 x 0 . 7 2 ~ 4 0.7854 x 2-0736
To the first and last cross-sectional areas, add four times the even 0.7854 x 0 . 6 ~ 1 0.7854 x 0.36
cross-sectional areas and twice the odd; multiply this sum by one-third
the common interval and the result is the volume of the object.

Example. A casting of light alloy, 750 mm long, has a variable Common interval = 0.96 t 4 = 0.24 m
cross-sectional area throughout its length. At regular distances of Volume = 0.7854 x 6.1472 x f x 0.24
125 mm apart, starting from one end, the sectional areas are, 12.2, 17.5, = 0.3863 m
3
Ans. (i)
23.2, 27.9, 21.0, 11.2 and 0 cm2 respectively. Find the volume and its
= 386.3 1 Ans. (ii)
mass if the density of the material is 3.2 g/cm3.

Sectional Simpson's Products


Note that the common multiplier of 0.7854 is carried through to
Areas (cm2) Multipliers
the final sum of the products.
12.2 1
This could be expressed in formula fashion thus:
17.5 4
23.2 2 h
27.9 4 Volume = - x 0.7854(d;
3
+ 4d: + 2d32+ etc.)
21.0 2
11.2 4
0 1

Common interval = 12.5 cm


Volume = sum of products x common interval
= 327 x f x 12.5
= 1362.5cm3 Ans. (i)
Mass (m) = volume (V) x density ( p )
= 1362.5 x 3.2
= 4 3 6 0 g = 4 . 3 6 k g Ans. (ii) Fig. 114
212 R E E D ' S MATHEMATICS FOR E N G I N E E R S MENSURATION OF VOLUMES A N D MASSES 213
where dl, d2, d3, etc., are the diameters, and h is the common interval Cross-sectional Simpson's
between measurements, or Radii r Areas = n? Multipliers Products
3 n x9 1 nx 9
h
Volume = - x n(r12+ 4rZ2 2r:
3
+ + etc.) 2 n x4 4 n x 16
1.5 n: x 2.25 2 z x 4.5
where r,, r2, r3, etc., are the radii. 1.2 z x 1.44 4 n x 5.76
1 nx1 1 nx 1
Example. Plot the graph xy = 6 between the limits x = 2 and x = 6. Sum = n x 36.26
If the area under this graph is rotated about its x-axis through one
complete revolution, calculate, by Simpson's rule, the volume swept out.
Common interval between ordinates = 1
6
xy = 6 .'. y = - Volume = n x 36.26 x x1
X
3
Plotting points for values of x between the limits x = 2 and x = 6, = 37.98 units Ans.

x= 2 3 4 5 6

FLOW OF LIQUID THROUGH PIPES, ETC.


VOLUME FLOW (V) is the volume of a fluid flowing past a given point
in unit time (m3/s).
The velocity or speed of flow is the 'length' of liquid which passes in
a given time. For instance, if the velocity of the liquid is 2 m/s it means
that a column of the liquid 2 m long passes every s, hence, volume flow
is the product of the cross-sectional area of the flowing liquid and its
velocity. In basic units:

Volume flow [m3/s] = cross-sect. area [m2] x velocity [m/s]


V=AV

MASS FLOW (m) is the mass of fluid flowing past a given point in unit
time.
Fig. 115a Fig. 115b Since density is the mass per unit volume, then the mass flow is the
product of the volume flow and the density.

The graph is shown in Fig. 115a. When the area under this graph is Mass flow [kg/s] = volume flow [m3/s] x density [kg/m3]
rotated about its x-axis through one revolution, the volume swept out m = Vp
appears as shown in Fig. 115b. The y-ordinates of the graph become the
radii of the solid at regular intervals along its length. Putting the
cross-sectional areas at these regular intervals through Simpson's rule Example. Oil of density 0.85 g/ml flows full bore through a pipe
as in the previous example: 50 mm diameter at a velocity of 1.5 m/s. Find the quantity flowing,
214 REED'S MATHEMATICS FOR ENGINEERS MENSURATION O F VOLUMES AND MASSES 215

(i) in m3/h, (ii) kg/h, (iii) t/h. A practical example is a valve over a valve seat, neglecting the area
taken up by the wings of the valve, the maximum effective lift is
Velocity = 1.5 m/s = 1.5 x 3600 m/h one-quarter of the valve diameter. If the lift is more than this, no
Volume flow [m3/h] = area [m2] x velocity [m/h] more liquid can flow through than that which is allowed by the area
of the bore of the seat, but if the lift is less than one-quarter of the
v = 0.7854 x 0 . 0 5 ~x 1.5 x 3600 diameter, the circumferential area of escape is less than the area of
= 10.6 m 3 /h Ans. (i) the seat bore, and the quantity of liquid flowing through depends
Density = 0.85 g/ml = 0.85 x lo3 kg/m3 upon the area, circumference x lift.
Mass flow [kg/h] = volume flow [m3/h] x density [kg/m3] Example. Calculate the quantity of water flowing, in l/min,
m = 10.6 x 0.85 x lo3 through a valve 100 mm diameter when the lift is 15 mrn and the
= 9012 kg/h Ans. (ii) velocity of the water is 3 m/s, assuming that the wings of the valve
take up one-sixth of the circumference.
= 9.012 t/h Ans. (iii)
One sixth of the area is obstructed by the wings, this leaves five-
sixths of the area for the water to flow through. Working in m:
FLOW THROUGH VALVES. When a liquid flows out of the open end Circumferential area of escape between valve and seat
of a pipe, the maximum quantity of liquid escaping depends upon the
area of the bore of the pipe end. The flow can be restricted by a
cover over the pipe end so that the area of escape is less than the area
of the pipe bore.
Referring to Fig. 116, the area of escape is the circumferential
opening, circumference x lift. The maximum effective lift will be lm3 = 1o3 l hence
when this circumferential area of escape is equal to the area of the
bore, thus,

circumference x lift = area of bore = 706.9 l/min Ans.

CENTRES OF GRAVITY BY FIRST MOMENTS


The moment of a force about a given point is the product of the force
and the perpendicular distance of its line of action from that point.

Fig. 117

Figure 117 illustrates a force of 300 N acting on the end of a lever, at


a perpendicular distance of 2 m from o, its moment therefore is
Fig. 116 300 x 2 = 600 Nm. This is the effect of the force which tends to turn
216 REED'S MATHEMATICS FOR ENGINEERS MENSURATION OF VOLUMES AND MASSES 217

Moments about o o:
Clockwise moments = Anticlockwise moments
Wl x x l + W, x x 2 + W3 X X =
~ F x ~

Since the total upward force F must be equal to the total downward
+ +
force, then F= Wl W2 W3
:. w1 x X I + w, x x, + w3 x x3 = (W, + W2 f W,) x x
- W,x,+W2x2+W3x3
x=
w1+ w2+ w3
In words this is:
PLAN
- Summation of all moments of weights
X =
Summation of all weights
- C moments of weight
X =
C weights
0 As previously explained weight is equal to mass x g, and since g is
Fig. 118 constant, then weight can be represented by mass:
- C moments of masses
x= . . . . . . . . . (ii)
the lever in a clockwise direction around the axis 0. A force of C masses
400 N acting at 1.5 m, or 600 N at 1 m leverage, would have the Mass is obtained from, volume x density, so that if the material is
same turning effect. the same kind throughout, the same value for the density of the
Consider a piece of plate cut to shape shown in Fig. 118. Imagine material will be in every term of the above expression and will cancel
this plate supported horizontally on one single support, call this out, hence:
support the fulcrum. The fulcrum (F)must be positioned exactly at the
centre of gravity of the plate if the plate is to be perfectly balanced
-C moments of volumes
x= . . . . . . . . . (iii)
because 'the centre of gravity is that position through which the whole C volumes
weight can be considered as acting'. Further, volume =area x thickness, therefore if the thickness of
Now take moments about the end o o, this means to imagine the plate the material is uniform throughout as well as its density, then the
to be temporarily hinged at this en4 for perfect equilibrium the moments thickness will also cancel from every term,
of all the forces tending to turn the plate clockwise around the hinge
- C moments of areas
must be equal to the moments of the forces tending to turn the plate x= ... . . . . . . (iv)
anticlockwise about the hinge.
C areas
Hence, for convenience take moments of masses, moments of
Let W,, W2 and W3 represent the weights of the top, centre and volumes and moments of areas. The above expressions provide methods
bottom parts. of finding the position of the centre of gravity of objects and figures
Let xl, x2 and x3 represent the distances of the centres of gravity of made up from regular shapes.
these parts from oo. Moments as used above are usually referred to as 'first moments' to
Let f represent the position of the fulcrum (F)from o o, and as distinguish them from 'second moments' which are used in applied
previously stated this is the centre of gravity of the whole plate. mechanics.
218 R E E D ' S MATHEMATICS FOR E N G I N E E R S MENSURATION OF VOLUMES A N D MASSES 219

Fig. 119
Fig. 120
Moments about base o o, working in cm:
Example. To find the position of the centre of gravity of the plate
illustrated in Fig. 119 which is of uniform thickness throughout, the
y = C moments of areas
C areas
dimensions shown being all in mm. 0.7854 x 92 x 4.5 - 0.7854 x 32 x 2
Working in cm: y'
0.7854 x 92 - 0.7854 x 32
32 x 4.5 - 2 --38.5
Area of top flange = 11 x 2 = 22 cm2 y= -
32 - 1 8
Area of centre web = 10 x 1.5 = 15 cm2 = 4.8125 cm from bottom, = 48.125 mm
Area of bottom flange = 16 x 3 = 48 cm2 or, 48.125 - 45
Distance of c.g. of top flange from base = 14 cm =3.125mmfromdisccentre. Ans.
Distance of c.g. of centre web from base = 8 cm (Alternatively, moments can be taken about centre of disc.)
Distance of c.g. of bottom flange from base = 1.5 cm Note, in this example area is lost by boring the hole, therefore the
summation of areas is the net area obtained by subtracting the area of the
Taking moments about base, hole from the area of the disc; also, the summation of moments
C moments of areas of areas is the difference between the moments of areas of the disc
y= and hole.
C areas
In each of the above two cases, it is obvious that the centre of gravity
lies on the vertical centre line because the figures are symmetrical,
therefore it is sufficient to calculate the position of the centre of gravity
in one direction only. For figures that are not symmetrical, it is
=5.88cmor58.8mmabovethebase. Ans. necessary to express the position of the centre of gravity in two
directions at right angles to each other, say from the base and from
one side, this is done by taking moments about these two datum lines
Example. A hole 30 mm diameter is bored through a solid disc
separately.
90 mm diameter, the centre of the hole being 25 mm from the centre of
the disc. (Fig. 120) Find the position of the centre of gravity of the disc IRREGULAR FIGURES. Simpson's rule can be employed to find the
after the hole has been cut out. moment of an irregular area in a similar manner to which it is applied in
finding the area.
220 R E E D ' S MATHEMATICS FOR E N G I N E E R S MENSURATION OF VOLUMES A N D MASSES 22 1
To express the moment about a given point, the perpendicular distance Common interval = 4.8 + 4 = 1.2 cm
of each ordinate is measured from that point, then: Area=48 x f x 1.2 = 19.2cm2 .. . (i)
Add together, the moment of the first ordinate, the moment of the last Moment of area about base = 76.8 x f x 1.2 = 30.72 cm3
ordinate, four times the moments of the even ordinates, and twice the moment of area
moments of the odd ordinates; multiply this sum by one-third of the Centroid from base =
area
common interval. 30.72
The centroid can then be found by dividing the moment of the area - - 1.6 cmor 16mm (ii)
19.2
by the area.
As an example, take a right angled triangle of 80 mm base and It will be seen that the position of the centroid can be obtained by
48 rnm height as illustrated in Fig. 121, to fmd the area and position dividing the sum of column v by the sum of column iii thus,
of centroid, from base o o by Simpson's rule. By taking a regular
shape such as this it enables us to compare the results so obtained
with those calculated from formulae.

Further, if this triangle is swept through one complete revolution


about its base o o it will sweep out the volume of a cone of dimensions
48 mm radius of base and 80 mm perpendicular height.
By Theorem of Pappus:

Volume swept out = area x distance its centroid moves


= 19.2 x 271 x 1.6
= 193.1 cm3 . . . . . . . . . (iii)
Fig. 121
The above agrees with results by formulae:

Measurements from base, in cm. Area = $(base x perpendicular height)

i 11
...
111 iv v
4
= x 8 x 4.8 = 19.2 cm2
Centroid = f of perpendicular height
Distances of
= f x 4.8 = 1.6cm
Ordinates Simpson's Products ordinates Products
multipliers of i and ii from base of iii and iv Volume of cone = f x area of base x perpendicular height
=fx 71 ~ 4 . x8 8 ~= 193.1 cm3
8 1 8 0 0
6 4 24 1.2 28.8
4 2 8 2.4 19.2
2 4 8 . 3.6 28.8 SHIFT OF CENTRE OF GRAVITY DUE TO
SHIFT OF LOADS
0 1 0 4.8 0
Consider a system composed of loads which weigh w,,w2 and w 3 as
Sum = 48 Sum = 76.8 shown in Fig. 122, the centre of gravity of each being h l , h2 and h3
222 R E E D ' S MATHEMATICS FOR E N G I N E E R S MENSURATION OF VOLUMES A N D MASSES 223

respectively from the base o o. Let yl be the distance of the centre of As shown previously, weight can be represented by mass, therefore
gravity of the whole system from the base, then: write:
mass shifted x distance it is moved
X moments of weights Shift of c.g. =
j1 = total mass
Z weights

TEST EXAMPLES 10

1. An I-section steel girder of 150 mm overall depth has unequal


flanges, the top flange is 100 mm wide by 12 mm thick and the
bottom flange is 140 mm wide by 14 mm thick. The centre web is
10 mm thick. Considering the flanges as rectangular in section by
neglecting radii and fillets, calculate the mass in kg/m run if the
density of the material is 7.86 g/cm3.

2. A hollow steel shaft, 400 mm outside diameter and 200 mm


inside diameter, has a coupling 75 mm thick and 760 mm
diameter at each end, and the overall length is 6 m. Neglecting
fillets and coupling bolt holes, find the mass of the shaft in
t taking the density of steel as 7.86 x lo3 kg/m3.
Fig. 122
3. A cylinder and sphere and base of a cone are all the same diameter,
and the heights of the cylinder and cone are each equal to the
If w3 be lifted into the position shown, through a height of d the new
diameter of the sphere. Find the ratio of the volumes of the cylinder
centre of gravity of the whole system from the base, represented by j2 is:
and sphere relative to the volume of the cone.

4. A piece of flat steel plate, having a mass of 6.5 kg/m2, is cut to


the shape of a sector of a circle of radius 180 mm and subtended
angle at the centre 240°, and the sector is rolled into a cone. Find (i)
the mass of material used, (ii) the diameter of the base of the cone,
The shift of the centre of gravity of the whole system in the direction
(iii) the perpendicular height of the cone, (iv) the capacity of the
measured from the base is y2 - jl which is:
cone in 1.

5. An object is constructed by brazing the base of a solid cone to the


In words this is: flat surface of a solid hemisphere, the diameter of the base of the
cone and the diameter of the hemisphere both being 60 mm, and the
weight shifted x distance it is moved perpendicular height of the cone 50 mm. Find the mass of the
Shift of c.g. =
total weight object if the density of the materials is 8.4 g/cm3.
224 R E E D ' S MATHEMATICS FOR E N G I N E E R S M E N S U R A T I O N OF V O L U M E S A N D M A S S E S 225
6. A hollow lead sphere has a uniform thickness of 10 mm and its +
14. Plot the graph y = 5 4x - 2 between the limits x = -1 and
mass is 3 kg. Taking the specific gravity of lead as 11.4, find its +
x = 5. If the area under this curve is rotated about its x-axis, find
outside diameter. the volume swept out.

7. A hole 24 mm diameter is bored centrally through a sphere 5 1 mm 15. A water-trough has a regular isosceles triangular section, the
diameter. Calculate the volume of the drilled sphere in cm3 and angle at the bottom being 80". Calculate the volume flow of water
its mass if the density of the material is 7.86 x lo3 kg/m3. along the trough, in m3/h, when the depth of the water in the
trough is 180 mm and it is flowing at a velocity of 0.5 m/s.
8. A tapered hole is bored through a right circular cone, concentric
with the axis of the cone. The base diameter of the cone is 64 mm 16. Find the height of the centre of gravity of a frustum of a cone which
and the perpendicular height is 60 mm. The diameter of the hole at
is 80 mm diameter at the base, 60 mm diameter at the top, and
the base of the cone is 28 mm and the diameter where it breaks
40 mm perpendicular height.
through the surface of the cone is 16 mm. Calculate the volume and
mass of the remaining hollow frustum, taking the density of the
material as 8.4 g/cm3. 17. Find the position of the centre of gravity of the beam knee plate
illustrated in Fig. 123 giving the distances from the 375 and
9. The lengths of the sides of the base of a regular hexagonal 300 mm straight sides as represented by i and j respectively.
pyramid is 25 rnrn and the perpendicular height is 60 mm. Find
the volume in cm3. If this pyramid is cut through a plane parallel
to its base at half the height, find the volume of the remaining
frustum.

10. A vessel in the form of a hollow cone with vertex downwards, is


partially filled with water. The volume of the water is 200 cm3 and
the depth of the water is 50 mm. Find the volume of water which
must be added to increase the depth to 70 mm.

1 1 . The diameter of the base of a hollow cone is 300 mm and its


perpendicular height is 500 mm. It is partly filled with water so that
when resting on its base the depth of the water is 250 mm. If the Fig. 123
cone is inverted and balanced on its apex, what will then be the
depth of the water? 18. A bolt has a round head and plain round shank, the head is 60 mm
diameter and 40 mm deep, the shank is 40 mm diameter and the
12. The surface area of a solid sphere is 1 times the surface area overall length of the bolt is 270 mm. A ring of the same material as
of a smaller sphere, and the difference in their volumes is 10 cm3. the bolt, and of the same dimensions as the bolt head, is made to
Find the volume and diameter of the smaller sphere. slide on the shank to a position of 13 1 mm from the shank end to
the mid-point of the ring. Find the position of the centre of gravity
13. The diameters of a barrel are 395 mm at each end, 477 mm at of the whole from end of shank.
quarter and three-quarter lengths, and 500 mm at mid-length, and
the total length is 581 mm. Using Simpson's rule calculate the
capacity of the barrel in 1.
CHAPTER 1 I

DIFFERENTIAL CALCULUS

The development and application of the differential calculus may be


explained by the relation between equations and their graphs.
The meaning of the gradient at any point on such a graph is of
particular importance in this context and it is therefore dealt with first.

GRADIENT OF A LINE
It will be recalled, from previous work on graphs, that the gradient
change in y
(or slope) of a line is the between any two points on the
change in x
line.

Fig. 124

For the line shown in Fig. 124,


change in y
gradient = between points A and B
change in x

The following examples are given by way of revision, to show how


the gradient of a line is calculated:
228 R E E D ' S MATHEMATICS FOR E N G I N E E R S DIFFERENTIAL CALCULUS 229
Y
Example. Plot the graph of the equation y = 3x between the limits I
x = 0 and x = 4 and find the gradient of the graph.

change
in y

I
V
change In x

Fig. 126

change In x (Note that the value of y changes from 2 to zero, i.e. the change in y is
Fig. 125 -2.)
-L
.'. Gradient =-
8
Referring to Fig. 125:

change in y
Gradient =
change in x
Two important points are noted from the previous examples:

=3 Ans.

Note: The change in y is three times the change in x.


Thus the gradient indicates the rate of change of one quantity, with
respect to another, related, quantity.

Example. Plot the graph of the equation y = 2 - 0 . 2 5 between


~
the limits x = 0 and x = 8. Find the gradient of this graph.
Fig. 127 Fig. 128
The graph is shown in, Fig. 126

change in y (i) If y increases as x increases (Fig. 127), the gradient is positive.


Gradient =
change in x (ii) If y decreases as x increases (Fig. 128), the gradient is negative.
230 R E E D ' S MATHEMATICS FOR E N G I N E E R S DIFFERENTIAL CALCULUS 23 1
--

ZERO GRADIENT DIFFERENTIATION FROM FIRST PRINCIPLES

The gradient of a horizontal straight line is zero, since the change in y is From Fig. 130 it was seen that the gradient of the tangent at P was
zero for any given change in the value of x (Fig. 129). required, in order to determine the gradient of the curve at P
Chord PA is now constructed, as shown (Fig. 131).

k
gradient = 0

Fig. 129

1;' GRADIENT OF A CURVE


Fig. 131

The gradient of a curve is not constant, but is changing from point to 6 is used, for convenience, to express mathematically the change
point along the length of the curve. in the value of y and x between points P and A.
The gradient of a curve at any given point is the gradient of the Thus, 6x, pronounced 'delta ex', means a small change in the
tangent to the curve at that point. value of x. It does not mean 6 x x.
Similarly, 6y means the corresponding change in the value of y.
From Fig. 131,

SY
gradient of chord PA = -
6x

The gradient of this chord is obviously quite different to the gradient


of the tangent to the curve.
However, if point A is now allowed to move along the curve towards
point P, it is seen (Figs. 132a and 132b), that the slope of the chord more
and more resembles the slope of the tangent as A gets closer to P
In fact, when point A gets very close to point P, the slope of the
chord and the tangent are virtually the same.
Referring to Fig. 130: As A gets very close to P, so length 6x is becoming very small,
until it finally approaches zero value.
Gradient of the curve at P = gradient of the tangent at P i.e. 6x approaches zero value (denoted by 6x + 0).
From the above reasoning, it follows that, when 6x has become
The gradient of a curve may be obtained by drawing the tangent and infinitely small, the ratio 6y/6x has reached a special value, or limit,
measuring the gradient, or mathematically, by the dzfferential calculus. where it represents the gradient of the curve at point I?
232 R E E D ' S MATHEMATICS FOR E N G I N E E R S DIFFERENTIAL CALCULUS 233

Fig. 132a Fig. 132b

In order to indicate that this is a special value of 6y/6x, it is identified Fig. 133
by changing the notation from 6y/6x to dyldx. This relationship is
expressed mathematically as:
Divide equation (iii) by ax, thus obtaining gradient,
SY= -
limit - dy
SXHO 6x dx

The term d y l h is called the differential coefficient of y with respect


to x and the process of finding dyldx is called differentiation. .. (iv)

Example. From fist principles, derive an expression for the If point A now moves very close to point P, so length 6x will approach
gradient of the curve y =? at any point on the curve. zero value.
The sketch of the graph of y =2is shown in Fig. 133. Point P
represents any point on the curve. i.e. SY= 2x
limit -
Chord PA is drawn as shown. 6x+O 6x
1. The co-ordinates for point P are (x, y).
+
2. Hence, the co-ordinates for point A are (x 6x), 0.,+ 6y).
3. Now, the equation of this curve is y = x2
Hence, at any point on the curve y =xZ
:. At point, p , y = 2 . . . . . . . . . . . . (i)
the gradient of the curve = 2x Ans.
and, at point A, (y + 6y) = (x + 6x1~
:. y + 6y = 2 + 2x 6x + ( 6 ~ ) .~. . (ii) Note: Since 6x becomes very close to zero value, it can be omitted
from the right-hand side of equation (iv) as being negligible compared to
Subtract equation (i) from equation (ii), the term 2x. This does not happen on the left side of equation (iv),
because, as 6x becomes very small, so too does Gy, hence the ratio 6y/6x
remains a significant quantity.

Example. Calculate the gradient of the curve y =x 2 at the points


6y = +
2 x 6 ~ (6x1~ ... (iii) x = - 3 and x = +2.
234 R E E D ' S MATHEMATICS FOR E N G I N E E R S DIFFERENTIAL CALCULUS 235
From the previous example, (The differential coefficient or gradient may be calculated without
actually drawing a graph, although a sketch of the graph is often useful
d~ when solving problems.)
gradient at any point on the curve = -= 2x.
dx
:. At the point x = -3
gradient = 2 x (-3)
= - 6 Ans.(i)
At the point x = 2,
gradient = 2 x (2)
= 4 Ans. (ii)
Subtract equation (i) from equation (ii),
These results can be confirmed by drawing tangents to the curve
y = x 2 a t x = - 3 andx=2. y + 6y = 3x3+ 9 2 6x + 9x(6x)' + 3(6x13
The gradient of each of these tangents, when measured, should be y = 3x3
reasonably close to the calculated values. (The tangents can be drawn on
Fig. 134.) 6y = 9 2 6x + 9~(6x)'+ 3(6x13
Divide by 6x,

SY= 9 2
limit -
Sx+O 6x

.'. For the equation y = 3 2 ,


differential coefficient = 9 2 A ~ S .

GENERAL RULE FOR DIFFERENTIATION


From the previous section, the following results were obtained:

Fig. 134
when =2 dy=2x
dx
Example. From first principles, find the differential coefficient of y,
with respect to x, for the equation y = 3x3.
and, when y = 3x3, 9
dx
=9 2
DIFFERENTIAL CALCULUS 237
236 R EED ' S M A THE M A TI CS F O R E N G I N E E R S

From the form of these results (which is confirmed by solving further


examples), the following general rule is noted:
when y = ax"
. dy - -x-2
" dx-
-
1
- -- Ans. (e)
e.g. when y = 9 x 4 . . . (i.e. a = 9 , n = 4 ) x2

x3
y= -
12
dy,36x3
- A~S. .dy 3 x 1
" dx -
" dx 12
--
x'
Example. Differentiate the following equations with respect to x: - Ans. (f)
4
(a) Y 'X (b) y = 2x-3 (c) y = -7xP4
1 x3 DIFFERENTIAL COEFFICIENT OF A CONSTANT
(d) y = x (e) y = -
X
(f) Y = 12
As shown previously, when y is a constant, the resulting graph has
zero gradient (see Fig. 129).
i.e. if y =a (where a is any constant)

6
- -- Ans. (b)
- Y'
DIFFERENTIAL COEFFICIENT OF A SUM OF TERMS
The differential coefficient of a sum of terms is obtained by
differentiating each term separately.

28
=- Ans. (c)
x5

Example. Differentiate the equation

=I Ans. (d) with respect to x.


238 R E E D ' S MATHEMATICS F O R E N G I N E E R S DIFFERENTIAL C A L C U L U S 239

Rearranging the equation, Using symbol v for velocity, s for distance and t for time, this equation
x3 may be expressed in calculus notation:
y = - - 3 ~ +- 3x~ 6s
3 average velocity = -
6t
9 = 2 + 6xp3+ 3
dx If the velocity at a given instant (the instantaneous velocity) is required,
6 the time interval 6t must be very small, i.e. 6t must approach zero
=2+-+3 Ans. value (Fig. 135).
x3
s
Example. Differentiate the equation s = 3t2 - 20t 40 with + I
respect to t. (Note: Symbols s and t represent two related quantities, as
do x and y.)
s = 3? -20t+40

SECOND DIFFERENTIAL COEFFICIENT Fig. 135


In some cases, having differentiated an expression once, it is Thus,
necessary to differentiate a second time. 6s ds
limit - = -
The notation d2y/dX2 is used to denote the second differentiation of 6t+0 6t dt
an expression. Hence, at any given time t, the instantaneous velocity of a moving
body may be expressed as:
e.g. Consider the equation y = x4
ds
v=-
differentiating once. 2
dx
= 4x 3
dt
Similarly,
differentiating twice, !f? = 12.2 change of velocity
d2 average acceleration =
time to change
By the same reasoning as that above, the instantaneous acceleration a
Example. Determine the second differential coefficient of the
is given by the expression:
following equation:
y=?-42+3x-7

Example. The distance s moved by a body in time t is given by


&= 6x- 8 Ans. the formula s = t 3 - 3t 2 . Find expressions for the velocity and
dx2 acceleration of the body at any instant.

DISTANCE, VELOCITY AND ACCELERATION


For a moving body,

average velocity =
distance travelled
time taken
ds
dt = 3t 2 - 6t
velocity, v = -
dv
acceleration a = -
dt = 6t
1 Ans.
DIFFERENTIAL CALCULUS 24 1
240 R E E D ' S MATHEMATICS F O R E N G I N E E R S
Calculate (a) the velocity after 2 s; (b) the t~metaken for the body
Note: The expression for the acceleration of the body is seen to be
to come to rest.
the second differential equation of the original equation relating
distance s to time t.

when t = 2
v = 40 - 20
= 20 m/s. Ans. (a)
(b) When the body comes to rest, v = 0
.'. 4 0 - lot = 0
Example. A body moves s metres in t seconds according to the
+
relationship s = t3 - 79 3.
(a) Derive expressions for the velocity and acceleration of the
body at any instant.
(b) Use these expressions to find the velocity and acceleration
of the body after 5 s. MAXIMA AND MINIMA
An important application of differential calculus involves functions
which have maximum or minimum values.
The graph shown below (Fig. 136) represents a function which
has a local maximum value of y at point A and a local minimum value
of y at B.
Such points are called turning points and have zero gradient.
:. At any instant
d~ = 0 .
i.e. at a turning point, -
velocity = 3 8 - 14t m/s dr
2
acceleration = 6t - 14 m/s

t = 5,
v = 3 x (5)2 - 14 x (5)
= 5 m / s Ans.(b)
a = 6 x (5)- 14
= 16m/s 2 Ans. (b)

Example. The distance (displacement) s moved by a body in time t


is given by the expression
242 REED'S MATHEMATICS FOR ENGINEERS DIFFERENTIAL CALCULUS 243

Example. The curve y = 3x - 2 + 1 has a turning point. Inspection of Fig. 136 shows that the gradient changes sign when
(a) Find the value of x at which this occurs. passing through a turning point.
(b) Determine whether the turning point is a maximum or minimum This presents an alternative test for maximum or minimum values:
value. (i) At a maximum value, the gradient dy/dx is positive before the
(c) Find the value of y at the turning point. turning point and negative after it.
Hence, dy/& is decreasing in value, as x increases, i.e. the rate
of change of dyldx is negative.

d 2y .
for turning points, .'. 1s negative.
3-2x=O (Note: since dyldx is a gradient, the rate of change of the
2x=3 gradient is d2y/dX2).
:. x = 1.5 Ans. (a) (ii) At a minimum value, the gradient is negative before the turning
(b) The nature of the turning point can be determined by sketching point and positive after it.
the curve: Hence, dyldx is increasing in value and the rate of change of
From the sketch (Fig. 137), at x = 1.5, a maximum value dy/dx is positive.
occurs. Ans. (b).
d 2y .
. 1s positive.

Note: If a curve has a point of inflexion the gradient does


not change sign when passing through this point, i.e. it is
stationaly, then both dyldx and d2y/dX2equal zero.
Summing up:
At a maximum value,

d2
is negative.
dX 2
At a minimum value.

Fig. 137 dr- 2 is positive.


d-
dX2

(c) To find the maximum value ofy, substitute x = 1.5 in the original
equation: Example.
(a) Determine the values of x which give maximum or minimum
values of y in the equation

:. Maximum value of y = 3.25. Ans. (c)


244 R E E D ' S MATHEMATICS FOR E N G I N E E R S DIFFERENTIAL CALCULUS 245
(b) Calculate these maximum or minimum values of y. Y

For maximum or minimum values,

Fig. 138
Prove that, for this box to have maximum volume, its depth should be
6 cm.

Maximum or minimum values of y occur when x = 1 or


.:
x=3. Ans. (a).
I
1 11
(b) When x = 1, Fig. 139a Fig. 139b
Referring to Fig. 139a
3
dx2
= 6 - 12 (i.e. negative)
let x = side of square removed
When x = 3, With the comer portions removed, the box is formed, with the
dimensions shown (Fig. 139b)
d2y
-= 18 - 12 (positive)
dx2
let V = volume of box
:. When x = 1, then, V = x(28 - 2x)(60 - 2x)
y = (113 - 6(112 + 9(1) v = 4x3 - 176x2 + 1 6 8 0 ~
.'. Maximum value of y = 4 Ans. (b)
When x = 3
for maximum (or minimum) value of
y = (313 - 6(312 + 9(3)
:. Minimum value of y = 0 Ans. (b) +
12.x?- 3 5 2 ~ 1680 = 0
solve for x : 3x2 -88x+420 = O
Note: These results are confirmed by the sketch of the graph shown I .'. ( 3 -~ 7 0 ) ( ~- 6) = 0
in Fig. 138.
hence, 3x-70=O or, x-6=0
Example. A rectangular sheet of steel is 60 cm wide and 28 cm x = 23.33 or, x =6
long. Four square portions are to be removed at the comers and the sides
turned up to form an open rectangular box. .'. x = 6 cm (rejecting the inadmissable solution)
I
DIFFERENTIAL CALCULUS 247
246 REED'S MATHEMATICS F O R ENGINEERS
For example, when x = 180" (i.e. TC rad):
Note: in a problem of this type, it is evident that x = 6 cm must give
a maximum volume, and not a minimum. -I
The result can of course, be confirmed by differentiating again: from the sinx curve, gradient = -
1
dy = -1
.1.e. -
dx
and, from the cos x curve, cos x = - 1
When x = 6 i.e. differential coefficient of sinx = cosx

d'Y
dx2
= 144 - 352 Similarly, the gradient at any point on the cosx curve is numerically
equal to the corresponding value of sinx. In this case, however, the
= -208 (negative) sign must be reversed, because the cosx curve has a negative
gradient where sin x has positive values.
:. Maximum volume is obtained when depth is 6 cm. Ans.
Example. Differentiate the following expressions with respect to x:

DIFFERENTIATION OF SIN x, COS x (a) y=-sinx


(b) y = 2 sinx
For any angle x,
(c) y=4sinx-3cosx
dy
If y=sinx, - = cosx
dx (a) y=-sinx
dy = - sinx
If y = cosx, -
dx 3 = - cos x ~ o s(a).
dx
(b) y = 2 sinx
These differential coefficients can be demonstrated by reference to
the sketch graphs of sinx and cosx (Fig. 140). " = 2 x cosx
dx
= 2 cos x
Ans. (b)
(c) y=4sinx-3cosx
dy = 4 cos x
- + 3 sinx ~ n s (c)
.
dx

The second differential coefficients of sinx and cosx can be


determined quite easily by repeated differentiation:

!
"
dx
y = sinx

= cosx

d2y-
2
- sin x
Fig. 140 dx -
248 R E E D ' S MATHEMATICS FOR E N G I N E E R S DIFFERENTIAL CALCULUS 249

Similarly, y = cosx Example. Obtain the second differential coefficient of the equations
y = l n x and y = 6 .
dy - - sinx
-
dx- y = lnx
2 - 1-
dx- x
= x-I
Example. Obtain the second differential coefficients of (a) 4 cosx,
(b) cos x - 2 sin x.
-
1
-- Ans.
(a) y = 4 cos x x2

dx2
fi
= -4 cosx Ans. (a)

(b) y=cosx-2sinx

FUNCTIONAL NOTATION
+
In an equation such as y = 2x or y = 5.2 3x, the value of y
Note: It is seen that, for sines and cosines, the second differential obviously depends upon the value chosen for x. Hence, y is said to be a
coefficient is equal to the original function, but of the opposite sign. function of x and the general expression for such a relationship is:

Functional notation may also be used to indicate the differentiation


DIFFERENTIAL COEFFICIENT OF lnx AND e"
process, using the symbols shown below:
Naperian logarithms (In) have special applications, for example,
when finding the work done by an expanding gas.

When y = lnx
dy - 1
z-x
When For example if y = 5x3 + 2x
then f (x) = 5x3+ 2x
f (x) = 1 5 2 + 2
t

The proof of those relationships are beyond the scope of this work. fU(x)= 30x.
250 REED'S MATHEMATICS FOR ENGINEERS DIFFERENTIAL CALCULUS 25 1

- ! $ TEST EXAMPLES 11 8. Determine the maximum value of y in the equation


111 "
y = 12x+32-2x3.
1. Differentiate the following equations with respect to x:
- $ a
mc, ' (a) y = 2 + 3 2 - 9 ~ + 4
1.4 , 9. For a beam of length I, the bending moment M at any distance x
It
2x3 7 from one end is given by the equation
::;,1 (b) Y = ~ - - + X
.m x2
11111111
nm2? . , (c) ~ = ~ , @ + 1 M = - wlx
-- w2
2 2
"V>. . ,

(d) y = 5cosx-7cosx+2sinx
,
.*'*
111
.
* where w is the uniform load per unit length. Show that bending
Calculate the gradient of the curve 2 + 3x - 7 at the points moment is a maximum at the centre of the beam.
where x = 3 and x = -2.
10. Differentiate the following equations:
The displacement s metres of a body from a fixed point is given aun - 1
by the equation s =20t - 5t2+ 4 where t is the time in seconds. (b) z = -
C
Find:
(a) The velocity after 2 s (c) y = 3x(g - 4) (d) y = 2 c o s 8 + 5
(b) The displacement when the velocity is zero
(c) The acceleration. 11. Determine the minimum value of f(x) for the equation
2
UI 4. Determine the gradient of a tangent to any point in the curve f(~)=~-2$+3x+1

X
12. Power (P) and Voltage (V) of a lamp are related by P = a v b , where
Show that there are two points where the gradient is zero. a and b are constants. Find an expression for (a) the rate of
change of power with voltage and (b) power per volt at 100 volts,
5. Find the co-ordinates of the point on the graph of y = 3 1 - x + 2 when a = 0.5 x lo-" and b = 6.
at which the gradient is equal to -7.
13. A line of length I is to be cut up into four parts and put together
6. The angle 8 rad through which a shaft has turned after time t as a rectangle. Show that the area of the rectangle will be a
seconds is given by the equation maximum if each of its sides is equal to one quarter of I (i.e. a
square).
t2
8=2+16t--.
2 14. Determine the second differential coefficient of:
Find the angular velocity after 2 s and the time for the shaft to (a) f (8) = cos 8 - In 8
come to rest. (Note: angular velocity w = dO/dt). (b) f(t) = at 2 +21nt
(c) f ( 4 = 5 8
7. (a) Determine the second differential coefficient of the expression
+
y = 2 + 3, lnx with respect to x. 15. A body moves so that its displacement (distance) x metres, which it
(b) Determine the second differential coefficient of the expression +
travels from a certain point 0, is given by: x = 0.2t2 10.4 where t
+
y = 3 cos 8 - 7 cos 8 8 with respect to 8. is the time in seconds. Find the velocity and acceleration (a) 5 s
252 R E E D ' S MATHEMATICS FOR E N G I N E E R S

after the body begins to move and (b) when the displacement is
100 m.
CHAPTER 12
16. Angular displacement (8 rad) from rest of a revolving wheel is
given by: 8 = 2.1 - 3.2t + 4.8t2 where t is the time in seconds. INTEGRAL CALCULUS
Find the angular velocity and angular acceleration after 1.5 s.

17. Verify that the equation f (x) = 2 - 5x has a maximum and a Integration may be considered as reversing the process of
minimum value and determine the value of x and f(x) at these differentiation.
points. That is, given the differential coefficient of a fknction, we are
required to find the original
- function.
+ + +
18. The gradient function of y = a? bx c is 4x 2. The function
has a minimum value of 1. Find the values of a , b and c .
The symbol I
J
is used to denote the integration process. This is the
old-fashioned letter 'S' and the reason for its use becomes evident when
integration is used to find the 'sum' of a number of quantities.

CONSTANT OF INTEGRATION
Consider the three equations below:

For each of these equations,

Obviously, when reversing the differentiation process (i.e. integrating),


provision must be made for the possibility of a constant in the
original equation.
This is achieved by adding a constant, C, called the constant of
integration.

which may be interpreted as:


(the integral of) 2x (with respect to x) =x 2 + C.
254 R E E D ' S MATHEMATICS FOR E N G I N E E R S INTEGRAL CALCULUS
255
8
GENERAL RULE FOR INTEGRATION x
(4 /x7dx = % + C Ans. (a)
Consider the following differential equations:

I x6
=- +
x6
3x5u!x=3x-+c
6
C Ans. (b)
2

From these equations, the following integrals are obtained:

(dl 8 dx = 8x +C Ans. (d)

This last result may be confinned by differentiating the answer, or by


applying the general rule, as shown:

I 8 dx may be written as

Applying the general rule,


/(8 x xO)A, since xo = 1.

It is seen that in each case, the procedure is to raise the power of x x1


J(8 x x 0 ) u ! x = 8 x - + C
by 1 and divide by this raised power. 1
Expressing this mathematically, a general rule for integration is =8x+C
obtained, which includes a constant of integration:

INTEGRATION OF A SUM OF TERMS


The integral of a sum of terms is equal to the sum of their separate
integrals.
Note: there is an important exception to this rule, the integral of
x-' or llx, which is dealt with later.

Example. Evaluate the following integrals:

(a) /x7dx (b) / 3x5dx


x-~
(c) Jjdx (d) j8dx
Note: the constants which should be added to each separate term are
combined as a single constant.
256 R E E D ' S MATHEMATICS FOR E N G I N E E R S INTEGRAL CALCULUS 257
Example. Evaluate the integral Example. The curve of a graph has a gradient of lox - x2. If
the curve passes through the point (3, 52), find the equation of the
graph.

x3
Example. Integrate the expression 3t2 + t + 1 with respect to t. .: y = 5 2 - - + c
3
Substituting x = 3, y = 52,

Example. Integrate the expression 3x2 - 6x + with respect to x.

2
The equation is y = 5 2 - - + 16 Ans.
3
EVALUATING THE CONSTANT OF INTEGRATION
The value of the constant of integration, for a given function, can be INTEGRATION OF SIN x, COS x, llx, 8
calculated, provided a corresponding pair of values of x and y are The following differential coefficients are known:
known.
y = sinx, dy-
- - cosx
+
Example. The gradient of a curve is 4x 5. If the curve passes dx
through the point (x = 0, y = -4), find the equation of the curve.
dy
Gradient of curve = - = 4x + 5 y = lnx, dy-
- --1
dx dx x

Reversing the process, the following integrals are obtained:

S cosx dx = sinx + C

To find the constant of integration C, substitute x = 0, y =-4, in


equation (i): .I sinxdx = -cosx+ C

= lnx+c

+
The equation of the curve is y = 2x2 5x - 4. Ans.
258 REED'S MATHEMATICS FOR ENGINEERS INTEGRALCALCULUS 259

8 3

I* u ,
WLl ",'
.,

," #I /
-1

me( ."
11
Example. Evaluate the integral
I (2 cosx - sin x)dx

, LV511,
I IC, "I
I(2 cosx - sinx) dx = 2 sinx - (- cosx) +C
('"Cn **
.*me, a 11111 =2sinx+cosx+C Ans.
~ l U q . ~'

i..-.
with respect to 8.
Example. Evaluate the integral
I (cos 8 + 5 sin 8)

Fig. 141

s
Example. Evaluate the integral (5/x) dx.
Integrating both sides of this expression with respect to x,

=5lnx+C Ans.
.'. A = / y dx, between the limits

x=a and x = b
AREA BY INTEGRATION, DEFINITE INTEGRAL
In Fig. 141, the curve ac represents the sketch of a function of x. This general rule is expressed mathematically as:
The area abc may be divided into a large number of strips, or elements,
one of which is shown.

Let A = area abc


6A = area of one strip Example. Find the area between the curve y =3,
the x axis and the
ordinates x = 2 and x = 4 (Fig. 142).
If 6x is made very small, the effect of the curve at the top of the
strip becomes negligible. The strip may then be considered as a
rectangle of height y and width 6x. Area =
s: y dx
260 R E E D ' S MATHEMATICS F O R E N G I N E E R S INTEGRAL CALCULUS 261

Fig. 143

Fig. 142

x3
whenx=4, - + ~ = 2 1 f + C
3
x3 Integrate:
whenx=2, - + ~ = 2 5 + C
3
Subtract:

(The integral is inserted in square brackets, with upper and lower limits
Area=[21f + C ] - [ 2 i + C ] of x placed as shown above.)
=21f+c-23-c
=18$ units2. Ans.

Note: The constant of integration C always disappears when integrating


between limits.
Because of this, when an integral has limits, it is called a dejinite
Solve: Area =
["I3
+ (53+ 5
I E3 I
- -+ (212 + 2

integral.

Example. Find the area enclosed by the c w e y = 2 + 2x + 1, the


x-axis, and the ordinates x = 5, x = 2. = 63 units2. Ans.
The sketch graph of the equation is shown (Fig. 143).

Note: When an area is to be calculated, a sketch of the graph should


Area =
1: y dx always be made.
Other problems involving definite integrals may not require a sketch.
In either case, the use of upper and lower limits eliminates the constant
of integration.
I 262
REED'S MATHEMATICS FOR ENGINEERS
1
INTEGRAL CALCULUS 263

INTEGRATION AS A SUMMATION
+ 1) dx
" B ,

,-
vr:

,4nw
",
'"me

BW
-,...
":
Example. Evaluate the definite integral
I-, (x4 - 2 2
In the previous section, it was shown that the area under a curve
could be determined by dividing the area into very narrow strips.
* 4u.' . ' *
:, :;
"* *
I. .
"k

I 111
(~~-g+i)dw=

= [A] [-A]
-
I
Fig. 144

--
l 6 Ans.
15 I One such strip is shown in Fig. 144. The width of the strip may be
represented by symbol dx if its width approaches zero value.
The arrangement of these solutions should be carefully noted. Area of one strip = y x dx
The lower value of x must be placed at the bottom of the integral sign I
and the bracket. The total area abcd could be obtained by adding together all these small
The upper value of x must be at the top of the integral sign and areas, such as the one shown.
bracket.
I i.e. Area abcd = the summation of all the areas such as ydx
Example. Evaluate the definite integral
I," (sinx) dx
Note: For this integral, upper limit = n rad (i.e. 180°),
between the limits x = a and x = b
rb

lower limit = 0 rad (i.e. 0").


Thus, the symbol S may be interpreted as meaning the summation
of the quantities to which it is applied.
I
= [- cos n] - [- cos 01
= [-(-I)] - [- 11
VOLUME OF A SOLID OF REVOLUTION
=If1
If a curve is rotated about the x-axis as shown, (Figs. 145a and 145b),
=2 Ans. the shape generated is called a solid of revolution.
264 R E E D ' S MATHEMATICS FOR E N G I N E E R S INTEGRALCALCULUS 265
The area under the curve is divided into a large number of elemental Y
strips, such as the one shown (Fig. 145a). 1

Fig. 146

Fig. 145a Fig. 145b Referring to Fig. 146:

By rotating this element about the x-axis, a solid disc is generated


V=z
1: Jdx

I (Fig. 145b).
where y = ~ 3 , a = 0 , b=2.

Volume of one disc = cross-section area x thickness


= .Jdr
Total volume = the summation of the volumes of all such
discs, between x = a and x = b.
b
i.e. volume = J,
ZJ A

Since the constant factor 71 is not affected by the integration process, the
equation may be written:

.'. Volume = 57.45 units3. Ans.


Volume of a solid of revolution = 71
S:Jdr Example. Use integral calculus to prove that the volume of a sphere
is $nr3where r is the radius of the sphere.

Example. Calculate the volume generated by rotating the c w e Note: For convenience, the volume of a hemisphere is initially
y =x3 about the x-axis between x = 0 and x = 2. calculated.
266 R E E D ' S MATHEMATICS FOR E N G I N E E R S INTEGRAL CALCULUS 267

From Figs 147a and 147b: DISTANCE AND VELOCITY BY INTEGRATION


In the previous chapter, it was shown that the relation between
distance s, time t and velocity v could be expressed as:

Integrating both sides of this equation with respect to t, the following


equation is obtained:

Likewise, the relation between acceleration a, velocity and time was


defined as:

Fig. 147a Fig. 147b


Integrating both sides of this equation gives the following expression:
If the quadrant ABC is rotated about the x-axis, a hemisphere is
generated. (Lower and upper limits of curve AB are x = 0, x = r.)

For any point on the curve AB:


V =
J adt ... . . . ... (ii)

by Pythagoras theorem: 3= 2-2


Example. The velocity v of a body after time t is given by the
.: Volume of hemisphere = n 11J dx
expression v = 10 - 2t. If displacement (distance) s = 24 when t =2,
obtain an expression for distance s in terms of t .

= n 0r ( -~$ ) a
s=
J vdt

.: s = 1 0 t - ? + c ... . . . ... (9
Substituting s = 24, t = 2, in equation (i),
24=20-4+C
.'. C = 8 . . . . . . . . . . . . (ii)
2
:. Volume of sphere = 2 x -R?
3
From equations (i) and (ii),
4
= -nr 3 Ans.
3 s = 10t- t2 + 8 Ans.
268 REED'S MATHEMATICS FOR ENGINEERS INTEGRAL CALCULUS 269
. W,."",k,
Example. The velocity v m/s after t seconds for a body is given TEST EXAMPLES 12
,. -.""
u r e:":, by v = 4 + 71. Find the distance travelled by the body in the interval
.am,...
::.
,I1 11
from t=O to t=5 s. 1. Evaluate the following integrals:
bffi?.

5
s= vdt (note: definite integral)
0

0
= [107.5]- [0]
.. Distance = 107.5 m. Ans.
2. A curve has a gradient of 2 + x - 2. If the curve passes through the
point x = 2, y = 5, find its equation.

Example. A body moves such that its acceleration a m/s2 after time 3. Evaluate the integrals:
t seconds is given by a = 18 - 2t.
(a) Derive an expression for the velocity v m/s of the body, given (a) I(3 cos x - 2 sinx + 4) dr
that v = 20 m/s when t = 0.
(b) Use the expression to find the velocity of the body after 3 s.

... v = 1 8 t - 8 + ~ ... ... .. . 6)


4. Find the area between the curve y = x3 - 4 2 + 3x and the x-axis
Substitute v = 20, t = 0 in equation (i), between the limits x = 1 and x = 3.
20=0-0+C
:. C = 20 ... .. . ... . . . (ii) 5. The c w e shown overleaf (Fig. 148) was plotted during the
isothermal expansion of a gas, following the law pV= C, between
volume VI and V2. Show that the area under the
From equations (i) and (ii), graph = p V In V2/ V,.

6. The acceleration a m/s2 of a body is given by the equation a =6t.


(a) Use calculus to obtain an expression for velocity v afier t
seconds given that v = 0 when t = 0.
(b) Calculate the average velocity during the period t = 2 to t = 3.
:. Velocity = 65 m/s. Ans. (b) I
(c) Calculate the instantaneous velocity at t = 2.5.
270 R E E D ' S MATHEMATICS F O R E N G I N E E R S INTEGRAL CALCULUS 271
pressure-volume diagram, between the stated volume limits, is
given by:

13. Evaluate the following integrals:

(b) 1; sinx dx

I I
I
I (c) j:/2(5 cosx + 3 sin* - x) dx
-v
0 Vl V2

Fig. 148 14. Find the volume of the ellipsoid formed by rotating the ellipse
+
x2/a2 g / b 2 = 1, of major axis 2a and minor axis 2b, about the
7. The velocity v m/s of a body after time t seconds is given by major axis.
+
the equation v = 3t2 8t + 12. Find the displacement of s
metres for the body after 10 s, given that s = 10 when t = 0. 15. A particle is projected with a horizontal velocity (u) and moves
into a resisting medium so that at time (t) its acceleration (dvldt)
equals a constant (k) multiplied by its velocity (v). Show that:
8. A cone is generated by rotating the line y =x/2 about the x-axis
from x = 0 to x =6. Calculate the volume of the cone. v
In- = -kt
U

9. Find the volume generated by rotating about the x-axis that part
of the curve y =x2 - x which lies between its intersections with 16. Sketch the curve and find an expression for the area between the
the x-axis. +
limits x = 0 and x = 1, the curve y = b/x a and the x axis.
Evaluate this area when a = 1 and b = 2.
10. Evaluate the following integrals: 17. Evaluate the following integrals:

(c) /:(a+26)(x+l)h (d) l 3 e ' d r (c) J(42 + 3 2 + 2.z + 1)dz (d) S(2 cos 0 - 5 sin @)dB

18. A particle P starts from rest at 0 with a velocity of 5 m/s and moves
11. Evaluate the area enclosed by the curve f (x) = 0.069 + 10, along a straight line OX with an acceleration of - 2t2 at time t
the x axis and the ordinates x = 6 and x = 8. seconds after leaving 0. Describe the motion after 3 s.
I

12. A gas is compressed from a volume of V 1 to a volume of V2


according to the law pV" = C, where p is the pressure and C is a
constant. Show that the area under the compression curve on a
SELECTION OF RULES
AND FORMULAE

FROM THE CHAPTERS OF THIS BOOK

LAWS OF INDICES

P x x " =P+"

written In
(xm)"= Pxn

P I " = "@
l m = 1 for all values of rn
1
X =x
x0 = 1 for all values of x except when x = 0

I LOGARITHMS

log, N = n means x" = N


log (x x y) = logx + logy
log (x +y) = logx - logy
log ( P ) = (logx) x rn
log "6= (logx) + rn
log,
log, N = - --
N
log, b
274 R E E D ' S MATHEMATICS FOR E N G I N E E R S S E L E C T I O N OF R U L E S A N D F O R M U L A E 275

ALGEBRA
sin 0
tane=-
cos 6
sin2 8 + cos2 0 = 1
cosec 2b' - cot2 0 = 1
sec2 0 - tan2 6 = 1
sin (A + B) = sin A cos B + cos A sin B
sin (A - B) = sinAcosB - cosAsinB
cos (A + B) = cos A cos B - sin A sin B
cos (A - B) = cosAcosB+sinAsinB

Linear equation: y=a + bx tan (A+B) =


tanA + tanB
1 -tanAtanB
Quadratic equation: y = ruL? + bx + c tanA - tanB
tan (A - B) = 1 +tanAtanB
Roots of quadratic equation of the form:

cos 2A = cos2 A - sin2 A


are:

2tanA
tan 2A =
TRIGONOMETRY AND GEOMETRY 1 - tan2 A
a b c
opposite SINE RULE :
sin 8 = sin A - sin B - sin C
hypotenuse
COSINE RULE :
adjacent
cos 8 =
hypotenuse
opposite
tang=- CROSSED CHORDS : ao x bo = co x do
adjacent
I
1 hypotenuse CYCLIC TRIANGLES:
cosec 8 = ---- =
sin 8 opposite
Angles in the same segment of a circle are equal.
1 hypotenuse
sec8 = -- I The angle in a semi-circle is a right angle.
cos 8 - adjacent I
The angle at the centre of a circle is double the angle at the
1 adjacent circumference for triangles in the same segment and on the same
=- =-
tan 8 opposite chord.
276 R EED ' S M A T H E M A T I C S F O R E N G I N E E R S SELECTION OF RULES A N D FORMULAE 277

CIRCULAR MEASURE: by one-third the common interval between ordinates. In symbols:

Circumference of circle = nd = 2nr

Circumference of ellipse = n -- (" approx: SURFACE AREAS

One circle = 360" = 2n rad.


C w e d surface of cylinder = ndh
One rad = 57.3"
Length of arc = Or Curved surface of sphere = nd2
Linear velocity = o r Curved surface of cone = nrl
Curved surface of frustum = xl(R + r)

PLANE AREAS
VOLUMES
n
fuea of circle = n? = - d 2
4
n 2
Volume of prism = area of end x length.
Area of annulus = n(R2 - 2)= - 4 (9- d ) Volume of cone and pyramid = f (area of base x perp. height)
7L
Area of ellipse = nRr = -
4
Dd
n 4
Volume of sphere = - d 3 = - nr3
Area of parallelogram = base x perp. height 6 3
Area of rhombus = $ product of diagonals n h2(3d - 2h)
Voh.une of spherical segment = -
5
Area of trapezium = sum of parallel sides 6
x perp. distance between Volume of frustum of cone = nh(& + ~d + d2)
e 13
Area of sector = - x 7
O

=f 7 ~ h +( Rr
~+ ~ 9)
360
Area of segment = ?(8 - sin 0)
fuea of triangle = $(base x perp. height)
= (ahsin C) CENTROIDS AND CENTRES OF GRAVITY

= Js(s - a)(s - b)(s - c)


Triangle = f height from base
Area of equilateral triangle = 0.433 side2
Area of hexagon = six equilateral triangles Pyramid = height from base
Semi-circular area = 0.424 r from diameter
Area of irregular figure by Simpson's rule: To the first and last ordinates
add four times the even ordinates and twice the odd, multiply this sum Hemisphere = 0.375 r from diameter
278 R E E D ' S MATHEMATICS F O R E N G I N E E R S

CALCULUS

SOLUTIONS TO TEST EXAMPLES 1

- lo@
10' = 10 Ans.
105; -

cos x sin x
- sinx cos x sinx+C
1
-x-2 - (i.e. x-') lnx + C
X
-
8-2
8-5
2
(Note result of subtracting -5 from -2 is + 3)
dy = 0
Maximum - -2 is negative
dx dx

dy = 0
Minimum - d2y is positive
-
dx a!2

Volume = 7t
C 3 dx
ds
-=v
dt
and s = v dt
J
dv
- = a and v = a dt
dt I

= 17.14 Ans.
280 R E E D ' S MATHEMATICS FOR E N G I N E E R S S O L U T I O N S TO T E S T E X A M P L E S 1 28 1
! 1
I 9. Strength cx breadth x depth2 x -
length
Strength x length
= constant
breadth x depth2
- SlX11 - S2XI2
S, = S2 and cancels
b, x d 1 2 -b2 ~d~~

= 2 4 ~ 3 2 = 1 6 x 9 = 1 4 4 Ans. 40 x 10 000 x 3
b2 =
5 x 6400
#" m.nIlll
IY..~ '
= 37.5 mm Ans.
i,,*""." " 4
7. Let x represent volume of second sphere
iafn: " III"

1st vol. : 2nd vol. : : 1st dia.3 : 2nd dia.3 10. Elongation = 62.5 - 50
24.25 : x :: l 3 : 23 = 12.5 mm
24.25 : x :: 1 :8 elongation
% elongation = x 100
original length
x x 1 = 24.25 x 8
x = 194cm 3 Ans.
Reduction in area = 80 - 48
= 32 m2
8. Quantity pumped by 1st pump in 1 h = of tank
a
Quantity pumped by 2nd pump in 1 h = of tank % reduction in area =
reduction
original area
x 100
Quantity pumped by 3rd pump in 1 h = of tank 32
- - x 100 = 40 Ans. (ii)
-
80
11. Ratio of powers,
When all three pumps are working together,
No. 1 No. 2 No. 3
1 1 1
Quantity pumped per hour = - -
12 4
+ + -
9
=I15 :95 : 100
=1.15 :0.95 :1
+ +
1.15 0.95 1 = 3.1
% of total power developed in No. 1 cylinder
1.15
I 4
= - of tank = - 3.1 x 100 = 37.1 Ans. (i)
9
I
% of total power developed in No. 2 cylinder
I 9 0.95
:. Time to empty whole tank = - h = 2 h. Ans.
1 4
= -x 100 = 30.6
3.1
Ans. (ii)
II
282 R E E D ' S MATHEMATICS FOR E N G I N E E R S S O L U T I O N S TO T E S T E X A M P L E S 1 283
% of total power developed in No. 3 cylinder As explained in the text it is essential to divide one total quantity by
1 another total quantity to obtain an average. Note that the average
= -x 100 = 32.3 Ans. (iii) cost is not
3.1

12. + +
% zinc content = 100 - (7 1 1 3)
= 100 - 75 = 25
Total mass of alloy = 500 kg
71 '
Mass of copper = --- x 500 = 355 kg
100
1
Mass of tin = -x 500 = 5 kg
100
3
Mass of lead = -x 500 = 15 kg ' Am.
100
25 log, N
log, N = -
Mass of zinc = -x 500 = 125 kg log, b
100
Total = 500 kg lg
log, N = - N
lg 2
13. Sum of measurements
=27+39+47+51 +48+32+20+11 + 8 + 5 = lg N x 3.322
= 288 mm .'. Converting multiplier is 3.322 Ans.
Total number of readings = 10
total of measurements 17. By common logs:
Mean height =
total number of readings
288 lg 0.06326-0.25= 2.8011 x (-0.25)
--
-
10 = -1.1989 x (-0.25)
= 28.8 mrn Ans. = 0.2997
Antilog = 1.994 Ans. (a)
14. Cost of 200 t @ £60/t = £12 000
Cost of 600 t @ £70/t = £42 000 By natural logs:
Total cost = £54 000
ln 0.06326-~"~= (In 6.326 + In lop2) x (-0.25)
Total number of tonnes = 200 + 600 = 800 = (1.8446 + 3.3948) x (-0.25)
total cost
Average cost = = j.2394 x (-0.25)
total number of tonnes
= -2.7606 x (-0.25)
= 0.6901
Antilog = 1.994 Ans. (b)
284 R E E D ' S MATHEMATICS FOR E N G I N E E R S

Note. In Example 17 (and 18) tables have been used. A calculator


simplifies the work as the minus number can be entered directly and
the use of bar quantities is avoided. SOLUTIONS TO T E S T EXAMPLES 2
18. Finding the logs:
1n2.55 = 0.9361
ln0.08234 = In 8.234 + In lop2
= 2.1083 f 3.3948 1. (i) 3x+4y - 5z
= j.5031 (i.e. -2.4969) add -2r-5~+4~
x - y - z Ans.
+ ln 1
In 0.36 = ln 3.6 O K '

= 1.2809 + 3.6974
(ii) 2a2b - ab + 3ab2
= 2.9783 (i.e. - 1.0217)
ln10.2 = In1~02+ln10
add +
-a2b ab + 5ab2
a 2b + 8ab2 Ans.
+
= 0.0198 2.3026
(iii) From 5x + 3y - 4z
subtract 2x + 5y - 32
3x - 2y - z Ans.
Ink = 0.9361 + 15.5031 - (2.9783 + 2.3224)) + 2
= 0.9361 + 13.5031- 1.3007) + 2 From 3a - 2b + 6c
= 0.9361 + 4.2024 i2 subtract -a - 5b - 4c
= 0.9361 + 2.1012 4 a + 3 b + 10c Ans.
= i.0373 (i.e. - 0.9627)
2. (i) 5~-3~-4~-2y+4y+2z-y
To find antilog:
=5x-4x-2y+4y-y-3z+22
subtract In lo-' i.e. j.6974 (i.e. -2.3026)
=x+y-z Ans.
i.0373 - j.6974 = 1.3399
Antilog of 1.3399 = 3.819
(ii) + +
2 . 5 ~ c - 1 . 2 ~ 2.5b - 3c +b+ 1 . 7 ~
Antilog of i.0373 = 3.819 x lo-' = 2 . 5 ~ -1.2a+ 1 . 7 a + 2 . 5 b + b + c - 3c
kx0.3819 Ans.
= 3a + 3.5b - 2c Ans.

(iii) b2 - 3ab2 + 2a2b - 4a2 - 2b2 + 5a2 - 2ab2


= - 4a2 + 5a2 + 2a2b - 3ab2 - 2ab2 + b2 - 2b2
= a 2 + 2a2b - 5ab2 - b2 Ans.
286 R E E D ' S M A THEM A T I C S F O R E N G I N E E R S DULU IIUNb 1U IEbl LXAMPLbS 2 28.1

xxx3 X X ~
3. (i)
x2 xY'
- X1+3+5-2-4

= x3 A n s . (i)

(ii) x5 X x - ~X x - ~
-
- x5-3-2 -
- x0 a3 - 3a2b + 3ab2 - b3 A n s . (iv)
=1 A n s . (ii)
6. (i) 2a - 3b)8a2 - 8ab - 6b2(4a + 2b A n s . (i)

(iii) h
, x xi x x-f 8a2 - 12ab
- 1+-1- 2
- $ 2 3-X3

or 3d? A n s . (iii)

2m (ii) 3x - 4y)9? - 9 2 y - 10xy2 + 8y3(32+ xy - 2 9 Ans. (li)


=2 y A n s . (iv)
92- l G y
3X2y - 10Xy2
3 2 y - 4Xy2
x3 - x3
(v) x3 - 0.5~3- 0 . 5 ~ ~ - 6$ + 8y3
1
- -= 2 A n s . ( v )
0.5

+ + + +
4. (i) (x 2y)(2r y ) = @ 5xy 2 3 Ans. (i) (iii) x - y)x3 - y 3 ( 2 + xy +$) A n s . (iii)
+
(ii) (2r y)(3x - 2y) = 6 2 - xy - 2 3 A n s . (ii) ~3 - 2y
(iii) ( 3 x - 4 y ) ( h - 3y) = 6 2 - 17xy+ 123 A n s . (iii)

5. (i) (a+b12 = ( a + b ) ( a + b ) = a 2 + 2 a b + b 2 A n s . (i)


(ii) (a - b)2 = (a - b)(a- b ) = a2 - 2ab b2 A n s . (ii)+
+ +
(iii) (a + b)3= (a b)2 x (a b) = (a 2 2ab b 2 ) x (a + b)+ +
a2 + 2ab + b2
a + b
a 3
+ 2a b + ab2
2 7. (i) + +
(a b) (c - d ) - (a - b) - (c + 4 + (a - b)
=a+b+c-d-a+b-c-d+a-b
a 2b + 2ab2 + b3
=a-a+a+b+b-b+c-c-d-d
a + 3a2b + 3ab2 + b3
3
A n s . (iii) =a+b-2d A n s . (i)
288 R E E D ' S MATHEMATICS FOR E N G I N E E R S SO L U T IO N S TO T E S T E X A M P L E S 2 289

(ii) + +
2(a - 3(a 2) 4(2a - 1 ) 5 ) + (iii) 9 2 + 12v + 4 = (3v + 2)(3v + 2)
=2(a-3a-6+8a-4+5) or (3v + 212 Ans. (iii)
+
= 2a - 6a - 12 16a - 8 10 +
= 12a- 10 Ans. (ii)
(iv) 4 2 - 12xy+9$ = ( 2 ~ - 3 ~ ) ( 2 ~ - 3 y )
(iii) 2x-[2~-{2~-(2~-2)-2)-2]-2
or (2x - 3 ~ Ans.
) ~ (iv)
=2x-[2x-{2x-2x+2-2)-2]-2
=2x-[2x-2x+2x-2+2-2]-2
=2x-2x+Zr-2x+2-2+2-2
= 0 Ans. (iii) 11. (i)
,.,38u##n .#
..,, 8,s

t#:e:::il~[f~
'
,,I,* .,,,,8
,
*,,u,,,,,,,,8 ,
88h.k~~
8. (i)
X
-
= 3(b 2 - 2b+3) Ans. (i) - - Ans. (i)
12

(ii) pv pvx+
+
=pv(1 x ) Ans. (ii)
(ii)

(iii)
3
ax - 2 b 2 + 3cx
=x(& - 2bx+3c) Ans. (iii)

(iv) 12a3b3c3- 8a2b2c2 4abc +


= 4abc(3a b c 2 2 2
- 2abc + 1 ) Ans. (iv)

02 - d 2 = ( D + d)(D - d ) Ans. (i)


2
+
1 - a = (1 a)(l - a ) Ans. ( i ~ ) (iii)
(iii) +
4 x V - 9 2 = (2xy 3 z ) ( w - 32) Ans. (iii)
+
T1 - T2 = ( T ~ T22 ) ( ~ - 1 ~2 2 ,
+
or (TI f T ~ ~ ) ( T2)(T1
T ~ - T2) Ans. (iv)
a2 + 8a + 16 = ( a + 4 ) ( a + 4 )
or (a + 412 Ans. (i)

d 2 - 10d +25 = (d - 5)(d - 5)


1
- - Ans. (iii)
or (d - 512 Ans. (ii) x+2
290 R EE D ' S M A T H E M A T I C S F O R E N G I N E E R S SOLUTIONS TO TEST EXAMPLES 2 29 1
12. 2 ~ ~ - 2 ~ + X y 2 - x3= ~2 ,~y = - 2 16. Factor theorem: If ( x - a ) is a factor of f(x), the remainder is zero,
and f (a)= 0
= 2(213- (2)2(-2) + (2)(-2)2 - 3(-213
= 2(8)- (4)(-2) + (2)(4)- 3(- 8) Let x - a = x + 4
= 16- (- 8) + 8 +24 then a = -4
= 16+8+8+24 x + 4 is a factor, :. f (- 4) = 0
= 56 Ans. f(x) = 2+2- 10x+c
+
3[4x 2{x - 2y - (3x y)} - 3x1 +
+
f (- 4)= (-4)3 (-412 - lo(-4) +c
0=-64+16+4O+c
+
= 3[4x 2{x - 2y - 3x - y } - 3x1
~ = 6 4 -16- 40
+
= 3 [ 4 ~ 2r - 4y - 6~ - 2y - 3x1
= 8 Ans.
= 1 2 x + 6 ~ - 12y- 1 8 ~ - 6 y - 9 x
= - 9~ - 18y
17. Factor theorem:
substituting x = - 2, y = -3 Let x - a = x + l
- 9(- 2) - 18(-3) then a = - 1
= +18+54 x + 1 isafactor, .-.f(-1) = O
= 72 Ans. + 3 2 - bx - c
f (x) = x3
f(- 1)= (- 1)3 + 3 ( - 1 ) ~- b(-1) -c
14. Remainder theorem: I
0=-1+3+b-c
R = f (a) when f (x) is divided by ( x - a ) I c=b+2 . . . . . . . . . . . . . . . (9
Let x - a = x - 3 Remainder theorem:
I
then a = +3 I
I
Let x - a = x + 2
+ 5x 6
f (x) = 2x3 - 4 2 - then a = -2
R = f (a) = 2(3)3- 4(312+ 5(3)- 6 I f(x) = 2+ 3 2 - bx - c
=54-36+15-6
I f (- 2)= (-2)3 + 3(-212 - b(- 2)- c
= 27 Ans. 15=-8+12+2b-c
c = 2 b - 11 . . . . . . . . . . . . (ii)
15. Let x - a = x + 5 I
I From (i) and (ii),
then a = - 5
+ 6 2 + bx + 9
f (x) = x3 2b- 11 = b + 2
R = f ( a ) = (-5)3 + 6(-512 + b(- 5)+ 9 b = 13
4=-125+150-5b+9 Substituting for b into (i)
+
5b = 150 9 - 125 - 4 = 30 C= 13+2=15
b = 6 Ans. ! Constants are 13 and 15. Ans.
I
292 REED'S MATHEMATICS FOR ENGINEERS

S O L U T I O N S TO T E S T E X A M P L E S 3

8+5~-7=3x+9
5x-3x=9+7-8
2x=8
x= 4 Ans.

2. 2(a + 3 ) + 3(2a - 4 ) = 4(11 - 3a)


2a+6+6a- 12=44- 12a
2a+6a+12a=44+12-6
20a = 50
a =2; Ans.

I multiplying throughout by 60,

7x = 70
x = 10 Ans.
294 R E E D ' S M A TH EM A T I CS F O R E N G I N E E R S S O L U T I O N S TO T E S T E X A M P L E S 3 295

multiplying throughout by 12a, speed x time (of fast ship) = speed x time (of slow ship)
17.5xx=16x(x+4$)
= 16x
17.5~ + 72
17.5-
~ 1 6=
~ 72
1 . 5=
~ 72
-4a2 and + 4a 2
cancels,
x = 48
-3a - 2a = -30 - 2 .'. time for fast ship to overtake = 48 h Ans. (i)
-5a = -32 distance = speed x time
a=6.4 Ans. = 17.5 x 48
= 840 naut. miles
.'. Distance from port = 840 naut. miles Ans. (ii)

Let b = breadth of plate, in m, then


+
Note that (x - 3)(x 2) are the factors of x2 - x - 6, therefore the
4b = length of plate, in m,
L.C.M. of the denominators is x2 - x - 6, or more conveniently
(x - 3)(x + 2 ) for the purposes of multiplying throughout this length x breadth = area
equation. 4bxb=1
4b2 = 1
Multiplying by (x - 3)(x + 2), b2 = 0.25
b= = 0.5
.'. breadth is = 0.5 m
length = 4 x 0.5 = 2 m
] Ans.

x=1.8 Ans. 9. Note that Ro cancels,

7. The equality of the ships is that the distance from port to meeting
I
place is the same, therefore the equation is: Distance travelled
by fast ship = Distance travelled by slow ship, and for distance
write speed x time.
I
Let x = time taken by fast ship (h)
+
then x 4; = time taken by slow ship (h)
296 R E E D ' S MATHEMATICS FOR ENGINEERS SOLUTIONS TO TEST EXAMPLES 3 297

..?-- Inserting values, 12. 40.59~- 56.36

x=
0.6021 x 0.59
1 = 4.929 Ans. (a)
- -x 0.2756
0.0042 .1.95 = 12.4xo,53
=65.6 Ans. (ii)

Square both sides,

x = 5.888 Ans. (b)

Multiply both sides by R and divide by 2,


R=- 03 Ans. (i)
3.5 nd2
Substituting values,

= 290 mm Ans. (ii)

x = 1808 Ans. (d)


Note. Logarithms to any base can be used to solve Examples 11
and 12.

n=-= 1.347 Ans.


0.9097
S O L U T I O N S TO T E S T E X A M P L E S 4 299
Multiplying (i) by 60, and multiplying (ii) by 16,

40x - 36y = 45 . . . . . . . . . . . (iii)


S O L U T I O N S TO T E S T E X A M P L E S 4 8x - 4y = 13 . . . . . . . . . . . . (iv)

Multiplying (iv) by 5 and subtracting (iii) fiom the result,


1. Let x = first number, and y =second number.
From the first statement,
2 i x + 3iY = 19 . . . . . . . . . . . . 6)

From the second statement,


212 Y - 3 i x = 3 . . . . . . . . . . . . (ii) Substituting y = 1 into (iv),

Re-arranging terms in (i) in the same order as (ii),


3 i y + 21x = 19 . . . . . . . . . . . . (iii)
21.2y - 3 12 . x ~ 3 . . . . . . . . . . . . (iv)
Multiplying (iii) by 5 and multiplying (iv) by 7 ,
The values of x and y are 2: and 1 ; respectively. Ans.
17iy + 1 2 i x = 95 . . . . . . . . . . . . (v)
17iy - 244x = 21 . . . . . . . . . . . . (vi)

Subtracting (vi) from (v)


37x = 74
x= 2
Removing brackets,
Substituting x = 2 into (i)
a +2ab = 3 . . . . . . . . . . . . (9
2; x 2 + 3 i y = 19 a - 3ab = 0.5 . . . . . . . . . . . . (ii)
3iy= 19- 5
3iY = 14 If (ii) is subtracted from (i) to eliminate a, it would still leave a
y= 4 1 term containing two unknowns, therefore proceed to eliminate the
terms containing ab by multiplying (i) by 3, and (ii) by 2,
The numbers are 2 and 4. Ans.
+
3a 6ab = 9
2x 3y-
---= 3
............ I 2a - 6ab = 1
2. 0)
x- - -y= - 13 add 5a = 10
. . . . . . . . . . . . (ii)
2 4 16 I a=2
300 R E E D ' S M A T HEM A T I CS F O R E N G I N E E R S SOLUTIONS TO TEST EXAMPLES 4 301
Substitute a = 2 into (i), Substituting x = 12 into (ii),

Their ages are 12 and 8 years respectively. Ans.


The values of a and b are 2 and 0.25 respectively. Ans.
Alternatively divide the original equations, thus, 5. Let x = one number
and y = other number,
x- y=2 . . . . . . . . . . . . (i)
1 +2b 2-J=6 . . . . . . . . . . . . (ii)
a cancels, -= 6
1 - 3b
Factorise (ii) and divide by (i),

x - y cancels,
x+y=3 . . . . . . . . . . . . (iii)
Substituting b = 0.25 into the first equation,
Add (iii) to (i),
a(l + 2 x 0.25) = 3
~3
1 . 5= x-y=2
a=2 x+y=3
.'. a = 2 and b = 0.25 as before. 2x =5
I
x = 2 12
Substitute x = 24 into (iii),
4. Let x = age of grandson
and y = age of grand-daughter I 2i+y=3
4x + 3y = 72 . . . . . . . . . . . . (i) y =l- 2
3x+4y=68 . . . . . . . . . . . . (ii) The numbers are 24 and $. Ans.
I

Multiplying (i) by 4 and (ii) by 3 and subtracting, 6. Inserting the two sets of values of F and m,

subtract 35 = a + b x 30 . . . . . . (ii)
20 = 40b
SOLUTIONS TO TEST EXAMPLES 4 303
I 302 R E E D ' S MATHEMATICS FOR ENGINEERS
1
Substituting x = 16 into (i),
Substituting b = 0.5 into (i),
55 = a + 0.5 x 70 9 x 16
55 = a + 35
a = 20
Constants are, a = 20 and b = 0.5. Ans. (a) :. Original speeds of ships are 16 and 18 knot respectively. Ans.
+
Linear law is, F = 20 0.5 m. Ans. (b)
When m = 60, X
8. Let a represent -and let b represent -:1
F = 20 + 0.5 x 60 2-Y X
then the two equations may be written,
= 20 30+
=SON Ans.(c) a+6b=4 . . . . . . . . . . . . (9
2a - 9b = 1 . . . . . . . . . . . . (ii)
7 . Let x =original speed of A
and y =original speed of B, Multiplying (i) by 2 and subtracting (ii) from the result,
I I
Time for A to be overtaken = 9 h
I/
I Time for B to overtake A = 8 h

Distance from port to point of overtaking is the same for each ship,
I
and distance = speed x time.
.'.xx9=yx8
4
Substituting b = into (i),

If speed had been 4 knot slower,


Speed of A would be (x - 4 ) knot
Speed of B would be 0)- 4 ) knot,
X
Time for A to be overtaken = 7 h a = 2, . -= 2 Ans. (a)
2-Y
Time for B to overtake A = 6 h 1 1 - 1-
... (x -- 4 ) x 7 = 0).4 ) x 6 b=- . - - Ans. (b)
3' " x 3
7 x - 28 = 6 y - 24 1 -1
--
- :. x = 3 Ans. (c)
7x - 6y = 4 ........... x 3'

Substituting value of y from (i) into (ii),


6 x 9x
7x--=4 8 3=4-2y
7x - 6 i x = 4 2y=4-3
tx=4 2y = 1
x = 16 Y=i Ans. (d)
SOLUTIONS TO TEST EXAMPLES 4 305
304 REED'S MATHEMATICS FOR EN GIN EERS
Multiplying (i) by 5, subtracting (ii) from the result, and solving y,
9. From the first equation,
ZX = qr
2x = 2 s
.: x=2y . . . . . . . . . . . . (i)

From the second equation,


4x-1 = 2y+l Substituting y = 2.8 into (i) and solving x,

22-2 = 2y+l
:. 2 x - 2 = y + 1
2x-y=3 . . . . . . . . . . . . (ii)

Substituting x = 2y from (i) into (ii),


The values of x and y are 1.4 and 2.8 respectively. Ans.
2x2y-y=3
3y = 3 11. Taking square root throughout,
y=l

Substituting y = 1 into (i),


From d--
- Y dD
x=2 D' y=- ......
X
d- - x
The values of x and y are 2 and 1 respectively. Ans. From -- 2 = dy . . . . . . . . . . . . (ii)
x Y'
Substituting the value of y in (i) into (ii),

Expressing the equations in log form,


+ +
(lg 1.259)(x 1) (lg 1.175)b- 1) = lg 2.323
(Ig 3.162) x x + (lg 1.778) X Y = lg 25.12

Inserting log values, Inserting values of d and D,

0.1000(x + 1) + 0.07000, - 1) = 0.3660


0.5000 x x + 0.2500 x y = 1.4000 Inserting values of d, D and x into (i),
Simplifying the equations,
0. lx + 0 . 0 7 ~= 0.336 . . . . . . . . . . . . (i)
0 . 5+~ 0 . 2 5 ~= 1.4 ............ (ii) The values of x and y are 36.06 and 52 respectively. Ans.
306 R E E D ' S MATHEMATICS FOR E N G I N E E R S

2a+3b+4c=26 . . . . . . . . . . . . (ii)
+
4a - 2b c = 10.25 . . . . . . . . . . . . (iii) SOLUTIONS TO TEST E X A M P L E S 5
Multiplying (i) by 2 and adding (ii) to the result,

1. (i) +
(2x 8)(3x - 5) = 0
. . . . . . . . . (iv) +
either 2x 8 = 0 I or 3x - 5 = o
then &=-8 then 3x=5
Multiplying (iii) by 4 and subtracting (ii) from the result,
x = -4 x = 1'3
16a - 8b + 4c = 41 x = -4 or 1: Ans. (1)
2a + 3b + 4c = 26
14a - l l b = 15 . . . . . . . . . . . . (4
Multiplying (iv) by 7, multiplying (v) by 4, and subtracting,
(ii) ~ 10)(0.25x+ 5) = 0
( 0 . 5-
56a + 1056 = 283.5
either 0 . 5 -
~ 10 = 0 or 0.25~+5 = 0
56a - 446 = 60
then 0 . 5=~ 10 then 0 . 2 5 ~= -5
x = 20 x = -20
x = f20 Ans. (ii)
Substituting b = 1.5 into (iv),
8 a + 15 x 1-5 =40.5
8a = 40.5 - 22.5 (iii) (5x + 0.5)(4x + 0.8) = 0
8a = 18 either 5x + 0.5 = 0 or 4x+ 0.8 = 0
a = 2.25 then 5~ = -0.5 then 4x = -0.8
Substituting a = 2.25 and b = 1.5 into (iii), x = -0.1 x = -0.2
4 x 2.25 - 2 x 1 . 5 + ~
= 10.25 X = -0.1 or - 0.2 Ans. (iii)
C = 10.25- 9+3

c = 4.25
Values of a, b and c are 2.25, 1.5 and 4.25 respectively. Ans. 2. (i) 32+2x-33=0
+
(X- 3)(3x 11) = 0
either x - 3 = 0 or 3~+11=0
then x=3 (
then 3 x = -11
x = -323
x=3 or -33 Ans. (i)
308 R E E D ' S MATHEMATICS F O R E N G I N E E R S S O L U T I O N S TO T E S T E X A M P L E S 5
-
309

(ii) 41-17x+4=0 (iii) 42-9x+2=0


(x - 4)(4x - 1) = 0
either x - 4 = 0 or 4x - 1 = 0
then x =4 then 4x = 1
x =' 4
x=4 or $ Ans. (ii)

(iii) 1G+10x-12=0
+
2(62 5x - 6) = 0
2(3x - 2)(2x 3) = 0+
either 3x - 2 = 0 ( or 2x 3 = 0 + x = 2 or $ Ans. (iii)

then 3x = 2 then 2x = -3
x =z3 x = -1I2 I
x =: or -1 Ans. (iii)
4. (i) 3 2 -2x+0.25 = O
I a = 3, b = -2, c = 0.25

3. (i) 2-x-3:=0
1- x + (+)*= 3; + (?)1 2
x-+=*A 1 1
x = -2 or 6- Ans. (i)
x=f2+i
x = 2 4 or - 1 Ans. (i)
(ii) 5 2 + 4x - 5.52 = 0
a = 5, b = 4, c = -5.52
(ii) 32+2x-1=0
2+$x-f=o
2 + $x + (+I' = f+ (4)'

X=f2--1
3 3
x = + or - 1 Ans. (ii) x = 0.724 or - 1.524 Ans. (ii)
310 R E E D ' S M A THEM A TI C S F O R E N G I N E E R S S O L U T I O N S TO T E S T E X A M P L E S 5 311

(iii) 102 -x- 0.2 = 0 The solution of this quadratic gives,


a = 10, b = -1, c = -0.2 x = 3.5 or 1.6
1k,AT3 x = vI.4, .. v=
x=
2 x 10
l f 3 v 2/35 or 1.41/16
x=-
20 V = 2.446 or 1.399 Ans.
I
4 -2
x=-
20
or -
20 ~ 8. 2 -xy+29 = 16 . . . . . . . . . . . . (i)
x = 0 . 2 or -0.1 Ans. (iii) x + 2y = 8 . . . . . . . . . . . . (ii)
log 0 . 5 =
~ 2 x log(x - 6) From (ii)
.&..
@z:lU[l 0 . 5 = (X- 6) 2
I.,
, 2y=8-x
0 . 5 x = 2 - 12x+36 y = 4 - 0 . 5 ~ . . . . . . . . . . . . (iii)
2 - 1 2 . 5 +~ 36 = 0
Substitute this value of y into equation (i),
Solving this quadratic,
x=8 or 4.5 Ans.

6. 6b4 - 246b2 + 0.24 = 0


Dividing throughout by 6,
b4 - 0.41b2 + 0.04 = 0

Let x represent b2,


By factors,
2 - 0 . 4 1+
~ 0.04 = 0
(x - 2)(x - 4) = 0
The solution of this quadratic gives, either x - 2 = 0, then =2
or x - 4 = 0, then x = 4

From (iii)
if x=2, y=4-0.5~2=3
b = f0.5 or f0.4 Ans.
if x=4, y = 4 - 0 . 5 ~ 4 ~ 2

7. ~ 2 . 8- +5.6 1 0 Values are:


Let x represent
2 - 5 . 1 +~ 5.6 = 0
x=2
or x = 4
and y = 3
and y = 2
] Ans.
312 R E E D ' S MATHEMATICS FOR E N G I N E E R S S O L U T I O N S TO T E S T E X A M P L E S 5 313
9. Let x = speed of slow ship (knot). 11. G + 3 ~ ? - 1 7 x - 3 0 = 0
330 Finding the first root by trial,
Time to do 330 naut. miles = -x h . . . . . . . . . (9 ,
I
(x + 3;) = speed of fast ship (knot). i
334
Time to do 334 naut. miles = -
x+3$
. .. . . . . . . (ii) i
Time (ii) is 5 h less than time (i), therefore,
330 334
-5

x = 3 satisfies the equation, this is one root and x - 3 must be a


The solution of this quadratic gives x = 13.2, factor. Divide the cubic equation by this factor.
:. speed of slow ship = 13.2 knot
+
speed of fast ship = 13.2 3.5 = 16.7 knot
1 hs.

10. Working in cm;


Let L =length, and B =breadth,
length x breadth = area, .'. L x B = 76 (9
length + breadth = $ perimeter, .'. L + B = 17.5 (ii)
Equating the resulting quadratic to zero and finding its roots by
76 formula,
Substituting B=- L from (i) into (ii),

Multiplying throughout by L,
L~ + 76 = 17.5L
L~ - 17.5L+76 = 0
The solution of this quadratic gives L = 9.5 or 8,
... length = 9.5 cm = 95 mm
breadth = 8 cm = 80 mm
} hs.
The roots of the given equation are:

3, - 2, and - 2.5. Ans.


314 R E E D ' S MATHEMATICS FOR E N G I N E E R S I S O L U T I O N S TO T E S T E X A M P L E S 5 315
x = 4 is a root and x - 4 is a factor. Divide the cubic equation
I by this factor,

Multiplying terms within the brackets by the multiplier outside, I

to eliminate the brackets,

Multiply throughout by the least common denominator, which


is 3,to eliminate fractions, Finding the roots of the resulting quadratic by formula,

2x3 - 32x+48 = 3 2

Arrange all terms on left hand side in order of descending powers

2x3-32-32x+48=0

Proceed to solve this cubic equation by finding the first root


I
I
by trial, I

The roots of the given equation are:


4, 1.5, and -4 Ans.

Tryx = 4,
! SOLUTIONS TO TEST EXAMPLES 6 317

S O L U T I O N S TO T E S T E X A M P L E S 6

1. Finding plotting points:

(i) y = 2 + x ,

when x = 0, y=2+ 0=2


when x = 12, +
y = 2 12 = 14

when x = 0, y=-1-0=-1
when x = 12, y = -1 - 6 = -7

when x = 0, y=-4+ 0=- 4


when ~ ~ 1 2 y =, - 4 + 1 5 = 1 1

The four plotted graphs are as shown.


318 R E E D ' S M A T HE M A T I C S F O R E N G I N E E R S S O L U T I O N S TO T E S T E X A M P L E S 6 319

3. From the above graph:


a=4
b=-9+3=-3

.'. Law is y = 4 - 3x. Ans.

2. From the graph:


a = 11
b = -17.5 + 10 = -1.75

.'. the equation is y = 11 - 1 . 7 5 ~ . Ans. POWER


320 R E E D ' S MATHEMATICS FOR E N G I N E E R S S O L U T I O N S TO T E S T E X A M P L E S 6

4. Reading the graph rate of consumption and power (previous page):

.'. Law is m = 30 + 9.5P Ans.

5. Expressing y in terms of the other quantities,


first equation, 3x + 5y = 23
5y = 23 - 3x
y = 4.6 - 0 . 6 ~ . . . . . . (i)
second equation, 5x - 2y = 12.5
-2y = 12.5 - 5~
2y = 5~ - 12.5
y = -6.25 + 2 . 5 ~ . . . . . . (ii)

Choosing two plotting points for the graphs of each equation,


equation (i), when x = 0 , y = 4.6 - 0 = 4.6
when x = 5, y = 4 . 6 - 3 = 1.6
equation (ii), when +
x = 0 , y = -6.25 0 = -6.25
when +
x = 4, y = -6.25 10 = 3.75

The two graphs are now drawn from which read the point of
intersection,
x = 3.5
y = 2.5
} Ans.

6. Expressing p in each equation in terms of the other quantities,

2p = -4.8 + 39
p = -2.4 + 1.5q ...... (ii)
S O L U T I O N S TO TEST E X A M P L E S 6 323
322 RE E D ' S M A THEM A TI CS F O R E N G I N E E R S
7. ?-5x+56=0
Plotting points for each equation:
Let y = x Z - S x + 5 6
when x=O, y= 0- 0+5;=5i
when x = 1, y= 1- 5+5i=li
when x=2, y= 4-10+5$=-$
when x=3, y= 9 - 1 5 + 5 '4- --? 4
when x=4, y = 16-20+5i= 1:

From the graph,


From the graphs:
y=O when x = 1.5 and 3.5
p =2.4 and q = 3.2. Ans.
Therefore the roots of the equation are 1.5 and 3.5. Ans.
324 R E E D ' S M ATHEMATICS FOR E N G I N E E R S S O L U T I O N S TO T E S T E X A M P L E S 6
325
8. Finding plotting points, (iv) The difference between
0 . 5 1 - 2x - 6
and 0 . 5 2 - 2x (subtract)
is -6

Hence, values of x are read where the graph crosses the line of
y =-6,
x=0 and 4 Ans. (iv)

The points are plotted and the graph drawn as shown.


(i) 0 . 5 2 - 2x - 6 = 0
The roots of this equation are where the graph crosses the
base line y = 0, thus
x=-2 and + 6 Ans. (i)
(ii) The difference between
0.52-2x-6
and 0 . 5 2 - 2x - 4 (subtract)
is -2
Thus, when y= - 2 on the graph of y = 0 . 5 2 - 2x - 6 we
read the roots of the equation 0 . 5 2 - 2x - 4 = 0, that is,
x = -1.5 and + 5.5 Ans. (ii)
(iii) The difference between

and 0 . 5 2 - 2x - 1 (subtract)
is -5
Therefore the values of x in this equation are where the graph
crosses the line of y = - 5, thus,
x=-0.5 and +4.5 Ans. (iii)
I(
326 R E E D ' S MATHEMATICS FOR E N G I N E E R S S O L U T I O N S TO T E S T E X A M P L E S 6 327
(v) The difference between Y
0.52-Zr-6
and 0.52 - Zr +1 (subtract)

Then read off the graph values of x when y = - 7,


x = 0.6 and 3.4 Ans. (v)

9. The plotting points are calculated and the two graphs drawn for each
equation.
2 - 2 . 5 ~- 3-5 = 0
can be written 2 - ( 2 . 5 ~+ 3.5) = 0
and represented by y , - y2 = 0
where y, = 2, and y2 = 2 . 5 ~ 3-5 +
Hence, where the graphs intersect gives the value of x where
yl - y~= 0 and therefore the roots of the equation
a? - 2 . 5 ~- 3.5 = 0.
.'. x = -1 and + 3.5 Ans. I

10. The two graphs are plotted (p. 328). The points of intersection
produce the values of x and y as:
x=
x=10
1 and y = -0.6
and y = 1 2
) Ans.

11. The graphs are as plotted (p. 329). The points of intersection give
the values of x and y as:
X = 3.65
x = 5.4
and y = 7.6
and y = -9.7
} Ans.

I
12. The graphs are as plotted (p. 330). Note that y = e-" is y = I / @
where e = 2-718. Point of intersection is solution of equation
S O L U T I O N S TO T E S T E X A M P L E S 6 329
328 R E ED ' S M A T HE M A T I C S F O R E N G I N E E R S
330 REED ' S M A THEM A TI CS F O R E N G I N E ER S

S O L U T I O N S TO T E S T E X A M P L E S 7

1. (i) 1 rad = 57.3" = 57'18'


114'36' = 114.6"
114.6 s 57.3 = 2 rad. Ans. (i)
286'30' = 286.5"
286.5 + 57.3 = 5 rad. Ans. (i)

(ii) (a) Radius of circle = 5 m


10 + 5 = 2 rad. Ans. (ii) (a)
30.4 t 5 = 6.08 rad. Ans. (ii) (a)

(ii) (b) Deg = rad x 57.3


2 X 57-3 = 114.6' = 114O36' Ans. (11) (b)
6.08 X 57.3 = 348.384" = 348'23' Ans. (ii) (b)

v = wr
10.52 x 0.I = 1.052 m/s Ans. (i)
10.52 x 0.5 = 5.26 m/s Ans. (ii)
Circumference = x diameter
In one revolution, rim travels 71 x 2 m
In one min., rim travels 71 x 2 x 125 m
7 1 x 2 125
~
In one s, rim travels m
60
Linear velocity = 13.09 m/s Ans. (iii)

=-=45 rnm Ans. (i)


332 REED ' S M A THE M A TI C S F O R E N G I N E E R S SOLUTIONS TO TEST EXAMPLES 7 333

36
'pposite ,-
sin B = ,0.8 Ans. (ii)
hypotenuse 45
adjacent = ,0.6
27 Ans. (iii)
COSB=
hypotenuse 45
= opposite
- , 36
- = 1.333 Ans. (iv)
adjacent 27

7. sin 80" = 0.9848


sin 8 = JiTZGG I
cos 80" = 0.1736
= J8%% = 0.8704 Ans. (i)
sin 28 = sin 160' = 0.342
sin8 0.8704 Ans.
tang=-=- r
case 0.4924 I
cos 28 = cos 160' = -0.9397
= 1.767 Ans. (ii) sin2 8 = (0.9848)~= 0.9701
cos2 8 = (0.1736)~= 0.03013
COSINE TANGENT
5. ANGLE SINE
10'33' 0.1831 0.9831 0.1862
0.6831 1.0692
46'55' 0.7304 Ans.
150"47' 0.4882 -0.8728 -0.5593
-0.9314 0.3909
201°21' -0.3641
-3.2235
I Substituting this value of sin2 8 into the given equation,
287' 14' -0.9551 0.2963 I

sine and cosine curves We as shown @. 333). Ans.

6. RATIO VALUE ANGLE


sine 0.3783 22O14' and 157'46'
sine -0.7005 224'28' and 315'32'
cosine 0.9687 14'22' and 345'38' Ans. This is a quadratic equation, re-arrange in the usual way in
cosine -0.8769 151D16'and 208"44' descending powers of the unknown,
tangent
tangent
0.2010
-3.2006
11'22' and 191'22'
107'21' and 287'21' 1 cos2 8 + cos 8 - 1 = 0
S O L U T I O N S TO T E S T E X A M P L E S 7 335
334 REED'S MATHEMATICS FOR ENGINEERS

Solve this quadratic equation either by formula or 'completing the Simplifying right hand side of equation:
square', if preferred, let x stand for cos 8,
2+x-1=0
Solution to this quadratic is 0.618 or - 1.618 2
1 - sinA (1 - sin A ) ~
:. cos 8 = 0.618 (other value inadmissable) ={ cosA } = cos2A
8 = 51°50f
or (360" - 51"50f)= 308"10f
) hs. -
-
(1 - sin A)' - (1 - sinA)(l - sin A)
1 - sin2 A
-
+
(1 sinA)(l - sin A)
- 1 - sinA
= left hand side of equation. Ans.
+
1 sinA

12. 1
(sec A - cos A)(cosec A - sin A) =
tanA+cotA

sin28 = 5 C O S ~8 - 2 Simplifying left hand side of equation:


I
sin2 8 = 5(1 - sin2 8) - 2 (sec A - cos A)(cosec A - sin A)
2
sin2 8 = 5 - 5 sin 8 - 2
1
- --
6sin28=3 I -(cos A cos A } (I
sin A - sin A ]
sin2 8 = 0.5
1- C O S A 1 - sin2 A sin2 A cos2A
sin 8 = &m= f0.7071
-
-
cos A
~
x
sin A
--
-
cos A
x-
slnA
I
For angles between 0" and 180", only the positive value of the = sin A cos A . . . ... . . . . . . . . . . . .
sine is applicable.
sin 8 = 0.7071 Simplifying right hand side of equation:
I
:. 8 = 45" or 135" Ans. 1 - 1
tan A + cot A - sin A cos A
cos A
-+-
sin A
1
Simplifying left hand side of equation: -
- - sin A cos A
sin A + cos2A
2
sin2 A + cos2A
sin A cos A

- cos2A + sin2 A 1 - sin A cos A = sin A cos A


-
- -
2
... .. (ii)
2
sin A cos2 A sin A cos2A 1
= cosec2 A sec2 A = right hand side of equation. Ans.
From (i) and (ii) both sides of the equation are equal to the same
1-sinA quantity, which proves the given identity. Ans.
= (sec A - tan A ) ~
1 + sin A
336 R E E D ' S MATHEMATICS FOR E N G I N E E R S S O L U T I O N S TO T E S T E X A M P L E S 7 337
One triangle:
Remaining side, AC, = 18 mm
r = half-stroke = 0.5 m

n=
connecting rod length - -2-- 4
Remaining angles, B and C, = 2 0 r and 109 f ) Ans.

crank length 0.5 Other triangle:


0 = 80°, sin 80" = 0.9848, cos 80" = 0.1736
20 = 160°, sin 160" = 0.342, cos 160" = -0.9397
Remaining side, AC2 = 44 mm
Remaining angles. B and C, = 59 f' and 70 f'
48 : 60 :: 39 : BC
1 Ans.

BC = 48 . 7 5 mm Ans.

= 15-71 x 0.5 x 1.02755


= 8.072 m/s Ans. (i)

I
= -7.567 m/s2 i.e. deceleration Ans. (ii) Chord CD = 78
CO=DO=39-
-
14. There are two possible triangles which satisfy the conditions given
as shown.

Diameter = A 0 + BO
~ 2 . 0 3 1+749 = 751.031 mm
Increase = 75 1.03 1 - 750
= 1-031 mm Ans.
338 R E E D ' S M A T H E M A T I C S F O R E N G I N E ER S S O L U T I O N S TO T E S T E X A M P L E S 7 339

Radius of inscribed circle = 14.4 rnm. Ans.

this can be taken as 12750 km


CO = DO = observation distance.
AOxBO=COxDO
0.03 x 12750 = ( ~ 0 ) ~
CO = 19.55 krn Ans.

Radius of circumscribing circle = 28.6 rnm. Ans.


SOLUTIONS TO TEST EXAMPLES 8 34 1

SOLUTIONS TO TEST EXAMPLES 8


opposite = 120 x sin28'37'
= 57.48 mm Ans. (i)
adjacent = 120 x cos 28'37'
= 105.3 mm Ans. (ii)
area = (base x perp. height)
- 105.3 x 57.48
-
2
= 3026 mm2 or 30.26 cm2 Ans. (iii)
opposite (mast)
= tan 45'34'
adjacent (ground)
... h = 15 x tan45'34'
= 15.3 m Ans.

h = x x tan 48'30'
= x x 1.1303 . .. .. . ... 6)
also, h = (10 + x) x tan37'38'
opposite (cliff) = tan 14"25'
adjacent (sea) +
= 7.71 0 . 7 7 1 ~ . . . . . . . . . (ii)
From (i) and (ii),

95
+
1 . 1 3 0 3 ~= 7.71 0 . 7 7 1 ~
- = 369.5 m Ans. (i) 0.3593= ~ 7.71
tan 14'25'
opposite x = 2 1 . 4 6 m Ans. (a)
= sin 14'25'
hypotenuse From (i),
opposite
hypotenuse =
sin 14"25' = 21.46 x 1.1303
- 95
- = 381.5 m Ans. (ii) = 24.25 m Ans. (b)
sin 14O25'
342 R E E D ' S MATHEMATICS FOR E N G I N E E R S
S O L U T I O N S TO T E S T E X A M P L E S 8 343
B = 180" - (105'30' + 3.5'30') = 39".
By sine rule,

a b
sin A - sin B
b x sinA
a=
sin B

- 15 x sin35'30' = 13.84 m
-
sin 39"
b c
sin B - sin C
b x sin C
C =
sin B

- 15 x sin 105'30'
- = 22.97 m
By Pythagoras, sin 39"
diagonal across face = =
= 84.85 mm Ans. (i)
Length of jib = 22.97 m

Length of tie = 13.84 m


] Ans.
Cross diagonal to opposite corners, through centre,
= Jm
= J602 + 602+ 602 =
= 103.9 mm Ans. (ii)
I 7. On the sketch (p. 344) the x is the distance the crosshead has moved
from the top of its stroke when the crank is 35" past top centre.

Crank length = stroke = 400 mm


When crank is on top dead centre,
distance from shaft centre to crosshead
= 1600+400=2000 mm

When crank is 35" past top dead centre, distance from shaft
centre to crosshead is now to be calculated:

-- -
sin A - sin C
a x sin C
sinA =
344 REED'S MATHEMATICS FOR ENGINEERS I S O L U T I O N S TO T E S T E X A M P L E S 8 345

.'. A = 8' 15'


B = 180" - (35' +
8'15') = 136'45'
b
-- -
C
sin B - sin C
c x sin B
b=-
sin C
- 1600 x sin 136'45'
- = 1911 rnm
sin 35'
x = 2000 - 191 1 = 89mm Ans.

8. Let stroke = 1 , crank length = 0.5, conn. rod length = 2.


When crank is on top dead centre, distrance from shaft centre to
+
crosshead = 0.5 2 = 2.5
When crosshead has moved down 0.1 of its stroke, distance from
shaft centre to crosshead = 2.5 - 0.1 = 2.4,
B y cosine rule,

cos C =
a2 + b2 - c2
2ab

.: C = 33'7' Ans.
346 R E E D ' S MATHEMATICS FOR E N G I N E E R S I S O L U T I O N S TO T E S T E X A M P L E S 8 347
By cosine rule,
a 2 = b2 + c2 - 2bccosA
= 4 2 + 1 7 2 - 2 x 4 x 17xcos50 °
+
= 16 289 - 87.42
a=d m = 14.75
By sine rule,
a b
-- -
sin A - sin B
sin B =

-
-
b x sinA

4 x
a
sin 50"
= 0.2078
C 1 73O3

h at 18 knot represents a distance of 9 naut. miles.


A
(POI 1)

14.75 Let x naut. miles = distance of first ship from port


+
then (x 9) naut. miles = distance of second ship from port
By cosine rule,
:. Resultant speed of ship = 14.75 knot
Resultant direction of ship = 12"East of South
] Ans.
a 2 = b2 + c2 - 2bc cos A
2
50 = (x+ 912 + 2 2(x + 9) x x x ~ 0 ~ 7 3 ~ 3 9 '
-

B 2500 = 2+ 18x+81 + 2- 2 x x x 0.2815(~+9)


2500 = + 18x+ 81 0.5632 - 5.067~
-

A
2419 = 1.4372 + 12.933~
C 2 X NAUI. MILLS ( P O R r)
+
or, 1.4372 12.933~-2419 = O
Solving this quadratic,
x = 36.8
Let x = nearest ship's distance from port
then 2.x = other ship's distance from port.
Distance from port = 36.8 naut. miles
and (36.8 + 9) = 45.8 naut. miles
} Ans.
a2 + c2 - 2bc cos A
=b 2

322 = (2x1~+ 2 - 2 x 2.x x x x cos 23"


1024 = 4x2 + 2- 3.682.?
1024 = 1.3182

.'. distances from port= 27.87 naut. miles


and 27.87 x 2 = 55.74 naut. miles
} Ans.
348 R E E D ' S M A T HEM A TI C S F O R E N G I N E E R S SOLUTIONS TO TEST EXAMPLES 8 349

By cosine rule, 14. Area = Js(s - a)(s - b)(s - c)

S=
+ +
7.1 4.2 5.3
= 8.3
2
s - a = 8.3 - 7.1 = 1.2
s - b = 8.3 - 4.2 = 4.1

= 11.07 cm2 Ans.

:. Angle A = 180' - 69'50' = 110'10' Area = 0.433 x side2

By sine rule,
a b
-
sinA sin B = 11.5 cm Ans.
b x sinA
sinB =-
a

-
10.2 x sin 1 10"101
= 0.5838
16.4

:. Angle B = 35'43' Substituting this value of cos2 8 into the equation:

Angle C = 180' - (110'10' + 35'43') cos 28 = cos2 8 - sin2 8


= 1 - sin2 8 - sin2 8

The three angles are, :. cos28 = I - 2 sin2 8 Ans. (i)

110' 10'. 35'43' and 34'7' Ans.


sin2 8 + cos2 8 = 1
:. sin2 8 = 1 - cos2 8

Substituting this value of sin2 8 into the equation:


ab sin C
13. Area = -
2 cos 28 = cos2 8 - sin2 8
= C O S ~8 - (1 - C O S ~8)
= C O S ~8 - 1 + C O S ~8
2 2
= 17.78 m Ans. .'. cos 28 = 2 cos 8 - 1 Ans. (ii)
350 R E E D ' S MATHEMATICS FOR E N G I N E E R S S O L U T I O N S TO T E S T E X A M P L E S 8 35 1
(1 + cot2
8)(1 - cos 28) Positive solution of this quadratic for angle between 0" and 90°,
tan8=1,
.'. 8 = 45" Ans. (iii)
18. sin 38 may be written as a compound angle,
sin 38 = sin (28 8) +
Using the compound angle formula,
1 2 sin2 8
- ---- x 2 sin 8 = + +
sin (A B) = sin A cos B cos A sin B
sin2 8 sm 8
Let A = 28, and B = 8, then,
=2 Ans. (iii) + +
sin (20 8) = sin 28 cos 8 cos 20 sin 8
Substitute for sin 28 and cos 28 from the identities,
2tane sin 28 = 2 sin 8 cos 8
17. sin 28 =
1 +tan2e and cos 28 = 1 - 2 sin2 8
Simplifying right hand side of equation: thus,
+
sin (28 8) = sin 28 cos 8 + cos 28 sin 8
=(2sin8cos8) x cos8+(1 - 2sin 2 8) x sin8
= 2sin0cos28 + sin8 - 2sin 3 8
= 2 sin 8 cos 8 = sin 28 = left hand side of equation. Ans. (i)
I!,
/ /

1 Substitute for cos2 8 from the identity,


1- tan 2 8 cos2 8 = 1 - sin2 8
cos 28 =
1 +tan 2 8 thus,

Simplifying right hand side of equation:


sin (28 + 0) = 2 sin 8 c052 8 + sin 8 - 2 51n3 8
= 2 sin 8(1 - 51n2 8) + sin 8 - 2 51n3 8
= 2 sin 8 - 2 51n3 8 + sin 8 - 2 51n3 8
sin 38 = 3 sin 8 - 4 sin3 8 Ans.

cos28 - sin2 0 cos2 8


X ---- - cos28 - sin2 8
cos2 8 1
= cos 28 = left hand side of equation. Ans. (ii)
SOLUTIONS TO TEST EXAMPLES 9 353

SOLUTIONS TO TEST EXAMPLES 9

24
sin oc = - = 0.75
32
By cosine rule, working in cm:
+
(AD12 (cD)~- ( A C ) ~
COS ADC =
2 x (AD) x (CD)
each obtuse angle = 2 x 48'35' = 97"10'
each acute angle = 180" - 97"101 = 82'50'
} Ans. (i)
-
lo2+ 62 - 142
2 x 1 0 ~ 6 length of short diagonal = BE = 32 x cos48O35'
= -0.5
= 21.16
:. Angle ADC = 180" - 60" = 120"
.'. short diagonal = 21.6 x 2 = 42.32 mm Ans. (ii)
Obtuse angles are each 120"
Acute angles = 180" - 120" = 60"
] Ans. (i) Area = product of diagonals
-
48 x 42.32
( B D ) ~= AD)^ + (AB)~- 2 x (AD) x (AB) x cos BAD 2
= 1016 mm2 Ans. (iii)

short diagonal = 8.718 cm = 87.18 mm Ans. (ii)


Angle CDE = 60"
Perpendicular height CE = 6 x sin 60"
= 5-196 cm = 51.96 mm Ans. (iii)
Area = base x perpendicular height
= 10 x 5.196
= 51.96 cm2 or 5196 mm2 Ans. (iv)
354 REED'S MATHEMATICS FOR ENGINEERS

Refening to sketch, working in cm:


- S O L U T I O N S TO T E S T E X A M P L E S 9
2m
3.55

IIPI~'
\ \
Mean length = $ ( l o + 6) = 8 cm
Area = mean length x perp. height
~ 8 x 5
= 40 cm2 Ans. (i)
D
2
Each half area is to be 20 cm , let dividing line EF be at x from CD BY cosine rule,
+
( B D ) ~= ( B C ) ~ (cD)~- 2 x (BC) x (CD) x cos BCD
=22+1.52-2x2x 1 ~ 5 ~ ~ 0 ~ 1 1 7 ~ 1 7 '

By similar triangles,
+
= 4 2.25 2.75 +
~D=&=3m

Semi-sum of sides of triangle BCD

Area of BCD = Js(s - a)(s - b)(s - c)


= 43.25 x 1.25 x 1.75 x 0.25

Alternatively the area could be found by k a b sin C).


Mean length of half area FECD Semi-sum of sides of triangle ABD
= i ( 3 + 3.5 + 1) = 3.75 m
+
= ~ ( C D EF)
Area ABD = d3.75 x 0.75 x 0.25 x 2.75
=$(6+6+0.8~)
= 1.391 m2
+
=6 0.4~
Area of quadrilateral = sum of areas of the two triangles
Area = mean length x perp. height
= 1.333 + 1.391
+
20 = (6 0 . 4 ~ x) x
= 2.724 m2 Ans.
20 = 6x + 0.42
0.42 + 6x - 20 = 0
?+15x-50=0
Solving this quadratic, x = 2.8 1

.'. Dividing line should be at 2.81 cm = 28.1 mm from CD


Ans. (ii)
356 REED'S MATHEMATICS FOR ENGINEERS SOLUTIONS TO TEST EXAMPLES 9 357

Let x mm = thickness of piece cut off. The piece cut off is a


trapezium, consisting of a rectangle 80 x x and two equal triangles;
the height of each triangle being x, then the side perpendicular to
this, marked y in the sketch, is x tan 30" = 0.5774x, hence,

Area of piece cut off,

= (80 x x) + (2 x + x 0.5774~x x)
= 80x + 0.57742 . . . . . . . . . (i)

Area of hexagon = 6 x 0.433 side2 = 2.598 side2 Radius = A 0 = 60 mm

Area of piece cut off = 0.1 x 2.598 x 802 Being an equilateral triangle, all angles are 60"
= 1663 mm2 ... . . . (ii) Angle at centre AOC is twice the angle at the apex.
.'. 80x + 0.57742 = 1663
0.57742 + 80x - 1663 = 0 :. AOC = 120°, and AOD = 60"
+
or, 2 138.6~ - 2880 = 0 AD = A 0 x sin 60"
Solving this quadratic, x = 18.35 mm Ans. = 60 x 0.866 = 51.96 mm
length of sides = AC = 2 x AD
= 103.92 mm Ans. (i)
6. Considering one of the eight constituent triangles, OD = A 0 x cos 60"
= 60 x 0.5 = 30 mm
Apex angle = 360 t 8 = 45"
BD=BO+OD
Each base angle = ;(I80 - 45) = 67.5"
Perpendicular height = 15 x tan 67"301
+
= 60 30 = 90 mm
Area = (base x perp. height)
= 36.21 mm
Area of each triangle = +(base x perp. height)
;
= x 103.92 x 90

+
= x 30 x 36.21 = 4676 mm2 Ans. (ii)
2
Area of octagon = 8 x x 30 x 36.21 (or formula, Area = 0.433 x side could be used)
= 4346 mm2
Area of hole = 0.7854 x 502 8. Area of collar = 0.7854 (d - d 2)
2
= 1963 mm +
= 0.7854(D d)(D - d)
Area of plate = 4346 - 1963 +
= 0.7854(755 415)(755 - 415)
= 2383 mm2 Ans. = 0.7854 x 1170 x 340
R E E D ' S MATHEMATICS FOR E N G I N E E R S S O L U T I O N S TO T E S T E X A M P L E S 9

Effective area = 0.7 x 0.7854 x 1170 x 340 mm2 7C w e d surface area of s p h e ~


= nd x d

) ~ 2 = lo6 1rm2
1 m2 = ( 1 0 ~ mm zd2
surface area of hemisphere = -
-,
:. Area = 0-7 x 0.7854 x 1170 x 340 x 10-6 2

= 0.2187 m2 Ans. (i) 4F nd'


Surface area of circular base =-4
F nd2 nd 2

At a pressure of 2000 k ~ / m ' :


fii*
% Total surface area = -2 +-4

Total force = 0.2187 x 2000


= 437.4 kN Ans. (ii)

Working in cm:
5 cm or 50 mm dia. Ans.

BY crossed chords,

~ edtmm = depth of indentation = CO


DO = ball diameter - CO = 10 - d
A 0 = BO = I2 surface diameter = 2.5 mm

9
sin oc = - = 0.9,
10
.'. a = 64"9' d 2 - 10d + 6.25 = 0
0=2oc= 128'18' Solving this quadratic, d = 9.33 or 0.67
sin 128' 18' = sin (180" - 128"18') = sin 5 1'42' = :. depth = 0.67 mrn Ans.
128.3
---- = 2.239 rad
57.3
r'
Area of segment =- [O - sin 01
2

= 50 x 1.4542
= 72.71 cm2 Ans.
R E E D ' S MATHEMATICS FOR E N G I N E E R S

Curved surface area of indentation is equal to the curved surface


area of a slice of same depth off a circumscribing cylinder of same
diameter as the ball,
= 71 x ball diameter x depth of indent
/! I I
I \
= 21 so5 mm2 Ans. (ii)
// "tm i

:. Curved surface area = 183.8 x 271 x 125


= 1.444 x 105mm2 Ans.
+
Slant height = J 1 2 5 ~ 752 = 145.8 mm

this is the radius of the sector.


Arc of sector = circumference of base of cone

arc
Angle of sector = -
radius
471.3
I Diameter of bar = difference in outer and inner radii

- - 3.232 rad
145.8
Circumference of bar = IT x 100 mm
Position of centroid from centre axis o o = 220 + 50 = 270 mm
By Theorem of Pappus,
.'. dimensions of sector are: Area swept out = length of line x distance centroid moves
.'. Curved surface area = IT x 100 x 271 x 270
Radius = 145.8 mm
Angle at centre = 185.2"
) Ans.
= 5.331 x lo5 mm2 Ans.
362

15. Areas of similar figures vary as the square of their Corresponding


,.,*1~"'" ,,,,8
.~*',,..wll~, ' dimensions.
.'=,,#..':,
.#*,...
'
,,,,,'
,I;;[:;'

.;fl:;;::",,:m;;;;;
,,,, Ratio of corresponding dimensions = 125 : 175 Total area = 2 X 943.75
r4'"
I .. ,*",
s:4::.+.'"'"
'"' '
::#

"'
, Dividing each by 125, = 1887.5 m2 Ans.
.,* ,,*,.#, ",, '1.'" " 8

"'JHII:'::
e,,; ii Ratio of dimensions = 1 : 1.4
I&.-
.
...:::;I@;
,.I
,.., ,'
Ratio of areas = l 2 : 1.42
;Z,".n , ',, I """
= l : 1.96
,dq;::;:::##:
:~:11,11,1'1
;:;3 , ,." ,,.,- m = 100 : 196
.,@" ",,,,# *, , """"" ' '
i/i :!!/
#,,.,,-, " .'. Area of larger triangle is 96% greater than smaller Ans.
m,,e,,,,,, *.*#!#8d,Ub 1

",**",+'"'sY*r
,,,,j/~
,,,,,,'1
!,I

.*-@
.-;N~C,,.,-,,,
. b8' 16. SEMI- SIMPSON'S PRODUCTS
*" ,.L.l,#
,;,;;;,l,,ll,l ORDINATES MULTIPLIERS
0.1
3-0
5-85

The plotted curves are shown @. 364).


Finding area between curves by Simpson's rule:

4 30.0
2 12.6 ORDINATES SIMPSON'S
BETWEEN CURVES MULTIPLIERS PRODUCTS
z
4 15.0
1 - 0.5
sum = 226.5

Number of ordinates = 13
:. Number of spaces = 12
Common interval = length t no. of spaces
= 150 t 12
= 12.5 m
Common interval between ordinates = 1
Area = sum of products x 4 common interval Area = sum of products x f common interval
=92xixl
= 30$ units2 Ans.
SOL UTIONS T O TEST E X A M P L E S

Sum of mid-ordinates = 82.5 mm


Mean height = 82.5 t 10 = 8.25 mm Ans. (i)
Mean effective pressure = 8.25 x 160
Working in cm:
= 1320 kN/m2 Ans. (ii)
Area of top flange = 10 x 1.2 = 12 cm2
I Area of bottom flange = 14 x 1.4 = 19.6 cm2
Area of centre web = 12.4 x 1 = 12.4 cm2
Total area = 44.0 cm2

I Volume of 1 m length
= area x length
= 44 x 100 = 4400 cm3
Mass (m) = volume (V) x density ( p )
= 4400 x 7.86
= 3.458 x 1 0 ~ ~
= 34.58 kg/m run. Ans.

2. Working in m:
Volume of shaft body = area of end x length
= 0.7854 (0.4~- 0.2~)x 6
= 0.5654 m3
366 R E E D ' S MATHEMATICS FOR E N G I N E E R S S O L U T I O N S TO T E S T E X A M P L E S 10

Volume of two couplings


= 0-7854 ( 0 . 7 6 ~- 0 . 4 ~ x) 0.075 x 2 circumference of cone base = arc of sector
= 0.0492 m3
Total volume = 0.5654 + 0.0492 = 0.6146 m3
m = Vp d = $ x 2 x 180
= 0.6146 x 7.86 :. diameter = 240 mm Ans. (ii)
Mass = 4.83 1 t Ans. Slant height of cone = radius of sector = 180 mm
Radius of cone base = 120 mm
Perp. height of cone = J1802 - 1202
= 134.2 mm Ans. (iii)
Volume of cone = f x area of base x perp. ht.
=ixnrZxh
= $x n x 1202 x 134.2
= 2.024 x lo6 mm3

1 litre = lo6 mm3


.'. Capacity = 2.024 1 Ans. (iv)

1 n
5. Volume of hemisphere = - x - d 3
Vol. of cylinder : Vol. of sphere : Vol. of cone 2 6
n
12 x 63 cm3
=-

1 n
Volume of cone = - x - d 2 x h
71 3 4
dividing throughout by - d 3
12
= 3 : 2 : 1 Ans. n
+ 1271
Total Volume = - x 63 - x 62 x 5
12
= - x 62(6 + 5)
4. Working in m: 71
12
240
Area of sector = - x area of full circle
360
=$x n ~ 0 . 1 8 ~ m ~
Mass of material = $ x n x 0.182 x 6.5 Mass = 2 x 36 x 11 x 8.4
12
=O-4412kg Ans. (i) = 871 g Ans.
REED'S MATHEMATICS FOR ENGINEERS S O L U T I O N S T O T E S T E X A M P L E S 10

3000 Vol. of cylindrical hole = -d21


71
Volume of lead in hollow sphere = -cm3
11.4 4
71
= - x 2.42 x (2 x 2.25) = 20.36 cm3
4
Let D = outside diameter, in cm, then inside diameter = (D - 71
Volume of segment = - h2(3d - 2h)
6

Volume of the two end segments


71
=2x - x 0.3~(3x 5.1 - 2 x 0.3)
6
d - ( D ~- 6D2+ 1 2 0 - 8) = 502.5
6 d - 1 2 0 + 8 = 502.5
6 d - 120 - 494-5 = 0
or, d - 2 0 - 82.41 = 0 Net vol. = vols. (sphere - cyl. hole - 2 end segments)
= 69.465 - 20.36 - 1.386
Solving this quadratic equation, = 47.72 cm3 Ans. (i)
D = 10.135 cm or 101.35 mrn Ans. 3
Density = 7.86 g/cm
Mass = volume x density
= 47.72 x 7.86
Mass=375.1 g Ans. (ii)

radius of sphere = x 5.1 = 2.55 cm


radius of hole = x 2.4 = 1.2 cm
half depth of hole = x cm
x = 4 2 . ~-
5~1.22= J50625= 2.25 cm
Thickness of spherical segments at top and bottom of hole
= h cm
h = i d - x = 2.55 - 2.25 = 0.3 cm
71
Vol. of sphere = - d 3
6
370 R E E D ' S M A THEM A T I C S F O R E N G I N E E R S
-
By similar triangles, from sketch, Distance of Its centroid from centre axis
=ix24+8=16mm

height of hole = 60 - 15 = 45 mm

Distance of its centroid from centre axis


Volume of complete cone
=fx6+8=10rnm
= f x area of base x perp. height
Netvolume=540 x 271 x 16- 135 x 2 n x 10
i
= x 0.7854 x 642 x 60 = 64 340 mm3
= 2~(8640
- 1350)

Volume of top cone cut off


= $ x 0.7854 x 1 6 x~ 15 = 1006 mm3
I = 45 810 mm3 or 45.8 1 cm3 (as before)

Area of hexagonal base = six equilateral triangles


= 6 x 0.433 x side2
Volume of bored hole, this is a frustum of a cone,
= 2.598 x 2.52
= &"h(L? +
~d d 2 ) +
= x n x 45(2g2 28 x 16+ + 1 6 ~=) 17 530 mm2 Volume of pyramid = f x area of base x perp. ht.
3
= x 2.598 x 2-s2 x 6

Net volume
I =32.48cm3 Ans. (i)

= 45 804 mm3 or 45.804 cm3 Ans. (i) Volumes of similar objects vary as the cube of their corresponding
m=Vp dimensions, therefore,
= 45.804 x 8.4 vol. of top pyramid cut off - -
33 -1
- - -
Mass = 384.8 g Ans. (ii) vol. of whole pyramid 63 8

I .'. vol. of top pyramid cut off = $ of whole pyramid


Alternatively, the volume can be obtained by the Theorem of
Pappus. Volume remaining after boring is equal to the volume swept Volume of frustum = the remaining of the whole pyramid
out when triangular area ABD is swept through one complete
= x 32.48
revolution about the centre axis of the cone. This can be found by
the difference between the volumes swept out by triangles ACD and = 28.42 cm3 Ans. (ii)
BCD, the volumes in each case being area x distance moved by
centroid (see sketch).
10. Volumes of similar objects vary as the cube of their corresponding
4
Area of ACD = x 24 x 45 = 540 mm2 dimensions, therefore volume varies as depth3.
372 R E E D ' S MATHEMATICS FOR ENGINEERS SOLUTIONS TO TEST EXAMPLES 10

Let V = volume when depth is 7 cm comparing similar cones (after inversion):


V H~
v - 73 0.875 V - h23
200-53
200 x 73 h2 = 500 x 3J0875= 478.3 mm Ans.
V = = 548.8 cm3
53
Additional volume required = 548.8 - 200 12. Ratio of surface areas = 1.5 :1
3
= 348.8 cm i.e. 348.8 ml Ans. Ratio of diameters = 1/15 : f i
= 1.225 : 1
Ratio of volumes = 1 .2253 : l 3

Let v = volume of smaller sphere, in cm3,


+
then volume of larger sphere = (v 10) cm3

v = 11.94 cm3 Ans. (i)


7 1 3
Volumes of similar objects vary as the cube of their corresponding volume = - d
6
dimensions. Comparing similar cones (before inversion):
diameter =
Let V = volume of whole cone
H = height of whole cone = 2.836 cm Ans. (ii)
v = volume of conical empty space
h, = height of conical empty space 13. Working in decimetres,

SECTIONAL SIMPSON'S
AREAS MULTIPLIERS PRODUCTS
0.7854 x 3.95* 1 0.7854 x 15.6
0.7854 x 4.772 4 0.7854 x 91.0
0.7854 x 5.00~ 2 0.7854 x 50.0
thus, the volume of the empty space is one-eighth of the whole 0.7854 x 4.772 4 0.7854 x 91.0
0.7854 x 3.95i2 1 0.7854 x 15.6
volume, and the volume of the water is seven-eighths ( = 0.875) of
the whole volume. sum = 0.7854 x 263.2
R E E D ' S MATHEMATICS FOR E N G I N E E R S

There are 5 ordinates therefore 4 spaces


Common interval

= 5.81 t 4
Volume = sum of products x f common interval

-
0.7854 x 263.2 x 5.81
-
3x4
= 1 0 0 ~ 1 d m 3 = 1 0 0 ~ 1 1Ans.

Finding plotting points, y = 5 + 4x - 2,


x - 1 0 1 2 3 4 5

The graph of these values is shown. Sweeping the area bounded by


this graph through one revolution, the y ordinates of the graph
become the radii at regular intervals along the length of the solid
generated. Putting the cross-sectional areas of the solid through
Simpson's rule to find the volume:

CROSS-SECT.
RADII AREAS SIMPSON'S
Y IT3 MULTIPLIERS PRODUCTS
0 0 1 0
5 IT x 25 4 IT x 100
8 IT x 64 2 IT x 128
9 IT x 81 4 IT x 324
Half-breadth of water level = 0.18 x tan 40" = 0.15 1 m
Area of cross-section = ;(breadth x perp. ht.)
= 0.151 x 0.18 m2
Volume flow = area x velocity
Common interval between ordinates = 1
= 0.151 x 0.18 x 0.5 x 3600
Volume = IT x 780 x fx1
= 816.9 units3 Ans. =48.94m3/h Ans.
376 R E E D ' S MATHEMATICS FOR E N G I N E E R S S O L U T I O N S TO T E S T E X A M P L E S

4 cancel from every term,

=1~811cm=18~11mmfrombase. Ans.

considering the plate as a rectangle 300 mm by 375 mm with a


bangle 225 mm by 300 mm cut off one comer (as in Fig. 123),

h a of rectangle = 30 x 37.5 = 1 125 cm2


Distance of its centroid from 375 mm side = 15 cm
Distance of its centroid from 300 mm side = 18.75 cm
h a of triangle =g22.5 x 30) = 337.5 cm2
Distance of its centroid from 375 mm side
= 30 - f x 22.5 = 22.5 cm
Distance of its centroid from 300 mm side
= 37.5 - f x 30 = 27.5 cm
(centroid of triangle is at f height from its base)
h h+4
By similar triangles, - = -
6 8
I
I
Moments about 375 mm side:
C moments of areas
i=
C areas
i
c.g. of a cone is at height from base.
Volume of whole cone = f x IT x 42 x 16 cm3
c.g. from base = of 16 = 4 cm
Volume of top cone cut off = f x IT x 32 x 12 cm3
I Moments about 300 mm side:

c.g. frombase = 4 + i of 12 = 7 cm
Moments about base:
y = C moments of volumes
C volumes
- moment of whole cone - moment of top cone
-
volume of whole cone - volume of top cone
R E E D ' S MATHEMATICS FOR E N G I N E E R S

Taking moments about end of shank (p. 377):


X moments of volumes
c.g. =
C volumes NS TO TEST EXAMPLE
All parts being cylindrical of volume 0.7854 x d 2 x I, the
multiplier 0.7854 is common to all terms and therefore cancels

c.g. =
+ +
(62 x 4 x 25) (42 x 23 x 11.5) (62 - 42) x 4 x 13.1 2+3?-9x+4
+ +
(62 x 4) (42 x 23) (62 - 42) x 4 3?+6x-9 Ans. (a).
3600 + 4232 + 1048 8880

= 15cmor 150mm. ~ns. is, .


i.e. y =

dy - 2~?
-
dx-
+ 1 4 ~ +- 1~
- 14
- @ + - + 3I Ans. (b)
x

(4 Y=

+ 2 sinx
5 cosx - 7 cosx
3--5 sinx + 7 sinx + 2 cosx Ans. (d).
dx-
380 REED'S MATHEMATICS FOR ENGINEERS SOLUTIONS TO TEST EXAMPLES 1 1 381

y=2 + 3x - 7 (see sketch)


dy=2,+3
dx
when x = 3,

:=6+3
i.e. gradient = 9

when x = -2

dy=-4+3
dx ;. At any point on the curve,
i.e. gradient = - 1 Ans.
1
gradient = 4 - -
x2
3. (a) s = 20t - 5t2 +4
ds When the gradient is zero,
v=-=20- lot ... ... ...
dt
when t = 1 ,
1
4--=0
v = 2 0 - 10 x2
= 10 m/s Ans. (a)
(b) when v = 0 ,
20 - lot = 0
t=-
20 .'. x = fi (see above sketch)
10
.'. t = 2 s Ans. (b) Gradient is zero when x = and x = - i Ans.

(c) From equation (i), 5. y = 3 2 - x + 2 (see sketch, p. 382)


v = 20 - lot dy = 6x - 1
Gradient = -
dx
dv
a=-
dt When the gradient is -7,
.'. a = -10 m/s 2 . Ans. (c) 6 ~ - 1= - 7
6 ~ = -6
.: x = - 1
Substituting x = - 1 in equation (i),
d ~=4 - x - ~
Gradient = -
dx
382 RE E D ' S M A T H E M A T I C S F O R E N G I N E E R S

The gradient is -7 at the point x = - 1,y = 6. Ans.


(ii)

For maximum or minimum values,

when t = 2, i.e. 12 + 6x - 6 1 = 0
2+x-1=0
1-x-2=0
Solve for x:

when angular velocity = 0,


+
(x - 2)(x 1) = 0
. . x = 2 or x = - 1
Substituting in equation (ii):
when x = 2,

i.e. shaft comes to rest after 16 s. Ans. 9


dx2
= 6 - 24 (i.e. negative)

when x = -1,

9
dx
= 6 + 12 (positive)
2

. A maximum value occurs when x = 2.


Substituting in equation (i):
Maximum value of y = 24 + 12 - 16
REED'S MATHEMATICS FOR ENGINEERS T EXAMPLES

wlx
M=--- wX2
2 2

Differentiating with respect to x:


Ans. (c)

d 2 ~
-- - -w (i.e. negative)
dr2
For maximum or minimum value,
x3
f(x) =--2X2 +3x+ 1 ......... (i)
11. 3
f'(x) = 1- 4 x + 3 . . . . . . . . . . . . (ii)
From equation (i): fU(x)=2x-4 . . . . . . . . . . . . (iii)
f '(x) = 0 for a maximum or a minimum
0 = 1 - 4x + 3 from (ii)
= (x - 1)(x - 3)
:. x = 1 or x = 3
when x = 1 in (iii)
.'. Maximum bending moment occurs when fl'(x) = -2 i.e. a maximum
when x = 3 in (iii)
f "(x) = 2 i.e. a minimum
when x = 3 in (i)
i.e. At the centre of the beam. Ans.
f(x)=9-18+9+1
= 1 for a minimum Ans.
lo. (a)

= 0.5 x 10-lo x 6 x 100'


- -0.8 1.4 Ans. (a) = 0.5 x lo-''x 6 x 10"
5 f i 3
D
:. Power/Volt = 3 units at 100 V Ans. (b)

13. Let each of two sides be x long


dz
--
anun-l 1
Ans. (b) Remaining two sides are -2 - x long
du C
386 R E E D ' S M A T H E M A T I C S FOR E N G I N E E R S

Length = 2 - - x
G > + 2x
. . . . . . . . . . . . . . . . . . (ii)
= 2 m/s i.e. velocity at t = 5 s
f"(x) = 0.4 m/s2 i.e. (constant) acceleration
100 = 0.28 + 10.4 i.e. x = 100 in (i)
+
500 = t2 52
1 t = 21.17 s
fl(A) = - - 2x . . . . . . . . . (ii)
2 fl(x) = 0.4 x 21.17 i.e. t = 21.17 in (ii)
= 8.468 m/s i.e. velocity
f "(A) = -2 i.e. a maximum
fl'(x) = 0.4 m/s2 i.e. acceleration
I
0 = - - 2x from (ii)
2
1
x=- Ans.
4 16. e = 2.1 - 3.2t + 4.83
fl(t) = 9.6t - 3.2
14. (a) f(8) =cosO-In8 = 9.6 x 1.5 - 3.2 a t t = 1.5 s

1
f '(0) = - sin 8 - -
= 11.2 rad/s angular velocity
f "(t) = 9.6 rad/s2 (constant) angular acceleration
) Ans.
6
1
f"(0) = - - cos6 Ans. (a)
e2
17. f(x)=x5-5x . . . . . . . . . 6)
f '(x) = 5x4 - 5 . . . . . . . . . (ii)
0 = 5x4 - 5 for maximum/minimum
f '(t) = 2at + -2t x4 = 1
2 x = 1 o r x = -1
f "(t) = 2a - -2
t x = 1 in (i) givesf (x) = -4
x = -1 in (i) gives f (x) = 4

(
= 2 a- -
ti>
Ans. (b)
f " ( ~ )= 2ox3 . . . . . . . . . (iii)
= 20 i.e. a minimum when x = 1
= -20 i.e. a maximum when x = -1
)
Ans.
f (x) = 5eX
:. x = 1,f(x) = -4, minimum
Ans.
fU(x) = 5ex Ans. (c) and x = - 1,f (x) = 4, maximum
388 REED'S MATHEMATICS FOR ENGINEERS

I
x5 2
2 I.(,) ( x 4 + 2 - 8 x + 5 ) d x = - + -5- 4 2 3+ 5 x + C Ans. (a)
dx2
= 4 i.e. positive, so minimum

0 = 4x + 2 for minimum S
. . x = - l .2
y=d+bx+c
The expression above is re-arranged as:
1= + 2x + c at minimum
I = 2(- ); +2(-4) + c
2 J(4x-2+x-l)cix= -4x-I + l n x + C
... c = 32 4
= --+lnx+C Ans. (b)
a=b=2andc=i Ans. X

(c) f(2x-3~?-l)dx=[2-$-~1:
1
=[9-27-31-[I-1-11
= [-211-
[-I]
= -21 1 +
= -20 Ans. (c)

when y = 5 , x = 2.

. . . . . . . . . . . (ii)
390 REED'S MATHEMATICS FOR ENGINEERS S O L U T I O N S TO T E S T E X A M P L E S 1 2 39 1
From (i) and (ii): Y

8
x3 X2
y=-+-- 2x+4; Ans.
3 2

(3cosx-2sinx+4)dx= 3sinx+2cosx+4x+C

Ans. (a)

X2
(b) l ( 4 c o s x - c o s x + x ) d x = 4 s i n x - s i n x + -2+ C
Ans. (b) .'. Area = 23 units2 Ans.

f cos x dx = [sin x t
= [sin ]: - [sin 01

=1 Ans. (c)

:. Area =
1: (x3 - 4 2 + 3x) dx

The law of expansion ispV = C . . . . . . . . . . . . (1)


C
1.e. p =-
v
The standard equation
rb
1 Note: the negative sign indicates that the area is below the x-axis. Area = J y dx
This is confitmed by the sketch. a
392 R E E D ' S MATHEMATICS FOR E N G I N E E R S
- S O L U T I O N S TO T E S T E X A M P L E S 12

must be amended to suit the symbols in this problem: whent=2, ~=3(2)~


v2
Area under curve = p dY
v,

. . Average velocity = -
12 +27
2 ,. .
= 19.5 m/s Ans. (b)
whentz2.5, ~=3(2.5)~
i.e. Instantaneous velocity = 18-75 m/s Ans. (c)

. . . . . . (i)
. (ii) Substitute s = 10, t = 0 in equation (i),
10=0+0+O+C
... C = 10 . . . . . . . . . . . . . . . (ii)
Note: In V2 - In Vl is numerically equal to ln-
From equations (i) and (ii),
From equations (i) and (ii), s=t3+4t2+12t+10
when t = 10,
v2
Area under curve = pV ln - Ans.
s = 1000 + 400 + 120 + 10
Vl Displacement = 1530 m hs.

. . (i)

Substitute v = 0, t = 0 in equation (i),

O=O+C
:. C = 0
Hence v = 3 8 Ans. (a)
394 REED'S MATHEMATICS FOR ENGINEERS S O L U T I O N S T O T E S T E X A M P L E S 12 395

Volume of solid of revolution = x


1: g d x (see previous sketch) 10. (a)
I (x+ l)(x + 2 ) dt- =
I(2+ 3x +2)
2 32
dx

= -3 + y + h + C Ans. (a)

I(:-$)dx=lnx+-+c 1 Ans. (b)


X

= 56.55 units3 Ans.


=a + 2b[(2 + 2) - (++ I)]
9. At the points where the curve intersects the x-axis, x has zero value. = $ ( a +2b) Ans. (c)

hence, x = 0 or x = 1.
:. The curve intersects the x-axis at x = 0 and x = 1. I = 3eX +C Ans. (d)

(0.02 x 512) + 80 - (0.02 x 216) - 60


= 0.02 x 296 + 20 = 5.92 + 20
2
= 25.92 units Ans.

= 0.105 units3 Ans.


396 REED'S MATHEMATICS F O R EN G IN EE RS S O L U T I O N S TO TEST E X A M P L E S 12 397

13. (a) / (sine-;):=;/ sindo-;/ $

= (- cos 71) - (- COS 0)


=l+l = nb2[(a - $)]
=2 Ans. (b)

(c) & ( 5 c o s x + 3 s i n x - x ) h = 5sinx-3cosx-- 3


4
Volume V = - ab2n Ans.
3

when t = O v=u so c = l n u
lnv - lnu = -k2
398 SOLUTIONS TO TEST EXAMPLES 12

so
RE E D ' S M A T H E M A T I C S F O R E N G I N E E R S

hea =
I
Y dx = lo,+,
I dx (see sketch) but s = 0 when t = 0
.'. k2 = 0

= b[ln(l + a) - ln(0 + a)]


a = - 28
v=5-%t
s = st -lt4
6
} at t seconds

= b[ln(l + a) - lna]
a = -18 m/s
1+ a
= bln-
a
= 21n2
s= 1; m

= 1.385 units2 Ans. .'. Particle is 1; m along OX moving in the direction XO at


13 m/s, decelerating at 18 m/s2. Ans.

17. (a)
=3lnx+C Ans. (a)

4
x
= -+
4a
C Ans. (b)

(c) J ( 4 2 + 3 2 + 2 z + l ) d z = z 4 + 2 + $ + z + c Ans. (c)

(d) ~ ( 2 c o s 6 - 5 s i n B ) d 6 = 2 s i n 6 + 5 c o ~ ~ +Ans.
~ (d)

= -+t3 + kl
but v = 5 when t = 0
. k, = 5
SELECTION OF EXAMINATION
QUESTIONS

1. Determine the second differential coefficients of:

(b) y = lnx + sinx

2. (a) Prove that x can have any value to satisfy the equation:

3. Water flows along a horizontal pipe of 100 mm internal diameter,


the depth of water in the pipe is 75 mm. Calculate the hydraulic
mean depth d of the water from the formula:
cross-sectional area of water stream
perimeter of wetted pipe surface

4. Given p = 120/ C: where p and V represent the pressure ~ m *and)


volume (m3) of a given mass of gas at constant temperature,
calculate by integration the work done when the gas expands from
V = 0.5 m3 to V = 6.5 m3

5. The volumetric analysis of a certain mixture is 68% water, 18%


alcohol and the remainder solids. Calculate (a) how much water
should be added to each 1 of this mixture to reduce the alcohol
content to 15%, (b) the percentage analysis of the new mixture.

6. Differentiate the following with respect to x.


402 R E E D ' S MATHEMATICS FOR E N G I N E E R S SELECTION O F EXAMINATION Q U E S T I O N S 403
cx3 b 12. Find the values of angle 6 between 0" and 360" which will satisfy
(a) Rgx - -
64L @) &f7 the equation:

(c) r2
Integrate the following.

+I
( P 6- 1) dk with respect to k, and
tan9 = cos6

13. (a) Determine the second differential coefficients of:

(d) I [(2v2- 4)v] dv with respect to v.


(i) 4cosx
4
+ lnx

(ii) 3 2 +-+6x-2
x2
+
7. Sketch the graph of y = 11 7x - 2\" and use differential calculus + +
(b) The expression T = 1700 16000/2 0.62 is used to
to determine the maximum value of y. calculate the hoop tensile strength Tof certain rotating discs. If x is
the disc radius, use calculus to determine the value of x for which T
8. A component consists of a cone of base diameter 52.5 mm and is a minimum and the corresponding value of Tat that radius.
perpendicular height 45 mm, standing concentrically on the top of a
solid cylinder 60 mm diameter and 40 mm high. The base of this 14. The following table gives corresponding values of x and y. Plot y
component is firmly fixed on to a flat base-plate. Calculate the total against x and, assuming they are connected by a straight line law,
surface area of the component exposed above the base-plate. find the values of the constants and state the law.

9. (a) The distance s metres travelled by a body in time t seconds is


given by:
s = t3 - 5.5t2 - 4t + 68.5 15. A sector of a circle is cut out of a piece of sheet metal, the radius of
Determine the time, distance travelled and acceleration of the body the sector being 102 mm and the angle at the centre 150". The
when its velocity becomes zero. sector is then rolled into the shape of a cone. Find the surface area
@) Integrate with respect to x:
of the metal used, the diameter of the base of the cone and its
perpendicular height.
(i) 3 ,/5 - 3 2
2
(ii) - 2
x3
+ 16. Find the values of x and y in the simultaneous linear equations:
+
10y 4b = -2r
10. A roof has the form of a square pyramid, the base is 24 m square 3a+x=y
and the length of the ridge from corner of base to apex is 18 m. Find given that the same values of a and b in the above also satisfy the
(i) the perpendicular height from base to apex, (ii) the total surface equations:
area of the roof.

+ +
11. (i) Multiply p - 3 2 2x 4 by x2 - 3x +5
(ii) ~ i v i d e6x5 +x 4 + p- 4 2 - 7 x + 4
by 3x3 -2 + 3 x - 4 17. The rim of a flywheel is elliptical in cross-section with the major
(iii) Multiply ( 2 ~ ~ by b )( ~3 ~ b ~ ) ~ axis in the radial direction. The major and minor axes of the rim
(iv) Divide ( 3 ~ b by ~ )(ab312
~ section measures 300 and 200 mm respectively, and the overall
404 R E E D ' S M A THEM A T I C S F O R E N G I N E ER S
I '
SELECTION OF EXAMINATION QUESTIONS 405

diameter of the wheel is 3 -6 m. If the material is cast iron of density


7.21 x lo3 kg/m3, find the mass of the rim.
I 23. Determine the second differential coefficient of.
1
(a) y=.?-5x+7--
X
18. Determine the gradient of a tangent to any point in the curve @) y = 2 c o s x - 3 s i n x
2 2
Y=4-T 24. A block of brass is 80 mm long and has a varying cross-section
throughout its length. Commencing at one end the cross-sectional
and show that there are two points where the gradient is zero. areas, at regular distances apart, are 0, 2.9, 3.8, 4.1, 3.7, 2.7 and
Determine the point on the curve where y is a minimum. 2
1.1 cm respectively. Find the volume by Simpson's rule and the
mass taking the density of brass as 8.4 g/cm3.
19. Factorise completely:
(i) 2.2 - 2 25. On a certain production machine, the cost C of manufacturing N
+
(ii) 7 2 9xy - 1 0 9 +
articles is given by C = a bN where a and b are constants. If it
(iii) 1 4 f l +
2@? - 12xy4 costs £8.50 for 300 articles, and £7.50 for 200 (a) produce an
+
(iv) 3x3 8 2 Y- x# - lo? given that ( x - y) is one of the equation to give the cost in pounds to manufacture any number of
factors. articles, (b) find the cost to make 150, (c) what is the minimum cost
of running the machine?
20. Using integral calculus determine the difference in area bounded by
thecurvesy=102+x+4andy=2/2-1/x-1/2andthex 26. Using integral calculus, determine the area bounded by the curve.
axis between the ordinates x = 1 and x = 5. y = sinx +$cosx
and the values x = 60" and x = 30"
21. (a) Using integral calculus, determine the area bounded by the
curve y = 2 cosx and the x-axis between the ordinates at x = 0
and x = x/4. 27. A piece of wire 1 m long is cut into two pieces and each is bent into
a form of a square.
(b) A curve which passes though the point (x = 1, y = -2) has a
gradient of 3 2 - 4. Determine the equation of the curve. Using differential calculus, determine the least possible sum of the
22. (i) Prove that the solution of a quadratic equation of the form, two areas.
&+bx+c=0 28. A flat mild steel plate of elliptical shape measuring 420 mrn across
can be obtained from, the major axis and 370 mm across the minor axis is 45 mm thick.
There is a groove around the whole of the perimeter, 13 rnm wide
-bf JQ=G by 13 mm deep. Taking the density of steel as 7.86 g/cm3, find the
x=
2a mass of the plate.
(ii) Solve the following quadratic equation by the method of
'completing the square', 29. Two chords, 60 and 80 mm long respectively, are drawn parallel to
each other inside a circle. If the chords are 10 mm apart, find the
2 + 5x = 84 radius of the circle.
(iii) Find the value of x in the following quadratic equation by
30. A railway cutting, in the form of a trapezium, is 10 m wide at the
factorising,
bottom, 15 m wide at the top, and 14 m deep. The cutting sweeps
2X2-3ax-bx=O around on a circular arc, the radius of the arc to the centre of the
406 R E E D ' S M ATHEMATICS FOR E N G I N E E R S S E L E C T I O N OF E X A M I N A T I O N Q U E S T I O N S 407

cutting is 805 m, and the angle subtended is 6". Calculate the


volume of earth removed, in m3, in the making of the cutting. I (ii) -
2ir3 1: .(? - ')dx

3 1 . In a triangle ABC the lengths of the sides AB and AC are 50 and +


38. (i) Prove that sin2 0 cos2 0 = 1
30 mm respectively and the angle BAC is 20". Calculate the length (ii) Find the values of 0 between 0" and 360" which will satisfy the
of the other side, the two remaining angles, and the area. equation:

32. Draw graphs of the following simultaneous equations between the


limits x = -2 and x = +3 and find the values of x and y,
X L y = 1
2y - 3x = 4
1 39. Prove that the radius of a circle inscribed in a triangle is given by:
2 x area of triangle
R=
perimeter of triangle
33. Determine the area contained between the curves I

78 7 i and calculate the radius of the circle inscribed in a triangle of sides


p = -v1.2
,p =-Vl.15 and the ordinates at V = 2 and V = 5. 1 50, 60 and 70 rnm respectively.

34. A body moves s metres in t seconds according to the relationship 40. Determine the value of x for which expression
3
s=t -6?+9t+3

Given that velocity v = dsldt and acceleration a = duldt, calculate is a maximum or minimum, and calculate the corresponding values
of y stating whether it is a maximum or minimum.
(i) the values of t at which the body is stationary
(ii) the values of the acceleration corresponding to the values of t
41. A solid consists of a right circular cone of 100 rnrn base diameter
determined in (i).
mounted concentrically on the flat surface of a hemisphere of equal
35. Find the value of x in the following equation:
diameter, the perpendicular height of the conical part being also
100 mm.
x+2 x - 1 - 7- Calculate the volume of the solid in cm3 and the curved surface area
x-l 3x- 4- 2 in cm2.
36. Calculate the area of the largest isoceles triangular plate with base
If another similar solid was made with all linear dimensions double
angles of 75" that can be cut out of a circular plate having an area of those of the original, what would be its volume and curved surface
804.2 cm2. area?

37. (a) Differentiate the following equations, +


42. Plot the graph of the equation y = .? - 8x 7 between x = 0 and
2 x = 9, using intervals of unity. From the graph solve for x in each of
(i) y = (Ei - Ri ) with respect to i
the following equations:
(ii) y = - [vl-" with respect to u (a) 2 - 7 x + 6 = 0
(b) evaluate the following integrals, (b) 2 - 9 ~ + 8 = 0
Suggested scales: x axis 2 cm = 1
yaxis 2 c m = 5
408 REED'S MATHEMATICS FOR ENGINEERS

43. For a certain ship at deadweight displacement, an expression


I 4
SELECTION OF EXAMINATION Q U E S T I O N S

48. (a) If dy/dx = - cosx - 2 sinx, determine an expression for y in


409

connecting the power of the main engines and the speed of the ship terms of x, given that y = 3 when x = 0.
is given by: @) The area bounded by the curve

P = V(a + bv2)
where, P = power of the engines in kW.
V = speed of the ship in knot. the ordinate x = 4 and the x-axis between 0 and 4 is rotated
once around the x-axis. Use integral calculus to determine the
a and b = constants.
volume of the solid generated.
If the powers are 6024 and 13408 kW when the speeds are 12 and
16 knot respectively, calculate the probable power when the speed is 49. Transpose the terms in the following equation to make K the
14 knot. subject:

r -I 1
44. Obtain the second differential coefficients of:
1 3
(i) y = - - - + 3 2 - ~
3 x3
(ii) y = 3sinx+4cosx
50. (i) If x varies directly as z and inversely as 3,calculate the
percentage change in x when z is increased by 12% and y is
45. A buoy is constructed by welding the base of a hollow cone to the decreased by 20%.
flat base rim of a hollow hemisphere, the common diameter of their (ii) The time for one beat of a pendulum varies as the square root of
bases being 2.44 m. The greatest length of the buoy is 4.27 m. its length. If a pendulum 174.4 mm long makes 105 beats in
Calculate (i) the curved surface area of the cone, (ii) the angle 1
44 s, calculate the length of a similar pendulum to beat exactly
subtended at the centre of the sector of a circle when the cone is once every second.
developed, (iii) the curved surface area of the hemisphere, (iv) the
external volume of the buoy. 51. Solve the following simultaneous equations to give the smallest
values of the angles x and y:
46. Solve for a , b, c and d in the following four simultaneous equations:
3cosx+4siny = 2y/Z- 1.5
5 cosx - &siny = -3.5

52. Show the height of a frustum of a right circular cone is one-third the
height of the complete cone when the volume of the frustum is
19/27 that of the complete cone.

47. A lens is flat on one side and convex on the other side, the convex 53. (a) Determine the area bounded by the curve y = 2 sin 8 and the 8
surface being part of a sphere. The diameter of the flat surface is axis between the ordinates 6 = 7116 rad and 8 = 5x16 rad.
60 mm. The maximum thickness of the lens is 4 rnrn and the (b) Evaluate:
minimum thickness is nil. Calculate the curved surface area of the
410 R E E D ' S MATHEM.

59. (a) Find the angles between 0" and 360" that will satisfy the
(ii) (2- 6x1 dx
1 1 +@: equation:

(b) Find the angles between 0" and 360" that will satisfy the
equation:
54. Express V in terms of v,fi andf, in the following expression where
K =f,/f, and find the values of V whenf, = 12,h = 18 and
60. The resistance R (newton) to motion of a body and its velocity V
(rn/s) is tabulated below:
V 5 10 15 20 25
R 10 25 50 85 130
+
55. Plot the graph y = 2 2x - 8 between the limits x = -4 and Show that these values are connected by the law R = k + cv2, find
x = +2. If the area bounded by this curve and the x-axis is rotated the values of the constants k and c and calculate the probable
through one revolution about the x-axis, calculate the volume resistance to motion when the velocity is 40 m/s.
generated, using Simpson's rule.
61. Transpose the following to express k in terns o f p and and find
56. The total area of metal A needed to make a box of stated volume is the value of k when p = 9.2 and V= 0.42,
given by the expression

62. (a) Calculate the area of a trapezium inscribed in a semi-circle of


where x is the side of the square base. Use calculus to find: radius 10 cm if the length of each non-parallel side of the
(i) the value of x for which A is a minimum. trapezium is equal to the radius of the semi-circle.
(ii) the minimum value of A. @) If this trapezium is rotated through one complete revolution
about an axis passing through the centre of, and perpendicular
57. Three steel ball bearings, each 2 cm diameter, are placed inside a to, its long side, calculate the volume of the solid frustum of a
cylinder 6 cm diameter, equally spaced around the bottom, then cone thus generated.
another ball bearing 4 cm diameter is placed on top of these.
Calculate the volume of water, in ml, required to pour into the 3. (a) Find the values of a, b and w in the simultaneous equations:
cylinder so that the top ball is just submerged.
If all linear dimensions were half of those given above, what would
be the volume of water required?

58. The distance from the centre to a comer of a regular pentagon is (b) Find the values of x and y in the simultaneous equations:
3 cm, find the area of the pentagon. If a piece is sliced off by a
straight cut from one corner at an angle of 30" to a side, find the 1 = 18.5
412 R E E D ' S MATHEMATICS FOR E N G I N E E R S
- SELECTION OF EXAMINATION QUESTIONS 413
1 -tan2e (c) Calculate the height of the mast.
64. (a) Prove, = cos28 - sin2 8
1+tan28 (d) Find the angle of elevation from point W to the top of the mast.
tane 1
+ +
(b) Prove, sin 8 sin 8 cot2 8 --
cos 9 - sin 8 cos2 8 +
71. Draw the graph y = 1 1 between the limits x = -2 and x = +3
and use this to solve the equation 2 - x = 2.
65. Draw the graphs of the following equations between the limits
x = -2 and x = +8 and find the area of the triangle enclosed by the I 72. (a) Prove that the area of a segment of a circle is equal to
f
- (8 - sin 8)
2
where r = radius of circle
8 = angle subtended (rad).
(b) A horizontal pipe is more than half full of water. The breadth of
the water-level is 60 cm and the height from water-level to
66. A hexagon of unit side is to have its area reduced by 20% by cutting
crown of pipe is 15 cm. Calculate (i) the diameter of the pipe,
it parallel to one side. Calculate the thickness of the piece cut off
(ii) the cross-sectional area of the water, (iii) 1 of water
and the distance from the centre of the original hexagon to the
contained over a length of 2 m.
corner of the new face.
73. (a) Find the values of WZ, W2Z, W2/Z and Z3/ w when W = f +b
67. Using Simpson's rule, with eleven ordinates, calculate the area
and z = 9-b
between the curve y = e-2X and the x axis between the limits x = 0
and x = +2 and hence determine the length of the mean ordinate (b) Find the values of x and y in the simultaneous linear equations:
for this section of the curve. 4 x + y = 10

68. The metal from the melting down of a solid cone 60 cm base
diameter and 50 cm slant height is to be cast into the form of a
hollow sphere of 1.5 cm uniform thickness. Calculate the outer and 74. In each of the following formulae, transpose and re-write in terms
inner diameters. of k:
69. Transpose for h in the following expression and use the result to
evaluate h when a = 1.5, c = 15, s = 16 and d = 17

a= '{"i]
2s d - h

70. Three points A, B and C lie on a horizontal ground. C is due North


of A, and B is due East of a point W which lies on the line joining A
and C. Avertical mast stands on point B and the angle of elevation +
75. The equation 7x2 - 28 7 3 = 0 represents the law of a circle. Use
to the top of the mast from A is 20°, AB = 60 m, BC = 32 m, and integral calculus to determine the volume of revolution generated by
AC = 68 m. the middle part of the circle about its x axis between the limits
(a) Prove that the angle at B is a right angle. x = + 1 a n d x = -1.
(b) Find the angles at A and C. Take each unit of x and y as representing 1 cm
--
G -I '-'+1x3
2x4
1 ~ 3 x 5
-x3~ 4 x 6 approx,
2
such an approximation for = 0.1.

7 7 Due to bad weather a ship's speed is &creased by zO% for A sbip,s fuel consumption varies inversely as the calorific of
ship then luns at speed for 10 h. In order to catch a tide directly as the (ship s p e d in hot).' The ship
'peed is then increased for 6 h. Over the 24 h period the fdand
tmms 60 t/&y of fuel of calorific value 40 M J / ~ % when travelling
was found to be unchanged from the normal at 20 lolot. Determine the daily COnSUmptlon w hen using
of
Given that the fuel consumption C is directly
calofific value 42 MJ/kg and steaming at l6 lolot.
proponiona1 to the cube of the ship5 speed v determine me
percentage increase in the speed in the last h.
*
By ploding mo graphs for unit values between x = and = lo On
78. (a) Simplify the expression hesame axis solve the equation
~ 3 n & Z i
5
@) Solve for x in the equation: 101 = 2 ~ - 4 0.5 Ig x = -
2x
79. (a) (i) Determine the square root of 64- 'a8b12
(ii) Determine the cube root of 2 16a-izbo.6 suggested scales x axis 1 unit= 2 cm
y axis 1 unit = 10 cm
for a, b and c in the following simultaneous equations:
a-b+~=3
84. Two chords aR drawn from a common point on the circumference
2a + 8b - 3c = 56 of a circle such that both lje on the same side of the centre Of the
7a + 3b + 3c = 69 circle. onechord r 20 cm long and the other is 27.5 cm long. The
straight line distance between the remote ends of the chords is
The codrdinates for *
are as f o l l ~ ~ s :
points on each of hosfaight line graphs 12.5 cm. Given that for a circumscribed circle
graph: X I = - 4,y, = 6 and x2 = 2, y2 = -10
--a - b - c = D
second graph: x3 = -5, y3 = - 10 and x, = 2 , y4 = q
in^ SinB SinC
(a) Draw the straight line graphs using the same axes.
Cb) Using the graphs determine: is be diameter of the circle, calculate the dime'eK of the
(i) Ihe algebraic expression for each straight line in the form where
y=ax+b: circle.
Oi) the Of x and Y which satisfy both expressions when
considered as simultaneous equations. 85. The sketch (overleaf) shows a diesel engine unit which has a stroke
Suggested scales x axis I unit = 2 cm of 300 mm a connecting rod AB of length 550 -. Between
Y axis 1 unit = 1 cm the venical and the position of the connecting rod is an angle CAB.
Twice dunng the downward movement Of the piston angle CAB

i
A tower 50 high s m d s on the top of a hill which has a 10". ~ ~ t ~ the & piston
e travel b e m e n these piston
The angle of depression from the ho,.jzonral hrn the top of positions.
416 R E E D ' S MATHEMATICS FOR ENGINEERS SELECTION OF EXAMINATION QUESTIONS 417

Determine the ratio by mass of the smaller cone which was removed
to the mass of the remaining frustum.

89. A glass vessel has a uniform circular bore and a hemispherical end
of the same radius. With the hemispherical end at the bottom the
container is filled to 0.7 of its overall height. When the container is
inverted the level of the liquid drops down to 0.6 of the vessel's
height and is below the hemispherical section.
The vessel contains 100 ml of water. Determine:

B (b) the overall height.

90. Determine the volume of the solid of revolution formed when the
shaded area shown is rotated about the x axis.
2 2
86. (a) For the function y = - - -:
4 2
(i) show that there are two points on the curve where the
gradient is zero;
(ii) determine the values of x and y at the point where the curve
is a minimum.
(b) Determine the second differential coefficient of
y = 4cosx - 3 sinx.

87. (a) Using integral calculus determine the area bounded by the curve
y = 4 cosx and the x axis between the ordinates
71 71
x = - andx=-
9 2
(b) A curve passes through the point x = 3 y = -2 and has a
gradient of 2.x? - 2.
Determine the equation of the curve.

88. (a) Determine the mass of the frustum of a cone made of wood of
density 0.9 g/cm3 given that the diameters of the two paralleled
surfaces are 20 cm and 40 cm and the vertical height of the
frustum is 16 cm.
(b) The frustum was made by removing a smaller top cone base
diameter 20 cm from the large original cone, base diameter
40 cm.
-

L U T I O N S TO E X A M I N A T I O N
QUESTIONS

y=f(x)=lnx+sinx
1
fl(x) = - COSX
X
+
f )(x) = x-I + COSX
ffl(x) = - x - ~ - sinx

f "(x) = - (sinx + 9 Ans. (b)

'difference between two squares', similar to


+
a 2 - b2 gives (a b)(a - b)
the given expression:
420 R E E D ' S MATHEMATICS FOR E N G I N E E R S

x cancels therefore it can have any value. Ans. (a)

1
.'. 2 + - =2 7 Ans. (b)(i)
x
Squaring both sides, I sin 8 = - sin (240" - 180") = - sin 60" = -0.866
Area of segment of angle 8,
= 0.5?(8 - sin 8)
1
x4 + - - 47 Ans. (b) (ii) = 0.5 x 5'[4.19 - (-0.866)]
A
! -
= 12.5 x 5.056 = 63.2 cm2

Wetted perimeter of pipe surface

Taking square root of both sides,


1
x+-=3
X
I Hydraulic mean depth
63-2
Multiplying throughout by x, - -3~017cmor30~17rnm Ans.
20.94

or,
2+1=3x
=O 4. W= Iv2

VI
p dV where W is work done

Solving this quadratic,


x=2.618 or 0.382 Ans. (b) (iii) = I,.,7
6.5 120 dV

v]!::
= 120[ln
120[ln6.5 - In 0.51

= 1201n13
= 120 x 2.565
W = 307.8 Nm Ans.

5. By adding x 1 of water, the original 1 1 of the mixture is increased


422 R E E D ' S MATHEMATICS FOR E N G I N E E R S

LITRE
I
WATER
04t WATER 11
I+X
LlTRES
=3&--
2b3
x Ans. (b)

ALCOHOL
- I

15x= 3
x = 0.2 1 or 200 ml Ans. (i)

% water in new mixture =


+
0.68 0.2
x 100
+
1 0.2

= 73; --
Av4 - 2 3
2
+C Ans. (dl
% solids = remainder = 100 - (734 + 15)
= 113

1
.'. New anlaysis is : Water = 73; %
Alcohol = 15% Ans.
Solids = 1 1 $ %

du
-
3C2
& --R g - - Ans. (a)
64L
424 R E E D ' S MATHEMATICS FOR E N G I N E E R S S O L U T I O N S TO E X A M I N A T I O N Q U E S T I O N S 425

y = 11 + 7x - 2\? (see sketch) I Area of annulus between top of cylinder and base of cone,
= 0.7854 x (d- d 2)
= 0-7854(6 + 5.25)(6 - 5.25)
For turning points,
= 6.628 cm2 ... . . . (ii)
Curved surface area of cylinder
= ndh

=nx6x4
From the graph, it is obvious that x = 1: gives a maximum value
of y. = 75.41 cm2 ... ... (iii)

Total surface area exposed = 42.96 + 6.628 + 75.41

= 124.998

say 125 cm2 = 12 500 mrn2 Ans.

8. 4
Radius of cone base = x 5.25 = 2.625 cm
Slant height of cone = J4.52 + 2.6252 = 5.21 cm
Curved surface area of cone = nrl
t = 4s i.e. minus time inadmissable

s = 43 - 5.5 x 42 - 4 x 4 + 68.5

= 13 m/s2
Time = 4 s
1
Distance travelled = 28.5
Acceleration = 13 m/s2 I when velocity is zero. Ans. (a)
426 R E E D ' S MATHEMATICS F O R E N G I N E E R S SOLUTIONS TO EXAMINATION QUESTIONS 427

2.2- 3 2 + 2x+ 4
2 - 3x+ 5
= J ( 3 ~ -~ 312 )~dr
- 2x312 -x3
- +c
+ 2x5 - 3x4 + 22 + 4x2
= 2&-x3 +C Ans. (b) (i) 2x5 - 9x4 + 2 1 2 - 17x2 - 2x + 2 0 Ans. (i)

32-2+3x-4)6?+ x4+2..?-42-7x+4(22+x- 1
= J (2xp3+2)dr sxS- 2x4 + 6x3 - 8 2 Ans. (ii)
3x4 - 4 2 + 4 2 - 7x
+2x+C
- -x-2
+
3x4 - 2 3 2 - 4x
1
= 2x - -
x2
+C Ans. (b) (ii) -32+ 2-3x+4
-32+ ?-3x+4

(2a2b13x (3ab2)' = 8a6b3 x 9a2b4


= 72a8b7 Ans. (iii)

(3ab213t (ab3)' = 27a3b6+ a2b6


--
27a3 b6
-
a 2 b6
= 27a Ans. (iv)

tane = C O S ~
sin 8
-= cos 8
cos 8
sin8 =cos 2 8

I From the standard identity,

=6m Ans. (i)


Area of one sloping triangular flat surface,
=;XBEXAD I Substituting for cos2 8 and simplifying:
Total surface area of roof = 4 x x 24 x 13.42 sin 8 = 1 - sin2 8
= 644.2 m2 Ans. (ii) sin 2 8+sin8- 1 = O
430 R E E D ' S MATHEMATICS FOR E N G I N E E R S

this is the circumference of the cone base, let its diameter=d, ultiplying (vi) by 2 and then subtracting from (v),
2x+10y= -12
2x- 2y= 12
150 x 2 x II x 10.2 12y = -24
d=
360 x n y=-2
= 8.5 cm = 85 mm Ans. (ii)
slant height of cone = radius of sector = 10.2 cm substituting value of y into (vi),
radius of cone base = x 8.5 = 4.25 cm x+2=6
perpendicular height = 4 1 0 . 2 ~- 4.252 x=4

= 9.27 cm = 92.7 mm x = 4, and y = -2 A ~ s .


Ans. (iii)

17. Area of elliptical section = 0.7854Dd


(i)
= 0.7854 x 0.3 x 0.2 m2
(ii)
Multiplying (i) by 7 and (ii) by 19, Centre of ellipse from centre of wheel
= 1.8 - 0.15 = 1.65 m
(iii)
(iv) By Theorem (Pappus), in one revolution of the elliptical sectional
area about centre of wheel:
Subtracting (iv) from (iii),
Volume swept out = area x distance its centroid moves
Volume of rim = 0.7854 x 0.3 x 0.2 x 2n x 1.65

Substituting value of b into (ii),


I Mass = Vp = 0.4887 x 7.21 x lo3
m = 3523 kg Ans.
2 2
y = - - - (see sketch).
18. 4 2
dy 3 2
Gradient = - =- - X A"S. (i)
dx 4

Substituting values of a and b,


1oy + 12 = -2.x For zero gradient,
-6+x=y 32 x= 0
--
4
432 REED'S MATHEMATICS FOR E N G I N E E R S SOLUTIONS T O EXAMINATION QUESTIONS 433

I +
Factors are: (x - y)(3? 1 lxy + 10f)
+ +
= (X - Y ) ( ~ x 5y)(x 2y) Ans. (iv)

. . x = O or x = 43
i.e. Gradient is zero at x = 0 and x = $ Ans. (ii)
When x = 0,
d2Y
-= - 1 (i.e. negative)
dX2
When x = $,

9
dr2
= 2 - 1 (positive)

:. y is a minimum at the point x = 1 f. Ans. (iii)


The solution is confirmed by the sketch of the graph.

19. (i) p-2=2(?-1)=2(x+l)(x-1) Ans. (i)


(ii) 7x2 + 9xy - 10J = (7x - 5y)(x + 2y) ~ n s (ii)
.
(iii) 14x3J + 2 2 2 9 12q4
-

= 2Xy2(7? + 11xy - 6 y )
= W ( 7 x - 3y)(x+2y) Ans. (iii) 21. (a)
(iv) Dividing first by the given factor:
-1
i.e. y z ~ -? 5 ~ + 7 - X

= [2 sin4S0]- [2 sin 0°]


A = 1.414 units2. Ans. (a)

=2~0~~-3sinx

y=x3 -4x+c
when x = l , y = -2, mereare 7 cross-sectional areas, therefore 6 spaces.

C=l
. The equation is y = 2 - 4x + 1 h s . (b) 11.6
7.6
22. (i) See Chapter 5 for proof of formula. Ans. (i) 16.4
7.4
2 + 5x = 84 10.8
2 + 5x + 2.52 = 84 + 2.52 -1.1
sum = 54.9
x+2.5=&2/-
x+2.5=f-
x = f9.5 - 2.5 Common internal = length t no. of spaces
x = + 7 o r - 12 Ans. (ii) =8cm+6
@-3ax-bx=O
x(2x - 3a - b) = O i
volume = sum of products x c o w o n internal
Either x = 0 54.9 x 8
- = 24.4 cm3 Ans. (1)
or, 2 x - 3 a - b = 0 3x6
2x=3a+b m=Vp
= 24.4 x 8.4
Mass = 205 g Ans. (ii)
436 R E E D ' S MATHEMATICS FOR E N G I N E E R S

25. C=a+bN
Inserting given values,
+
8.5 = a b x 300 . .. .. . .,.
+
7.5 = a b x 200 . . - ... .., (ii) 1
1.0 = 100b by subtraction secondarea =(?) -
- - 16
(lhh+?)
b = 0.01
Substituting value of b into (i),
dA
- -- - -1+ -x
+
8.5 = a 0.01 x 300
dx 8 4
a = 8.5 - 3 0 = - 1 + 2x for maximum or minimum
a = 5.5
.: cost equation is:
d2A 1 .
+
C = 5.5 0.01 N pounds Ans. (a) - = - 1.e. positive, as a minimum
dx2 4
VJhen N = 150, 1
+
C = 5.5 0.01 x 150
= £7.00 Ans. (b) 1
A = -m2, least possible sum of the two areas Ans.
Minimum cost is when machine is running idly and no articles are 32
being manufactured,
+ 0.01 x 0
When N = 0, C = 5.5
C = 5.5
.'. minimum cost = £5.50 Ans. (c)
1 28. Solid plate without groove:

I
y = sinx + -cosx
2

At bottom of groove:

Major axis = 42 - (2 x 1.3) = 39.4 cm


Minor axis = 37 - (2 x 1.3) = 34.4 cm

Cross-sect. area of plate at bottom of groove

= 0.7854 x 39.4 x 34.4 = 1064 cm2

Volume of groove

Area = 0.549 units2 Ans.


I 2.
438 R E E D ' S MATHEMATICS F O R E N G I N E E R S S O L U T I O N S TO EXAMINATION Q U E S T I O N S 439

Net volume of grooved plate Area of cross section = average width x depth
= 5490 - 202.8 = 5287.2 cm3
= $ ( I S + 10) x 14
= 175 m2
Mass = 5287.2 x 7.86 x lop3
6
= 41.56 kg Ans. Length of arc = 3-
60 x full circumference

-
--60
I x 2x x 805 m

By Theorem of Pappus,
Volume swept out = area of section x distance centroid moves
1
= 175 x -60 x 2x x 805
= 14750 m3 Ans.

Referring to sketch, working in cm:


?=4'+2
=16+2 ......... (i)
a = 32 + ( x + q2
=9+2+2x+1 =32+52-2x3x5xcos20C

= 10+2+2x . . . . . . . . . (ii) = 9 + 25 - 28.191

:. l 0 + 2 + 2 x = 1 6 + 2 a =6363
2x=6 = 2.41 cm = 24.1 rnm

x=3 - b
a =-
sinA sin B
From (i), 3 = 4' +2 2.41 3
r= Jm -=-
sin200 sin B
0'342
r = 5 cm = 50 mrn Ans. sin B = = 0.4257
2.41

. AngleB=2S012'
Angle C = 180" - (20" + 25"12')
= 134O48'
bc sin A
Area = -
2
440 R E E D ' S MATHEMATICS FOR E N G I N E E R S S O L U T I O N S TO EXAMINATION Q U E S T I O N S 44 1
Remaining side = 24.1 m m
Remaining angles = 25"12' and 134'48'
Area = 2.565 cm2 Calculating plotting points for this curve:

Being a straight line, two values only are required to plot this graph:

The graphs are now plotted as shown from which values of x and y
are read thus:

x = 2.64 and y = 5.96

or x = -1.14 andy= 0.3


] hs.
442 REED'S MATHEMATICS FOR ENGINEERS SOLUTIONS TO EXAMINATION QUESTIONS 4 3
Common denominator is 2(x - 1)(3x - 4),
Multiplying throughout by this:

A, = 54.42 units2
A = A, -A2 is area between the two curves
= 556.5 - 54.42 Solving by formula,
2
= 502.08 units Ans.

(i) When the body is stationary, velocity = zero,

I =2 or 1.353 Ans.

The body is stationary when t = 1 and t = 3s Ans. (i)


(ii) when t = 1,
acceleration = 6 - 12
= -6 m/s2 Ans. (ii)
when t = 3,
acceleration = 18 - 12
= 6 m/s2 Ans. (ii)
444 REED'S MATHEMATICS FOR ENGINEERS
- S O L U T l O N S TO EXAMINATION QUESTIONS 445
Area of circle = n?
$1: "'2 -2)d"

Angle at apex of triangle = 180" - (2 x 75") = 30°


= '[A
2r3 -:Ir 0

Angle at centre of circle for a triangle on the same base is twice 2[(?
2r3 )-: -01
angle at apex,
= 1 .O Ans. (b) (ii)

f
8 38. See Chapter 7 for proof of sin 8 + cos 8 = 1 Ans. (i)
2 2
Referring to sketch,
i? 2sin 2 8 - 2cos2d - 2 = 0
Angle COB = 30"
CB=rsin30° = 1 6 x 0 - 5 = 8 c m
'
2f
sin2 8 - cos2 8 - 1 = 0
substituting, cos2 8 = 1 - sin2 8,
Base of triangle = 2 x 8 = 16 cm
OB = rcos30° = 16 x 0.866 = 13.856 cm sin2 8 - (1 - sin2 8) - 1 = 0
+ +
AB = A 0 OB = 16 13.856 = 29.856 cm sin2 8 - 1 + sin2 8 - 1 = 0
i
Area of triangle = (base x perp. height) 2 sin2 8 - 2 = 0
= !j x 16 x 29.856 2 sin2 8 = 2
= 238.8 cm2 Ans. sin2 8 = 1
sin8=&=fl
37. (a) (i) . 8 = 90" and 270" Ans. (ii)
dy
- = E - 2Ri Ans. (a) (i)

I
di
39. See Chapter 8 for proof of formula. Ans.

Sumofsides=5+6+7=18cm
Semi-sum = 9 cm
dy C(1-n) -n
-=- v
dv 1-n Area of triangle = Js(s - a)(s - b)(s - c)
C =4 9 x 4 ~ 3 ~ 2
=- Ans. (a) (ii)
v"
= 14.7 cm2
2 x area of triangle
Radius of circle =
perimeter of triangle

= [1.5x8]: --2 x 14.7


- 18
= 1.5 Ans. (b) (i) = 1.633 cm = 16.33 mrn Ans.
446 R E E D ' S MATHEMATICS FOR E N G I N E E R S S O L U T I O N S TO E X A M I N A T I O N Q U E S T I O N S 447

Curved surface area of hemisphere


= surface area of circumscribing cylinder
= n x diameter x height
=n x 10 x 5 = 157.1 cm2
Total surface area = 175.6 + 157.1 = 332.7 cm2 Ans. (ii)
Volumes of similar objects vary as the cube of their corresponding
dimensions. As the large object has linear dimensions twice
that of the smaller, then:

Volume of larger solid = 523.6 x 23


=4188.8cm3 Ans. (iii)
d2y .
x = 1.5, - is positive i.e. minimum Areas of similar figures vary as the square of their corresponding
dr2 dimensions, therefore:
d 2y .
x = -1.0, - is negative i.e. maximum Surface area of larger solid = 332.7 x 22
dr2
= 1330.8 cm2 Ans. (iv)

42. y=2-8x+7 . . . . . . . . . (i>


y = -20.25, (x = 1.5), minimum ) Calculate plotting points for this equation:

y = 11.00, (x = - 1.0), maximum 1


4
Volume of cone = (area of base x perp. height)
A sketch of the graph is as shown.

1 n
Volume of hemisphere = - of -d 3
2 6

Total volume = 261.8 + 261.8 = 523.6 cm3 Ans. (i)


Curved surface area of cone
= nr x slant height
REED'S MATHEMATICS FOR ENGINEERS SOLUT IONS TO EXAMINATION Q U E S T I O N S 449

(a) To solve the equation 2 - 7x + 6 = 0 : Multiplying (i) by 3, and (ii) 47

From equation (i), by adding x - 1 , 40 224 = 48a + 12 288b ...... (iii)


X2-8~+7+x-l=X2-7~+6
24 096 = 48a + 69 12b ...... (iv)
:. 2-8x+7+x-1=0 subtracting (iv) from (iii),
2-8x+7=1-x 16 128 = 5376b
:. y = l - x b=3
Substituting,b = 3 into (ii),
Calculate plotting points for y = 1 - x
6024 = 12a 5184 +
x=O,y= 1 12a = 840
x = 9 , y = -8 a = 70
The straight-line graph for this equation is also shown. .. Lawis,p=7ov+3v3
At the points of intersection, x = 1 , and x = 6. Ans. (a)
inserting v = 14 knot and evaluating,
(b) To solve the equation 2 - 9x + 8 = 0 p = 70 x 14+3 x 1 4 ~
Adding -x +
1 to equation (i): = 980 + 8232
p=9212kW Ans.
i.e. ? - 8 x + 7 - ~ + 1 = 2 - 9 ~ + 8
2-8x+7-x+1=0
2-8x+7=x-1
... y = x - 1 y'T-3 '+32-&
Calculate plotting points, x3
: Y =- - 3 ~ - 3 ~2
3
+ - 2x
x=o, y = -1
x=9, y=8 dy=2+ 9 y 4 + 6 x - 2
dx
The graph is shown. At the points of intersection between the graph
and the c u r v e y = 2 - 8x+7. 6Y
dX2
= &- 36x8 + 6

x=landx=8 Ans. (b) 36


= & - - +x5
6 Ans. (i)
P = V(a+ bv2) y =3sinx+4~0sx
(ii)
or, P = uv + bv3

Inserting given values to form simultaneous equations: dy = 3 cosx - 4 sinx


dX
13408 = a x 16 +b x 1 6 ~ ......
6024 = a x 12 + b x 1 2 ~
- - -

450 REED'S MATHEMATICS FOR ENGINEERS

45. Referring to the cone (see Fig. 85):


- SOLUTIONS TO EXAMINATION QUESTIONS

From equation (iv),


45 1

Radius of base = 1.22 m d=9-2a


Perp. height = total height - height of hemisphere
Substitute d = 9 - 2a in equation (iii),
= 4.27 - 1.22
a + b + c + 9 - 2 a = 14
= 3.05 m
:. - a + b + c = 5 . . . . . . . . . . . . (v)
Slant height = J3.052 +
1 .222 = 3.285 m
Curved surface area = nrl To eliminate c, subtract (v) fiom (ii),
= x x 1.22 x 3-285 2 a + 6 b + c = 18
= 12.6 m 2
Ans. (i) - a+ b + c = 5

The slant height is the radius of the developed sector, angle 3a+5b = 13 ......
subtended at centre Multiply (v) by 2 and subtract fiom (i),
- arc of sector
- x 360"
circurn. of whole circle

. . . . . . . . . . (vii)
= 133.7" Ans. (ii)
To solve: subtract (vii) from (vi),
Curved surface area of hemisphere
= curved surface of circumscribing half cylinder
= $ nd2
= x n x 2.442
= 9.354 m2 Ans. (iii)

+ vol. of hemisphere
I Substitute a = 1 in (vii),
Total volume = vol. of cone
I 5 + 5 b = 15
1 n
= f (area of base x perp. ht.) + - x - d 3
2 6 I . b=2
Substitute a = 1, b = 2 in (v),

= 4.755 + 3.804 -1+2+c=5


= 8.559 m3 Ans. (iv)
Substitute a = 1 in (iv),
+ +
3a 7b 2c = 25 . . . . . . . . . . . . (1)
2 a + 6 b + c = 18 . . . . . . . . . . . . (ii) 2+d=9
a + b + c + d = 1 4 . . . . . . . . . . . . (iii)
2 a + d = 9 . . . . . . . . . . . . (iv)
48. (a) :. Volume=40~212units3 Ans. (b)
454 REED'S MATHEMATICS FOR ENGINEERS SOLUTIONS TO EXAMINATION QUESTIONS 455
44 1
105 x -=
105 x
&= 44
1052 x 174.4
12 =
442
= 993 mm Ans.

5cosx-&siny=-3.5 . . . . . . . . . (ii)
+
15 cosx 20 sin y = 6.64 . . . . . . . . . (iii)
15 cosx - 4.242 siny = -10.5 . . . . . . . . . (iv>
z
.:
.3
-= constant
Multiplying (i) by 5, Muliplying (ii) by 3.
z
Subtracting (iv) from (iii).
x cx
24.242 siny = 17.14
siny = 0.7071
y = 45"
Let each of the original values of x, y and z be represented by unity,
then: Substituting siny = 0.7071 into (1)
new value of y = 1 - 0.2 = 0.8 3 cosx + 2.828 = 1.328
new value of z = 1 + 0.12 = 1.12 3 cosx = -1.5
cosx = -0.5
x = 180" - 60" = 120"
x = 120°,y=45" Ans.
Fractional increase in x = 1.75 - 1 = 0.75
Percentage increase = 0.75 x 100 = 75 Ans. (i)

time cx Ja time
.'. --
45@ -

44
time for one beat of 1st pendulum = - s
105
time for one beat of 2nd pendulum = 1 s
456 R E E D ' S MATHEMATICS FOR E N G I N E E R S

19
Vol. of frustum = - x vol. of complete cone
27
Let volume of complete cone = 27 = [3 sin 180°]- [3 sin 90'1
Let volume of complete frustum = 19 = -3 Ans. (b)(i)
then volume of top cut off = 27 - 19 = 8
Volumes of similar objects vary as the cube of their corresponding
dimensions
. Vol. of complete cone - ~ e i ~ ofh complete
t ~ cone
.. -
3
Vol. of top cut off height of top cut off
= [-9 41 - [-2+]
27 H3
-- -- 3 = -6; Ans. (b)(ii)
8 h
h=Hx-
32/8
3
m
2
h=- H
3
.'. Height of frustum = f H Ans.
= 41.1 - 4.7
= 36.4 Ans. (b)(iii)

2.

1 - Firstly, take square root of both sides to get rid of the power.
I
Secondly, multiply both sides by the least common denominator to
I I eliminate fractions. Then simplify and proceed to get terms
I I containing Von one side with all other terms on other side of
1 I
O z Y 5r
F
&=-
V+v
v- v
Area =
6(2 sin 8) d8
5n
= [-2 cos 01;
&(v-v)=
&v-JKv=v+v
&v-v=&v+v
v+v

= [-2 x cos 150'1 - [-2 cos 30'1 v(& - 1) = v ( f i


1) +
= 1.732 + 1.732 a + l
v = v------
Area = 3.464 units2 Ans. (a) a- 1

- -----
---
458 R E E D ' S MATHEMATICS FOR E N G I N E E R S SOLUTIONS TO E X A M I N A T I O N Q U E S T I O N S 459
When the area bounded by the graph is swept one complete
revolution about the x-axis, they ordinates of the graph become the
radii at regular intervals along the length of the solid swept out.
Putting the cross-sectional areas of these radii through Simpson's
rule:
Inserting values,
CROSS-SECT.
RADII AREAS SIMPSON'S
r 7
13 MULTIPLIERS PRODUCTS
0 ITXO 1 0
5 IT x 25 4 71 x 100
8 IT x 64 2 71 x 128
9 IT x 81 4 71 x 324
8 IT x 64 2 IT x 128
5 IT x 25 4 71 X 100

Values of V are, 0 ITXO 1 0


sum=n x 780
- 198 and - 2.02 Ans.
+
55. Plotting points for y = 2 2x - 8 : Common intervals between ordinates
x -4 -3 -2 -1 0 +l +2
y 0 -5 -8 -9 -8 -5 0 =1
Volume generated = IT x 780 x $x 1
3
= 816.9 units Ans.

56. (i)

for maxmor min.m values,


460 REED'S MATHEMATICS FOR ENGINEERS

.'. For a max." or min." value of A,


I SOLUTIONS TO EXAMINATION QUESTIONS 46 1

BC = J(Ac)~ - ( m 1 2 = d i i
x = 20
= = 2.236 cm
d2A 16000
Differentiating again, -= 2
dX2
+ - x3 h = radius of 2 cm ball + BC + radius of 4 cm ball
16000
=2+- (i.e. positive) =1 + 2.236 + 2 = 5.236 cm
203
Internal vol. of cylinder up to height h
hence, when x = 20, A has a minimum value. Ans. (i).
(ii) For minimum value, = 0.7854 x d 2 x h

= 0.7854 x 62 x 5.236 = 148 cm3

:. Minimum area = 1200 units2 Ans. (ii).


n
Volume of sphere = - d 3
6
n
Voluine of 3 small balls = 3 x - x 23
6
n
Volume of the large ball = - x 43
6
n
Total volume of balls =,(3 x 23 + 43)

k2xJ-J ELEVATION

Volume of space for water to fill

= vol. of cyl. - vol. of balls

= 148 - 46.08

= 101.92 cm3

Volume of water = 101.92 ml Ans. (i)

If all linear dimensions were half of those above,


Referring to triangle ABC in elevation of sketch 1,
Volume required = (;l3 = $ of above
I +
AC = radius of 2 cm ball radius of 4 cm ball
I
=1+2=3cm because similar volumes vary as the cube of their corresponding
AB = radius of cyl. - radius of 2 cm ball linear dimensions.
I

I =3-1=2cm Volume = $ x 101.92 = 12.74 m1 Ans. (ii)


1
462 R E E D ' S MATHEMATICS FOR E N G I N E E R S S O L U T I O N S TO E X A M I N A T I O N Q U E S T I O N S

Remaining piece of pentagon


= 21.4 - 4.421 = 16.979 cm (ii)
Ratio of (i) and (ii)
= 4.421 : 16.979
Dividing both by 4.421
Ratio = 1 : 3.84 Ans. (b)

59. (a) 2sin2Q-COSQ


=1
From the identity,
sin2 Q + cos2 8 = 1
Pentagon is composed of 5 isosceles triangles, sin2 Q = 1 - cos2Q
Angles at centre = 360" t 5 = 72" Substitute for sin2 8 into original equations and simplify,
5
8 = of 72" = 36"
2sin 2 8 - cos8 = 1
h = 3 x cos 36" = 2.427
2(1 - cos2 8) - cos 8 = 1
b = 2 x 3 sin36" = 3.527
2 - 2 ~ o s ~ 8 - c o s 8 =1
Areas of isosceles triangle - ~ C O S ~ $ - C O S1O=+o
-
- Ibh
2 2c0s28+cos8- 1 = o
= x 3.527 x 2-427
Factorising,
= 4.28 cm2
Area of pentagon = 5 x 4.28
(2 cos Q - l)(cos Q + 1) = 0
= 21.4 cm2 Ans. (a) Either 2 cos 8 - 1 = 0, then cos 13 = 0.5
Angle C = 2 x (90" - 8) = 108" or cosQ+ 1 =O,then c o s Q = -1
AngleB= 180"-A-C=42"
When cos Q = 0.5,8 = 60" and (360" - 60") = 300"
When cos8 = -1,8 = 180"

3.527 x sin 30" .'. Angles are 60°, 180" and 300" Ans. (a)
a= = 2.636 cm
sin 42" (b) 3cos2Q+2cos8 = 0
4
Area of triangle abc = ab sin C 3(2cos 2 8- 1)+2cos8 = O
5
= x 2.636 x 3.527 x sin 108" 6~os~~-3+2cos~=0
. . (i) 6cos26+2cosQ-3 = O
-
464 REED'S MATHEMATICS FOR ENGINEERS SOLUTIONS T O EXAMINATION QUESTIONS
465
Solving by quadratic formula, Taking two points on the line such as:

- b f 4- when v2= 400, R = 85


cos 0 =
2a when V 2 = 100, R = 25

I Inserting these into R = k + c

+
85 = k c x 400
~ 2 ,

. . . . . . . . . (9
25 = k + c x 100 . . . . . . . . . (ii)
60 = c x 300 by subtraction

Whencos 0 = 0.5598,O = 55" 57' and 360" - 55" 57' = 304" 3'
When cos 0 = -0.8932,O = 180" - 26" 44' = 153" 16' From (i) +
85 = k 0.2 x 400
and 180" + 26" 44' = 206" 44'
k=85-80=5
Angles are, Constants are,
55"57', 153"16', 206'44' and 304"3' Ans. (b) k = 5 and c = 0.2 Ans. (i)
when V = 40,
R = k+cv2
=5 + 0.2 x 402
= 325 N Ans. (ii)
By plotting the points of R and v2,they lie on a straight line,
+
therefore the equation is of the form y = a bx where x represents
v2.

2 2
P2 - V =$-$+k~+kv
P - v2= 2kV
p2 - v2
--2V -k
p2 - V 2
or, k = ------ Ans. (i)
2v
466 REED'S MATHEMATICS FOR ENGINEERS SOLUTIONS TO EXAMINATION QUESTIONS 467

When p = 9.2 and V = 0.42, As an alternative to the latter part, the volume can be found by the
Theorem of Pappus:
3
Area of triangle = x 5 x 8.66 = 21.65 cm2
- (9.2
-
+ 0.42)(9.2 - 0.42) ~entroidfrornaxis=5+$~5=$~5crn
2 x 0.42 Area of rectangle = 5 x 8.66 = 43.3 cm2
- 9.62 x 8.78
-
0.84
i
Centroid from axis = x 5 = 2.5 cm
= 100.55 Ans. (ii) Volume swept out = area x distance centroid moves
Volume swept out by triangle = 21.65 x 271 x $ x 5 = 907 cm3
Volume swept out by rectangle = 43.3 x 2~ x 2.5 = 680 cm3
+
Total volume = 907 680 = 1587 cm3
(as previous)
As a further alternative, volume of frustum may be calculated from
volume of whole cone minus volume of top cone removed. All
these methods are explained in the text.
63. (a) 4a+3b+2w=2 . . . . . . . . . . . . (i)
2 a + 8 b + 3 w = 11 . . . . . . . . . . . . (ii)
The length of each non-parallel side being equal to the radius, the 3a + 5b - 6w = -36 . . . . . . . . . . . . (iii)
trapezium is composed of three equilateral triangles, therefore angle
Proceeding to eliminate w:
at the base is 60°, length of short side of trapezium = 10 cm, length
Multiply (i) by 3 and (ii) by 2 and subtract,
of long side = 2 x 10 = 20 cm, and h = 10 sin 60" = 8.66 cm.
i +
Area of trapezium = (a b) x perp. ht.
3 +
= (10 20) x 8.66
= 129.9 cm2 Ans. (a)
Multiply (i) by 3 and add (iii),
Alternatively, area of this trapezium
12a+ 9 b + 6w = 6
= area of 3 equilateral triangles 3a +5b - 6w = -36
2
= 3 x 0.433 x side
= 3 x 0.433 x lo2 Proceeding to eliminate b and find the value of a:
= 129.9 cm2 as above Multiply (iv) by 2 and add (v),
Volume of frustum = $ R ~ ( R Rr+ +
~ 9)
= $ x 71 x 8.66(102 10 x 5+ + 52)
= $x R x 8.66 x 175
= 1587 cm3 Ans. (b)
468 REED'S MATHEMATICS FOR ENGINEERS S O L U T I O N S T O EXAMINATION QUESTIONS 469

Substitute value of a into (iv) to find b, 64. (a) Simplifying left hand side:
8 x (- 2)- 7b = -16
-16 - 7b = -16 sin2 8
1 - tan2 1 --
-7b = o - COS~
2
e
b= 0 sin 8
l + a
Substitute values of a and b into (i) to find w,
4x(-2)+3 xO+2w=2 cos2 e - sin2 e
- c0s2 e - cos2 0 - sin2 8
-8+0+2w=2 -
+
cos2 0 sin2 8 - cos2 0 sin2 e +
2w = 10 COS~ e
w=5
2
Values of a, b and w are, respectively, - cos2 0 - sin 6' = cos2 8 - sin2 0 = r.h.s. Ans. (a)
-
1
-2,OandS Ans. (a)
(b) Simplifying left hand side:
(b) 1+? = 18.5 . . . . . . . . . . . . 6)
x-y=l . . . . . . . . . . . . (ii)
tane
From (ii), y = x - 1, substitute this into (i), sin 8 + sin 8 cot2 0 + -
cos e
1 + ( x - 112 = 18.5
1 sin 6'
1 + 1 - h + 1 =18.5 = sine + sin8 x -
tanze+cOse case
P-h-17.5=0
1-x-8.75=0 1.h.s. = sin8 + sin 8sinx 2cos2
0
+-cos2 8
0 sin 8

Solving this by quadratic formula,


- b f4 K T Z cos2 0 sin 0
x= =sine+-+-
2a sin8 cos2 0

- sin
-
2
+
8 cos2 0 cos4 8 + sin2 0
sin 8 cos2 0

-
+
cos2 e(sin2 8 cos2 0) + sin2 0
From (i), y = x - 1 sin 0 cos28
I f x = 3.5,y= 3.5 - 1 = 2.5
If x = - 2 . 5 , ~= -2.5 - 1 = -3.5 - cos2 6' x 1 + sin2 8
-
sin 6' cos2 0
Therefore, x = 3:5 and y = 2.5 1
or, x = -2.5 and y = -3.5 1.h.s. = = r.h.s. Ans. (b)
sin 8 cos28
R E E D ' S MATHEMATICS FOR E N G I N E E R S

+
4y - x - 20 = 0, .'. y = 5 0 . 2 5 ~
When x = -2, y = 5 - 0.5 = 4.5
Whenx=+8,y=5+2=7
y - x + l = o , :. y = - l + x
Whenx=-2,y=-1-2=-3
Whenx=+8,y=-1+8=7
2y + X - 7 = 0, .'.y = 3.5 - 0 . 5 ~
When x = -2, y = 3.5 f 1 = 4.5
When x = +8, y = 3.5 - 4 = -0.5

The graphs are shown plotted and enclose a triangle. Let the lengths
of the sides of the triangle be a, b and c.
Graph forming side a rises 2.5 over a horizontal length of 10,

a= Jw=
10.31

Graph forming side b rises 5 over a horizontal length of 5, therefore,

b= JFZi= 7.071 0 E

Graph forming side c drops 2.5 over a horizontal length of 5,

c= JZ-52f52 = 5.59
Area of triangle = Js(s - a)(s - b)(s - c)

A C B
s = i(10.31 + 7.071 + 5.59)
= 11.486 Hexagon is composed of 6 equilateral triangles with all angles of
60°, each side equal to unity.
Area = d11.486 x 1.176 x 4.415 x 5.896
OC = OA sin 60" = 1 x 0.866 = 0.866
= 18.75 units2 Ans. Area of triangle OAB = x 1 x 0.866 = 0.433
472 RE E D ' S M A TH E M A T I CS F O R E N G I N E E R S S O L U T I O N S TO E X A M I N A T I O N Q U E S T I O N S 473

(or obtain this from the formula, area of equilateral triangle y = e-a
2
= 0.433 side ) 1
ea
1.e. y =-
Area of hexagon = 6 x area of triangle
= 6 x 0.433 = 2.598 when x = O ~ ~ 1 . 0
Area to be cut off is the shape of a trapezium consisting of a central x = 0.2 y = 0.67
rectangle and a triangle at each end. x = 0.4 y = 0.449
Let thickness D G be represented by x x = 0.6 y = 0.301
Area cut off = area of rect. + two triangles x = 0.8 y = 0.202
=DExx+2x~xHGxx x = 1.0 y = 0.135
=I x x + 1 x (xtan30°) x x x = 1.2 = 0.091
=x + 0.57742 x = 1.4 y = 0.061
and this is to be equal to 20% of the area of the hexagon, x = 1.6 y = 0.041
+ 0.57742 = 0.2 x 2.598
x x = 1.8 y = 0.027
0.57742 + x - 0.5 196 = 0 x = 2.0 y = 0.018
a quadratic equation, divide by coefficient of 2, SIMPSON'S
2 + 1.732 - 0.9 = 0 ORDINATES MULTIPLIERS PRODUCTS
Apply the formula 1.0 1 1 -0
0-67 4 2.68
-b f -
/, 0.449 2 0.898
x=
2a 0.301 4 1.204
-1.732 f 0.202 2 0.404
-
2 0.135 4 0.54
-1.732 f2.569 0-091 2 0.182
- 0.06 1 4 0.244
2
0.041 2 0.082
= 0.4185 Ans. (i) 0.027 4 0.108
(negative answer inadmissible) 0.018 1 0.018
OJ=OC-x Sum = 7.39
= 0.866 - 0.4185 = 0.4475 Area = f x common interval x sum of products
HJ=HG+GJ
=xtan30"+$side
= 0.4185 x 0.5774 + 0.5 = 0.7416 area
Mean ordinate = (see sketch overleaf)
OH = ,/(HI)' + (0~12 length of base
0.4907
= J0.7416~ + 0.44752
-
2
= 0.8662 Ans. (ii) = 0.245 units. Ans.
474 R E E D ' S MATHEMATICS FOR E N G I N E E R S S O L U T I O N S TO E X A M I N A T I O N Q U E S T I O N S 475
Y
Solving this quadratic by formula,
1

d=
-3
2
178'9

+
= 87.95 cm

D = 87.95 3 = 90.95 cm
} Ans.

68. By Pythagoras, refemng to section of cone, (negative answer is inadmissible).

Perp. height = J(s1ant ht.)' - (radius)'

i
Volume of cone = area of base x perp. height

71 3
This is a quadratic equation. Set down in the usual way and solve by
Volume of hollow sphere = - ( D ~
-d ) quadratic formula:
6
+
Substituting D = d (2 x thickness) = d + 3,
71
Volume = - {(d
6
+ 3)3- d ) 3

where, a = c, b = 2as, c = -2asd

Equating (i) and (ii),

-
-
-as f Jas(as + 2cd) Ans. (i)
C
476 R E E D ' S MATHEMATICS FOR E N G I N E E R S S O L U T I O N S TO E X A M I N A T I O N Q U E S T I O N S 477
Substituting numerical values: Refemng to plan view,
WB = AB sin A
= 60 sin 28'4' = 28-23 m
Referring to elevation on WE$,

= 5.947 or - 9.147 Ans. (ii)

Angle W = 37" 44' Ans. (d)

Plotting points for y = 2 + 1:


x - 2 - 1 0 1 32
A Y 5 2 1 2 5 1 0
ELEVATION EL EVATI O N
O N A@ O N WB The graph is shown.
Difference between equations
y=2+1
and0=g-x-2
Refemng to plan view, if the angle opposite the longest side is a
right angle, then by Pythagoras, the longest side is the hypotenuse
and the square of this side will be equal to the sum of the squares of
the other two sides.
( A C ) ~= 682 = 4624 . . . . . . 6)
+
(AB)~+ ( B C ) ~= 602 322
=3600+ 1024=4624 . . . . . . (ii)
(i) and (ii) are equal, therefore angle B is a right angle. Ans. (a)

Angle A = 28" 4'


Angle C = 90" - 28" 4' = 61" 56'
Refemng to elevation on AB,
Height of mast TB = AB t'anA
=60tan20° =21.84m Ans. (c)
481
480 REED'S MATHEMATICS FOR ENGINEERS

From (i), y = 10 - 4x, substitute into (ii),


- SOLUTIONS T O EXAMINATION QUESTIONS

2
- + ------ = 3
x 10- 4x
Multiply every term by least common denominator, which is
.'. 271F = 1&
x(10 - 4 4 , 4712~2 =-
Lk
4 n 2 ~ 2=~ 1k

"
k= - 1
4712F2L
Ans. (b)

By quadratic formula,

x=
- bf nzz
2a
-315 2/%r=im
24
= 14 or 1;
From (i),
~henx =l!,y= 1 0 - 4 x 1!=4$
~henx =l $ , y = 1 0 - 4 x 1 i = 5
x= 1: a n d y = 4 2
3 ) Ans. (b)
or, x = la a n d y = 5
.'. k =
+
4cL - 8 2 b - 2~71b
Ans. (c)
+
2c71 1
74. (a)

.'. k = Qs/2gh," Q2d Ans. (a)


482 REED'S MATHEMATICS FOR ENGINEERS -
SOLUTIONS TO EXAMINATION QUESTIONS 483

For the last 6 h, with a speed increase from V to xV:


C3 = k(xv13 per hour
.: y = (see sketch) For the 24 h,
Volume of revolution = 71
1: J&
8Cl + 10C2+ 6C3 = 24C2
8C1+ 6C3 = 14C2
sk(0.8 v ) ~+ 6k(xv13 = 14k(v13
4.096 + 6x3 = 14

% Increase in speed in the last 6 h is 18.2. Ans.


y+4*
=7fz 78. (a) squaring gives :
X 3 n e
3
V = 23.04 cm Ans. 2n+gX6n-7
X6nX2n- I simplifying gives;
76. (l+x)"=l+nx+-
n(n - 1)
2x1
*+n(n3 xl)(n2 ~ 2)1x3 approx.
- -
X8n+l
- simplifying gives
1
X8n+lx -8n+l giving z? Ans. (a)

1 1x3 1X3X5 by taking logarithms then:


= 1+ - x + 2
2 2x4
+ x3 approx. ~ n s .
2 x 4 ~ 6 xlog10 = (x-4)10g20
+ +
(1 - 0.1)-IJ2 = 1 + 0.05 0.00375 0.0003125 xlglO=(x-4)lg20 tobase10
= 1.0540625 A ~ s . x x 1.000 = (X - 4) x 1.3012
x = 1.3012~- 5.2048
x = 17.29 Ans. (b)

For the first 8 h, 79. (a) (i) &Pa7


C1 = k(0.8 v ) ~per hour
For the next 10 h,
C2 = k(v13 per hour
484 R E E D ' S M A THEM A TI C S F O R E N G I N E E R S

3,/216a-12b0.6
(a) (ii)
-
- 6a-4b0'2

-- 5A Ans. (a) (ii)


- a4
(b) a-b+c=3 . . . . . . . . . 6)
+
2a 8b - 3c = 56 . . . . . . . . . (ii)
+
7a + 3b 3c = 69 . . . . . . . . . (iii)
+
add 3 times (i) to (ii) 5a 5b = 65 . . . . . . (iv)
+
add (ii) to (iii) 9a 1 1 b = 125 ...... (4
+
9 times (iv) is 45a 45b = 585
+
5 times (v) is 45a 55b = 625
lob = 40 .'. b = 4
+
sub. in (i) times 2 2a 2c = 14
sub. in (ii) 2a - 3c = 24
5c=-10 :. c = - 2
sub. for b and c in (i) :. a = 9
a = 9, b = 4 , c = -2 Ans. (b)

80. (a) As plotted. Ans. (a)


(b) (i) Graph 2.
4
Slope, a = - = 2
2
Intercept, b = 0
Equation y = 2x Ans. (b) (i) Angle D ~ =
A 35"
Graph 1. DAC = 55" - 20" = 35"
-4 :. Triangle ADC is isosceles
slope, a = - = -2.67
1.5
Intercept, b = -4.65
Equation y = -2.67~ - 4.65 Ans. (b) (i) D ~ =B 50"
(b) (ii) Solution of both as simultaneous equations i.e. intersection 50
-------- = -
DB
of graphs is sin ABC sin D ~ B
x = -l,y=-2 Ans. (b)(ii) 50 x 0.766
DB =
0.342

You might also like